%%%%%%%%%%%%%%%%%%%%%%%%%%%%%%%%%%%%%%%%%%%%%%%%%%%%%%%%%%%%%%%%%%%%%% % % % Project Gutenberg's Introduction to Infinitesimal Analysis, by % % Oswald Veblen. % % % % % % This eBook is for the use of anyone anywhere at no cost and with % % almost no restrictions whatsoever. You may copy it, give it away % % or re-use it under the terms of the Project Gutenberg License % % included with this eBook or online at www.gutenberg.net % % % % % % Producer's Comments % % % % Since the illustrations have been provided in pdf format, it is % % easiest to compile using pdflatex. However, running latex then % % dvips will also work if the graphics are converted to eps, % % provided that the package eepic is substituted for eepicemu, and % % pdftex driver option to graphicx and hyperref is changed to % % dvips. % % % % % % Things to Check: % % % % Spellcheck: OK % % LaCheck: OK, false positives: % % Book used \ldots never \cdots; % % ! for factorial, % % decimal points % % deliberate space after { % % Lprep/gutcheck: OK % % PDF pages, excl. Gutenberg boilerplate: 214 % % PDF pages, incl. Gutenberg boilerplate: 225 % % ToC page numbers: OK % % Index: OK % % Images: 8 PDF (in /images), 14 LaTeX (embedded). % % Fonts: % % Longtable (at back): aligned % % % % % % Compile history: % % % % 20th June 2006: LW compiled with pdflatex (tetex under MacOSX) % % % % pdflatex infinitesimal % % makeindex infinitesimal % % pdflatex infinitesimal % % pdflatex infinitesimal % % % % 2nd July 2006: JT compiled with pdflatex (MiKTeX / WinXP) % % % % pdflatex 18741-t % % makeindex 18741-t % % pdflatex 18741-t % % pdflatex 18741-t % % % % Front- and back-matter give pdfTeX warnings - this is a known % % issue with documents that restart numbering, and is safe to % % ignore. Also warns of math in section titles. % % 3 Overfulls, 2 Underfulls. % % % %%%%%%%%%%%%%%%%%%%%%%%%%%%%%%%%%%%%%%%%%%%%%%%%%%%%%%%%%%%%%%%%%%%%%% %%%%%%%%%%%%%%%%%%%%%%%%%%%%% PACKAGES %%%%%%%%%%%%%%%%%%%%%%%%%%%%%%% \documentclass[a4paper,12pt]{book}[2004/02/16] \usepackage{amsmath, amsthm}% Required \usepackage{amssymb}% Used for 3 symbols, lines below ensure file % will compile without it. \providecommand{\leqq}{\leq} \providecommand{\geqq}{\geq} \providecommand{\therefore}{\mathrel{{.}\kern-.05em\raise.40em\hbox{.}\kern-.05em{.}}} \usepackage{a4wide}% Optional; chooses nicer margins for a4 paper \usepackage{stmaryrd}% Used for \olessthan. % Could substitute {txfonts/pxfonts} package and use \circledless. % Line below ensures file will compile without it. \providecommand{\olessthan}{(<)} \usepackage{epic,eepicemu}% Required for figs 4, 5, 7, 12, 14, 16, 17, 18, 20 % If compiling via latex+dvips, put eepic in place of eepicemu. \usepackage{longtable}% Required for Wiley catalogue at back. % If unavailable, remove this, or split into a number of shorter tables % that fit on one page. \usepackage{color}% If unavailable, the file will compile if the 2nd % newcommand below is uncommented and the one above it commmented out. % Textual corrections will be underlined instead of highlighted in grey. \providecommand{\definecolor}[3]{} \providecommand{\colorbox}[2]{#2} \setlength{\fboxsep}{1pt} \definecolor{corr}{rgb}{0.89,0.89,0.89} \newcommand{\correction}[2]{\colorbox{corr}{#1}} %\newcommand{\correction}[1]{\underline{#1}} \usepackage[pdftex]{graphicx}% If unavailable, use "graphics" in place of % "graphicx". If both are unavailable or if pictures are absent, % add the option "draft" into the documentclass. Figures will not appear, % but nor will hyperlinks. Or, don't use draft mode but manually remove all % the figures; hyperlinks will still work. % If compiling via dvips, change [pdftex] to [dvips] and convert images to % Encapsulated Post Script. \usepackage{makeidx}% If unavailable, the following line makes the file % compile. Do not run makeindex. Document will have no index. \providecommand{\printindex}{} \usepackage[pdftex,plainpages=false,pdfpagelabels,colorlinks,linkcolor=blue]{hyperref} % If unavailable, the following lines ensure the file compiles, but % the document will not have hyperlinks. \providecommand{\hyperlink}[2]{#2} \providecommand{\hypertarget}[2]{#2} \providecommand{\phantomsection}{} \providecommand{\pdfbookmark}[3][0]{} \providecommand{\hypersetup}[1]{} %%%%%%%%%%%%%%%%%%%%%%%%%%%%% PREAMBLE %%%%%%%%%%%%%%%%%%%%%%%%%%%%%%% %%%%%%%% Fix hyperref page links \makeatletter \AtBeginDocument{\def\pageref#1{% \expandafter\@pagesetref\csname r@#1\endcsname\@empty{#1}}} \makeatother %%%%%%%% Hyperref setup \hypersetup{ pdfauthor = {Oswald Veblen and N. J. Lennes}, pdftitle = {Introduction to Infinitesimal Analysis} } %%%%%%%% Special notation for intervals \newlength{\intwidth} \newcommand{\interval}[2]{\settowidth{\intwidth}{$#1\ #2$}\overset{|\!\rule[0.25ex]{\intwidth}{0.5pt}\!|}{#1\ #2}} \newcommand{\linterval}[2]{\settowidth{\intwidth}{$#1\ #2$}\overset{|\!\rule[0.5ex]{\intwidth}{0.5pt}}{#1\ #2}} \newcommand{\rinterval}[2]{\settowidth{\intwidth}{$#1\ #2$}\overset{\rule[0.5ex]{\intwidth}{0.5pt}\!|}{#1\ #2}} % For more modern notation (as at 2006) remove the above four lines % and uncomment the following three: %\newcommand{\interval}[2]{[#1,#2]} %\newcommand{\linterval}[2]{[#1,#2)} %\newcommand{\rinterval}[2]{(#1,#2]} %%%%%%%% Special inequality signs \newlength{\chevron} \settowidth{\chevron}{$<$} \newlength{\equals} \settowidth{\equals}{$=$} \addtolength{\chevron}{\equals} % to get < and = centred wrt each other \newcommand{\weirdineq}[1]{% \mathbin{\lower0.3ex\hbox{$#1$\kern-.5\chevron\raise1.25ex\hbox{$=$}}} } \newcommand{\qqle}{\weirdineq{<}} \newcommand{\qqge}{\weirdineq{>}} %%%%%%%% Sections formatting \renewcommand{\thesection}{\S~\arabic{section}} \renewcommand{\sectionmark}[1]{} \renewcommand{\chaptermark}[1]{\markboth{INFINITESIMAL ANALYSIS.}{#1}} %%%%%%%% Theorems formatting (book also used parindent but it's ugly!) \newtheoremstyle{itheorem}{}{}{\itshape}{}{\bfseries}{.}{ }{#1\if!#3!\else\ \fi\thmnote{#3}} \newtheoremstyle{icorollary}{}{}{}{}{\itshape}{.---}{0pt}{#1} \newtheoremstyle{numcorollary}{}{}{}{}{\itshape}{.}{ }{#1\if!#3!\else\ \fi\thmnote{#3}} \newtheoremstyle{idefinition}{}{}{}{}{\bfseries}{.---}{0pt}{} \newtheoremstyle{ilemma}{}{}{\itshape}{}{\bfseries}{.---}{0pt}{#1\if!#3!\else\ \fi\thmnote{#3}} \newtheoremstyle{iother}{}{}{\itshape}{}{\bfseries}{.---}{0pt}{\thmnote{#3}} \theoremstyle{ilemma} \newtheorem*{lemma}{Lemma} \theoremstyle{itheorem} \newtheorem{theorem}{Theorem} \theoremstyle{iother} \newtheorem{other}{} \theoremstyle{icorollary} \newtheorem{corollary}{Corollary} \theoremstyle{numcorollary} \newtheorem{ncorollary}{Corollary} \theoremstyle{idefinition} \newtheorem*{definition}{Definition} \newtheorem*{definitions}{Definitions} \newtheorem*{defnorder}{Definition of Order} %%%%%%%% Proof environment \renewcommand{\proofname}{\upshape\bfseries Proof} \renewcommand{\qedsymbol}{}% For the default square box at the end % of proofs, remove this line. For any other text or symbol at % end of proofs, insert it in the second curly bracket. %%%%%%%% Miscellaneous \renewcommand{\dfrac}[2]{\frac{#1}{#2}}% Book always used displaystyle % for fractions, but it makes text ugly! \renewcommand{\indexname}{\protect\label{index}\protect\pdfbookmark[0]{INDEX.}{index}\protect\plainindexname{}INDEX.} \newcommand{\plainindexname}{\gdef\indexname{INDEX.}} \makeindex %%%%%%%%%%%%%%%%%%%%%%%% START OF DOCUMENT %%%%%%%%%%%%%%%%%%%%%%%%%% \begin{document} \thispagestyle{empty} \small \begin{verbatim} Project Gutenberg's Introduction to Infinitesimal Analysis by Oswald Veblen and N. J. Lennes This eBook is for the use of anyone anywhere at no cost and with almost no restrictions whatsoever. You may copy it, give it away or re-use it under the terms of the Project Gutenberg License included with this eBook or online at www.gutenberg.net Title: Introduction to Infinitesimal Analysis Functions of one real variable Author: Oswald Veblen and N. J. Lennes Release Date: July 2, 2006 [EBook #18741] Language: English Character set encoding: TeX *** START OF THIS PROJECT GUTENBERG EBOOK INFINITESIMAL ANALYSIS *** Produced by K.F. Greiner, Joshua Hutchinson, Laura Wisewell, Owen Whitby and the Online Distributed Proofreading Team at http://www.pgdp.net (This file was produced from images generously made available by Cornell University Digital Collections.) \end{verbatim} \normalsize \frontmatter \begin{titlepage} {\setlength{\fboxsep}{10pt} \framebox{% \begin{minipage}{0.9\textwidth} \setlength{\fboxsep}{1pt}\sf \label{transnotes}\pdfbookmark[0]{Transcriber's Notes.}{transnotes} \textbf{Transcriber's Notes.}\medskip\par A large number of printer errors have been corrected. These are shaded \correction{like this}{}, and details can be found in the source code in the syntax \texttt{\textbackslash correction\{corrected\}\{original\}}. In addition, the formatting of a few lemmas, corollaries etc.\ has been made consistent with the others. \medskip\par The unusual inequality sign $\qqge$ used a few times in the book in addition to $\geqq$ has been preserved, although it may reflect the printing rather than the author's intention. The notation $\interval{a}{b}$ for intervals is not in common use today, and the reader able to run \LaTeX{} will find it easy to redefine this macro to give a modern equivalent. Similarly, the original did not mark the ends of proofs in any way and so nor does this version, but the reader who wishes can easily redefine \texttt{\textbackslash qedsymbol} in the source. \end{minipage} }} \end{titlepage} %-----File: 001.png--- %[Blank page] %-----File: 002.png--- %[Blank Page] %-----File: 003.png--- % [Library stamp] % % Cornell University Library % % % BOUGHT WITH THE INCOME % FROM THE % SAGE ENDOWMENT FUND % THE GIFT OF % Henry W. Sage % 1891 % %-----File: 004.png--- %[Blank Page] %-----File: 005.png---Folio i------- \title{\label{titlepage}\pdfbookmark[0]{Title Page.}{titlepage}% INTRODUCTION\\ {\small TO}\\ {\Huge INFINITESIMAL ANALYSIS\\[1ex]} {\Large FUNCTIONS OF ONE REAL VARIABLE} } \author{{\small BY}\\ OSWALD VEBLEN\\ \textit{Preceptor in Mathematics, Princeton University}\\ {\small \textsc{And}}\\ N.~J. LENNES\\ \textit{Instructor in Mathematics in the Wendell Phillips High School, Chicago} } \date{% \textit{FIRST EDITION}\\ {\small FIRST THOUSAND}\\ \vspace{0.2\textheight} NEW YORK\\ JOHN WILEY \& SONS\\ \textsc{London: CHAPMAN \& HALL, Limited}\\ 1907 } \maketitle %-----File: 006.png---Folio ii------- \begin{center} \vspace*{0.4\textheight} Copyright, 1907\\ \textsc{by\\ OSWALD VEBLEN and N.~J. LENNES\\} \vfill ROBERT DRUMMOND, PRINTER, NEW YORK \end{center} \newpage %-----File: 007.png---Folio iii------ \chapter*{PREFACE} A course dealing with the fundamental theorems of infinitesimal calculus in a rigorous manner is now recognized as an essential part of the training of a mathematician. It appears in the curriculum of nearly every university, and is taken by students as ``Advanced Calculus'' in their last collegiate year, or as part of ``Theory of Functions'' in the first year of graduate work. This little volume is designed as a convenient reference book for such courses; the examples which may be considered necessary being supplied from other sources. The book may also be used as a basis for a rather short theoretical course on real functions, such as is now given from time to time in some of our universities. The general aim has been to obtain rigor of logic with a minimum of elaborate machinery. It is hoped that the systematic use of the Heine-Borel theorem has helped materially toward this end, since by means of this theorem it is possible to avoid almost entirely the sequential division or ``pinching'' process so common in discussions of this kind. The definition of a limit by means of the notion ``value approached'' has simplified the proofs of theorems, such as those giving necessary and sufficient conditions for the existence of limits, and in general has largely decreased the number of $\varepsilon$'s and $\delta$'s. The theory of limits is developed for multiple-valued functions, which gives certain advantages in the treatment of the definite integral. In each chapter the more abstract subjects and those which can be omitted on a first reading are placed in the concluding %-----File: 008.png---Folio iv------- sections. The last chapter of the book is more advanced in character than the other chapters and is intended as an introduction to the study of a special subject. The index at the end of the book contains references to the pages where technical terms are first defined. When this work was undertaken there was no convenient source in English containing a rigorous and systematic treatment of the body of theorems usually included in even an elementary course on real functions, and it was necessary to refer to the French and German treatises. Since then one treatise, at least, has appeared in English on the Theory of Functions of Real Variables. Nevertheless it is hoped that the present volume, on account of its conciseness, will supply a real want. The authors are much indebted to Professor E.~H. Moore of the University of Chicago for many helpful criticisms and suggestions; to Mr.~E.~B. Morrow of Princeton University for reading the manuscript and helping prepare the cuts; and to Professor G.~A. Bliss of Princeton, who has suggested several desirable changes while reading the proof-sheets. %-----File: 009.png---Folio v-------- \tableofcontents %-----File: 010.png---Folio vi------- %-----File: 011.png---Folio vii------ %-----File: 012.png--- %[Blank Page] %-----File: 013.png---Folio 1-------- \mainmatter \chapter{THE SYSTEM OF REAL NUMBERS.}\hypertarget{chapI}{}%[I] \section{Rational and Irrational Numbers.}\hypertarget{chIsec1}{}%[1] \index{Number} The real number system may be classified as follows: \begin{enumerate} \item[(1)]\hypertarget{item1p1}{} All integral numbers, both positive and negative, including zero. \item[(2)]\hypertarget{item2p1}{} All numbers $\frac mn$, where $m$ and $n$ are integers ($n\neq 0$). \item[(3)]\hypertarget{item3p1}{} Numbers not included in either of the above classes, such as $\sqrt{2}$ and $\pi$.\footnote{% It is clear that there is no number $\frac mn$ such that $\frac{m^2}{n^2}=2$, for if $\frac{m^2}{n^2}=2$, then $m^2=2n^2$, where $m^2$ and $2n^2$ are integral numbers, and $2n^2$ is the square of the integral number $m$. Since in the square of an integral number every prime factor occurs an even number of times, the factor 2 must occur an even number of times both in $n^2$ and $2n^2$, which is impossible because of the theorem that an integral number has only one set of prime factors.} \end{enumerate} Numbers of classes \hyperlink{item1p1}{(1)} and \hyperlink{item2p1}{(2)} are called rational or commensurable numbers, while the numbers of class~\hyperlink{item3p1}{(3)} are called \index{Rational!numbers}\index{Number!irrational}\index{Irrational!number}irrational or incommensurable numbers. As an illustration of an irrational number consider the square root of $2$. One ordinarily says that $\sqrt{2}$ is $1.4+$, or %-----File: 014.png---Folio 2-------- $1.41+$, or $1.414+$, etc. The exact meaning of these statements is expressed by the following inequalities:\footnote{% $ab$ signifies that $a$ is greater than $b$.} \begin{align*} &(1.4)^2 < 2 < (1.5)^2, \\ &(1.41)^2 < 2 < (1.42)^2, \\ &(1.414)^2 < 2 < (1.415)^2,\\ &\qquad\mbox{etc.} \end{align*} Moreover, by the foot-note above no terminating decimal is equal to the square root of 2. Hence Horner's Method, or the usual algorithm for extracting the square root, leads to an infinite sequence of rational numbers which may be denoted by $a_1, a_2, a_3, \ldots, a_n,\ldots$ (where $a_1 = 1.4$, $a_2 = 1.41$, etc.), and which has the property that for every positive integral value of $n$ \begin{align*} a_n &\le a_{n+1}, &a_n^2 &< 2 < \left(a_n + \frac{1}{10^n}\right)^2. \end{align*} Suppose, now, that there is a \emph{least} number $a$ greater than every $a_n$. We easily see that if the ordinary laws of arithmetic as to equality and inequality and addition, subtraction, and multiplication hold for $a$ and $a^2$, then $a^2$ is the rational number 2. For if $a^2<2$, let $2-a^2 = \varepsilon$, whence $2=a^2 + \varepsilon$. If $n$ were so taken that $\frac{1}{10^n} < \frac\varepsilon5$, we should have from the last inequality\footnote{% This involves the assumption that for every number, $\varepsilon$, however small there is a positive \correction{integer}{integrer} $n$ such that $\frac{1}{10^n} < \frac\varepsilon5$. This is of course obvious when $\varepsilon$ is a rational number. If $\varepsilon$ is an irrational number, however, the statement will have a definite meaning only after the irrational number has been fully defined.} \[ 2< \left(a_n + \frac{1}{10^n}\right)^2 = a_n^2 + 2a_n\cdot\frac{1}{10^n}+ \left(\frac{1}{10^n}\right)^2 < a_n^2 + 4\frac\varepsilon5 + \frac\varepsilon5 < a^2 + \varepsilon, \] so that we should have both $2=a^2 + \varepsilon$ and $2 2$, let $a^2-2 = \varepsilon'$ or $2 + \varepsilon' = a^2$. Taking $n$ such that $\frac{1}{10^n} < \frac\varepsilon5$, we should have \[ \left(a_n +\frac{1}{10^n} \right)^2 < (a_n^2) +\varepsilon' < 2+\varepsilon' < a; \] and since $a_n + \frac{1}{10^n}$ is greater than $a_k$ for all values of $k$, this would contradict the hypothesis that $a$ is the \emph{least} number greater than every number of the sequence $a_1,a_2,a_3,\ldots$ We also see without difficulty that $a$ is the only number such that $a^2 =2$. \section{Axiom of Continuity.}\hypertarget{chIsec2}{}%[2] The essential step in passing from ordinary rational numbers to the number corresponding to the symbol $\sqrt{2}$ is thus made to depend upon an assumption of the existence of a number $a$ bearing the unique relation just described to the sequence $a_1,a_2$,\correction{$a_3$}{$a_n$},$\ldots$ In order to state this hypothesis in general form we introduce the following definitions: \begin{definition}\index{Numbers!sets of}\index{Sets of numbers} The notation $[x]$ denotes a \textit{set},\footnote{% Synonyms of set are class\index{Class}, aggregate, collection, assemblage, etc.} any element of which is denoted by $x$ alone, with or without an index or subscript. A \index{Upper bound!of a set of numbers}\index{Lower bound!of a set of numbers}\index{Bounds!upper and lower}set of numbers $[x]$ is said to have an \emph{upper bound}, $M$, if there exists a number $M$ such that there is no number of the set greater than $M$. This may be denoted by $M \geqq [x]$. A set of numbers $[x]$ is said to have a \emph{lower bound}, $m$, if there exists a number $m$ such that no number of the set is less than $m$. This we denote by $m \leqq [x]$. \end{definition} Following are examples of sets of numbers: \begin{enumerate} \item[(1)] $1,2,3$. \item[(2)] $2,4,6,\ldots,2k,\ldots$ \item[(3)] $1/2,1/{2^2},1/{2^3},\ldots,1/{2^n},\ldots$ \item[(4)] All rational numbers less than $1$. \item[(5)]\hypertarget{item5p3}{} All rational numbers whose squares are less than $2$. \end{enumerate} %-----File: 016.png---Folio 4-------- Of the first set $1$, or any smaller number, is a lower bound and $3$, or any larger number, is an upper bound. The second set has no upper bound, but $2$, or any smaller number, is a lower bound. The number $3$ is the least upper bound of the first set, that is, the smallest number which is an upper bound. The \index{Least upper bound}least upper and the \index{Greatest lower bound}greatest lower bounds of a set of numbers $[x]$ are called by some writers the upper and lower limits respectively. We shall denote them by $\overline{B}[x]$ and $\underline{B}[x]$ respectively. By what precedes, the set~\hyperlink{item5p3}{(5)} would have no least upper bound unless $\sqrt{2}$ were counted as a number. We now state our hypothesis of continuity in the following form: \begin{other}[Axiom K]\hypertarget{axiomK}{}\index{Axioms!of continuity}\index{Continuity!axioms of} If a set $[r]$ of rational numbers having an upper bound has no rational least upper bound, then there exists one and only one number $\overline{B}[r]$ such that \begin{enumerate} \item[(a)] $\overline{B}[r] > r'$, where $r'$ is any number of $[r]$ or any rational number less than some number of $[r]$. \item[(b)] $\overline{B}[r] < r''$, where $r''$ is any rational upper bound of $[r]$.% \footnote{% This axiom implies that the new (irrational) numbers have relations of order with all the rational numbers, but does not explicitly state relations of order among the irrational numbers themselves. Cf.\ Theorem~\hyperlink{thm2}{2}.} \end{enumerate} \end{other} \begin{definition}\index{Continuous!real number system}\index{Real number system}\index{Number!system}\index{Continuum, linear}\index{Linear continuum} The number $\overline{B}[r]$ of \hyperlink{axiomK}{axiom~K} is called the least upper bound of $[r]$, and as it cannot be a rational number it is called an \index{Number!irrational}\index{Irrational!number}\textit{irrational} number. The set of all rational and irrational numbers so defined is called the \textit{continuous real number system}. It is also called \textit{the linear continuum.} The set of all real numbers between any two real numbers is likewise called a linear continuum. \end{definition} \begin{theorem}[1]\hypertarget{thm1}{} If two sets of rational numbers $[r]$ and $[s]$, having upper bounds, are such that no $r$ is greater than every $s$ and no $s$ greater than every $r$, then $\overline{B}[r]$ and $\overline{B}[s]$ are the same; that is, in symbols, \[ \overline{B}[r] = \overline{B}[s]. \] \end{theorem} \begin{proof} If $\overline{B}[r]$ is rational, it is evident, and if $\overline{B}[r]$ is irrational, it is a consequence of \hyperlink{axiomK}{Axiom~K} that \[ \overline{B}[r] > s', \] %-----File: 017.png---Folio 5-------- where $s'$ is any rational number not an upper bound of $[s]$. Moreover, if $s''$ is rational and greater than every $s$, it is greater than every $r$. Hence \[ \overline{B}[r] < s'', \] where $s''$ is any rational upper bound of $[s]$. Then, by the definition of $\overline{B}[s]$, \[ \overline{B}[r] = \overline{B}[s],\qedhere \] \end{proof} \begin{definition} If a number $x$ (in particular an irrational number) is the least upper bound of a set of rational numbers $[r]$, then the set $[r]$ is said to \textit{determine} the number $x$. \end{definition} \begin{ncorollary}[1]\label{cor1p5}\hypertarget{cor1p5}{} The irrational numbers $i$ and $i'$ determined by the two sets $[r]$ and $[r']$ are equal if and only if there is no number in either set greater than every number in the other set. \end{ncorollary} \begin{ncorollary}[2]\label{cor2p5}\hypertarget{cor2p5}{} Every irrational number is determined by some set of rational numbers. \end{ncorollary} \begin{definition} If $i$ and $i'$ are two irrational numbers determined respectively by sets of rational numbers $[r]$ and $[r']$ and if some number of $[r]$ is greater than every number of $[r']$, then \[ i>i' \text{ and } i'0$, then $a+b>b$, and that if $a<0$, then $a+b0,\ b>0, &\text{\ \ it\ follows\ that\ }&ab=\overline{B}[xy];\\ a<0,\ b<0, &``\hspace{2em}``\hspace{2em}``&ab=\overline{B}[xy];\\ a<0,\ b>0, &``\hspace{2em}``\hspace{2em}``&ab=\underline{B}[xy];\\ a>0,\ b<0, &``\hspace{2em}``\hspace{2em}``&ab=\underline{B}[xy]. \end{array} \] \begin{definition}\index{Irrational!numbers!product of}\index{Product of irrational numbers} If $a$ and $b$ are not both rational and $[x]$ is the set of all rational numbers between $0$ and $a$, and $[y]$ the set of all rationals between $0$ and $b$, then if $a>0$, $b>0$, $ab$ means $\overline{B}[xy]$; if $a<0$, $b<0$, $ab$ means $\overline{B}[xy]$; if $a<0$, $b>0$, $ab$ means $\underline{B}[xy]$; if $a>0$, $b<0$, $ab$ means $\underline{B}[xy]$. If $a$ or $b$ is zero, then $ab=0$. \end{definition} %-----File: 021.png---Folio 9-------- It is proved, just as in the case of addition, that $ab=ba$, that $a(bc) = (ab)c$, that if $a$ is rational $[ay]$ is the same set as $[xy]$, that if $a>0$, $b>0$, $ab>0$. Likewise the \index{Quotient of irrational numbers}\index{Irrational!numbers!quotient of}quotient $\frac{a}{b}$ is defined as a number $c$ such that $ac=b$, and it is proved that in case $a>0$, $b>0$, then $c=\overline{B}\bigl[\frac{x}{y'}\bigr]$, where $[y']$ is the set of all rationals greater than $b$. Similarly for the other cases. Moreover, the same sort of reasoning as before justifies the usual method of multiplying non-terminated decimals. To complete the rules of operation we have to prove what is known as the distributive law, namely, that \[ a(b+c)=ab+ac. \] To prove this we consider several cases according as $a$, $b$, and $c$ are positive or negative. We shall give in detail only the case where all the numbers are positive, leaving the other cases to be proved by the reader. In the first place we easily see that for positive numbers $e$ and $f$, if $[t]$ is the set of all the rationals between 0 and $e$, and $[T]$ the set of all rationals less than $e$, while $[u]$ and $[U]$ are the corresponding sets for $f$, then \[ e+f = \overline{B}[T+U]=\overline{B}[t+u]. \] Hence if $[x]$ is the set of all rationals between 0 and $a$, $[y]$ between 0 and $b$, $[z]$ between 0 and $c$, \[ b + c = \overline{B}[y+z] \quad \text{and hence} \quad a(b+c) = \overline{B}[x(y + z)]. \] On the other hand $ab=\overline{B}[xy]$, $ac=\overline{B}[xz]$, and therefore $ab+ac= \overline{B}[(xy+xz)]$. But since the distributive law is true for rationals, $x(y+z)=xy+xz$. Hence $\overline{B}[x(y+z)]=\overline{B}[(xy+xz)]$ and hence \[ a(b+c)=ab+ac. \] We have now proved that the system of rational and irrational numbers is not only continuous, but also is such that we may perform with these numbers all the operations of arithmetic. We have indicated the method, and the reader may %-----File: 022.png---Folio 10------- detail that every rational number may be represented by a terminated decimal, \[ a_k 10^k+a_{k-1}10^{k-1}+\ldots+a_0+\frac{a_{-1}}{10} +\ldots+\frac{a_{-n}}{10^n} = a_k a_{k-1}\ldots a_0 a_{-1}a_{-2}\ldots a_{-n}, \] or by a circulating decimal, \[ a_k a_{k-1}\ldots a_0 a_{-1}a_{-2}\ldots a_{-i}\ldots a_{-j}a_{-i}\ldots a_{-j}\ldots, \] where $i$ and $j$ are any positive integers such that $i0$ and $b>0$, then $ab>0$. \end{itemize} These postulates may be regarded as summarizing the properties of the real number system. Every theorem of real analysis is a logical consequence of them. For convenience of reference later on we summarize also the rules of operation with the symbol\index{Absolute value} $|x|$, which indicates the ``numerical'' or ``absolute'' value of $x$. That is, if $x$ is positive, $|x| = x$, and if $x$ is negative, $|x| =-x$. \begin{align*} |x| + |y| &\geqq|x+y|. \tag{1}\\ \therefore\quad\sum_{k=1}^n|x_k| &\geqq \Bigl|\sum_{k=1}^n x_k \Bigr|, \tag{2} \end{align*} where $\sum_{k=1}^n x_k = x_1 + x_2 + \ldots + x_n$. \begin{align*} \bigl| |x|-|y| \bigr|\leqq|x-y| &= |y-x|\leqq|x| +|y|. \tag{3}\\ |x\cdot y| &= |x| \cdot|y|. \tag{4}\\ \frac{|x|}{|y|} &= \left|\frac xy\right|. \tag{5} \end{align*} \[\text{If } |x-y| < e_1,\ |y-z| < e_2, \text{ then } |x-z| < e_1 + e_2. \tag{6} \] %-----File: 027.png---Folio 15------- If $[x]$ is any bounded set, \[ \overline{B}[x]-\underline{B}[x] = \overline{B}[|x_1-x_2 |]. \tag{7} \] \section[The Number $e$.]{The Number $\boldsymbol e$.}\hypertarget{chIsec6}{}%[6] In the theory of the exponential and logarithmic functions (see page~\pageref{s4p97}) the irrational number $e$ plays an important r\^ole. This number may be defined as follows: \hypertarget{eq1p16}{\[ e = \overline{B}[E_n], \tag{1} \]} where \[ E_n = 1+\frac{1}{1!}+\frac{1}{2!}+\ldots +\frac{1}{n!}, \] where $[n]$ is the set of all positive integers, and \[ n!= 1 \cdot 2 \cdot 3 \ldots n. \] It is obvious that \hyperlink{eq1p16}{(1)} defines a finite number and not infinity, since \[ E_n=1+\frac{1}{1!}+\frac{1}{2!}+ \ldots + \frac{1}{n!} < 1+1+\frac12+\frac{1}{2^2}+ \ldots +\frac{1}{2^{n-1}} = 3-\frac{1}{2^{n-1}}. \] The number $e$ may very easily be computed to any number of decimal places, as follows: %-----File: 028.png---Folio 16------- \begin{align*} E_0 &= 1 \\ \frac{1}{1!} &= 1 \\ \frac{1}{2!} &= \;.5 \\ \frac{1}{3!} &= \;.166666+\\ \frac{1}{4!} &=\phantom{1}.041666+ \\ \frac{1}{5!} &=\phantom{1}.008333+ \\ \frac{1}{6!} &=\phantom{1}.001388+ \\ \frac{1}{7!} &=\phantom{1}.000198+ \\ \frac{1}{8!} &=\phantom{1}.000024+ \\ \frac{1}{9!} &=\phantom{1}.000002+ \\ &\quad\ \rule{5em}{0.5pt}\\ E_9 &= 2.7182\ldots \end{align*} \begin{lemma} If $k>e$, then $E_k > e-\frac{1}{k!}$. \end{lemma} \begin{proof} From the definitions of $e$ and $E_n$ it follows that \[ e-E_k = \overline{B} \left[ \frac{1}{(k+1)!} + \frac{1}{(k+2)!} + \ldots \frac{1}{(k+l)!} \right], \] where $[l]$ is the set of all positive integers. Hence \[ e-E_k = \frac{1}{(k+1)!} \cdot \overline{B} \biggl[1+ \frac{1}{k+2} + \frac{1}{(k+2)(k+3)} + \ldots + \frac{1}{(k+2) \ldots (k+l)} \biggr], \] or \[ e-E_k < \frac{1}{(k+1)!} \cdot e. \] If $k>e$, this gives \[ E_k > e-\frac{1}{k!}.\qedhere \] \end{proof} %-----File: 029.png---Folio 17------- \begin{theorem}[7]\hypertarget{thm7}{}\label{t7p17} \[ e=\overline{B}\left[\left(1+\frac1n\right)^n\right], \] where $[n]$ is the set of all positive integers. \end{theorem} \begin{proof} By the binomial theorem for positive integers \[ \left(1+\frac1n \right)^n = 1 + n\left(\frac1n \right) + \frac{n(n-1)}{2!} \cdot \left(\frac1n \right)^2 + \ldots + \left(\frac1n \right)^n. \] Hence \begin{align*} E_n-\left(1+\frac1n \right)^n &= \sum^n_{k=2} \left(\frac{1}{k!}-\frac{n(n-1) \ldots (n-k+1)}{k!\, n^k} \right) \\\hypertarget{eqap17}{% &= \sum^n_{k=2} \frac{n^k-n(n-1) \ldots (n-k+1)}{k!\,n^k}, \tag{\textit{a}}} \\ &< \sum^n_{k=2} \frac{n^k-(n-k+1)^k}{k!\,n^k}. \end{align*} Hence by factoring \begin{align*} E_n-\left(1+\frac1n \right)^n &< \sum^n_{k=2} \frac{(k-1)(n^{k-1} + n^{k-2}(n-k+1) + \ldots + (n-k+1)^{k-1}) }{k!\,n^k} \\ &< \sum^n_{k=2} \frac{(k-1)k n^{k-1}}{k!\,n^k} \\ &< \frac1n \sum^n_{k=2} \frac{(k-1)k}{k!} \end{align*} i.e., \hypertarget{eqbp17}{\[ E_n-\left(1+\frac1n \right)^n < \frac1n\left(1+\sum^{n-2}_{l=1} \frac{1}{l!} \right) < \frac en. \tag{\textit{b}} \]} From \hyperlink{eqap17}{(\textit{a})} \begin{align*} \hypertarget{eq1p17}{\tag{1} E_n &> \left(1+\frac1n \right)^n\\} \intertext{and from \hyperlink{eqbp17}{(\textit{b})}} \tag{2} \left(1+\frac1n \right)^n > E_n-\frac en, \end{align*} %-----File: 030.png---Folio 18------- whence by the lemma \hypertarget{eq3p18}{\[ \left(1+\frac1n \right)^n > e-\frac{1}{n!}-\frac en. \tag{3} \]} From \hyperlink{eq1p17}{(1)} it follows that $e$ is an upper bound of \[ \left[\left(1+\frac1n \right)^n\right], \] and from \hyperlink{eq3p18}{(3)} it follows that no smaller number can be an upper bound. Hence \[ \overline{B}\left[\left(1+\frac1n \right)^n\right] = e. \]\label{endpf18} \end{proof} \section{Algebraic and Transcendental Numbers.}\hypertarget{chIsec7}{}%[7] \index{Algebraic!numbers}\index{Transcendental!numbers}\index{Number!algebraic}\index{Numbers!transcendental} The distinction between rational and irrational numbers, which is a feature of the discussion above, is related to that between \textit{algebraic} and \textit{transcendental} numbers. A number is algebraic if it may be the root of an algebraic equation, \[ a_0x^n + a_1x^{n-1} + \ldots + a_{n-1}x + a_n = 0, \] where $n$ and $a_0,a_1,\ldots,a_n$ are integers and $n>0$. A number is transcendental if not algebraic. Thus every rational number $\frac mn$ is algebraic because it is the root of the equation \[ nx-m=0, \] while every transcendental number is irrational. Examples of transcendental numbers are, $e$, the base of the system of natural logarithms, and $\pi$, the ratio of the circumference of a circle to its diameter. The proof that these numbers are transcendental follows on page~\pageref{s8p19}, though it makes use of infinite series which will %-----File: 031.png---Folio 19------- not be defined before page~\pageref{dp71}, and the function $e^x$, which is defined on page~\pageref{dp57}. The existence of transcendental numbers was first proved by \textsc{J.~Liouville}, Comptes Rendus, 1844. There are in fact an infinitude of transcendental numbers between any two numbers. Cf.\ \textsc{H.~Weber}, \textit{Algebra}, Vol.~2, p.~822. No \textit{particular} number was proved transcendental till, in 1873, \textsc{C.~Hermite} (Crelle's Journal, Vol.~76, p.~303) proved $e$ to be transcendental. In 1882 \textsc{E.~Lindemann} (Mathematische Annalen, Vol.~20, p.~213) showed that $\pi$ is also transcendental. The latter result has perhaps its most interesting application in geometry, since it shows the impossibility of solving the classical problem of constructing a square equal in area to a given circle by means of the ruler and compass. This is because any construction by ruler and compass corresponds, according to analytic geometry, to the solution of a special type of algebraic equation. On this subject, see \textsc{F.~Klein}, \textit{Famous Problems of Elementary Geometry} (Ginn \& Co., Boston), and \textsc{Weber} and \textsc{Wellstein}, \textit{Encyclop\"adie der Elementarmathematik}, Vol.~1, pp.~418--432 (B.~G.~Teubner, Leipzig). \section[The Transcendence of $e$.]{The Transcendence of $\boldsymbol e$.}\hypertarget{chIsec8}{}%[8] \label{s8p19} \begin{theorem}[8]\hypertarget{thm8}{} If $c,c_1,c_2,c_3,\ldots,c_n$ are integers (or zero but $c \neq 0$), then \hypertarget{eq1p19}{\[ \tag{1} c+c_1e+c_2e^2+\ldots+c_ne^n \neq 0. \]} \end{theorem} \begin{proof} The scheme of proof is to find a number such that when it is multiplied into \hyperlink{eq1p19}{(1)} the product becomes equal to a whole number distinct from zero plus a number between $+1$ and $-1$, a sum which surely cannot be zero. To find this number $N$, we study the series\footnote{% Cf.~pages \pageref{dp71} and \pageref{t58p99}. } for $e^k$, where $k$ is an integer $\qqle n$: \[ e^k = 1 + \frac{k}{1!} + \frac{k^2}{2!} + \frac{k^3}{3!} + \ldots. \] %-----File: 032.png---Folio 20------- Multiplying this series successively by the arbitrary factors $i!\cdot b_i$, we obtain the following equations: \hypertarget{eq2p20}{\[ \left. \begin{array}{l} e^k\cdot 1!\cdot b_1 = b_1 \cdot 1! + b_1k \left(1 + \frac k2 + \frac{k^2}{2\cdot 3} + \ldots \right); \\ e^k\cdot 2! \cdot b_2 = b_2 \cdot 2! \left(1+\frac k1\right) + b_2\cdot k^2\left(1 + \frac k3 + \frac{k^2}{3\cdot 4} + \ldots \right); \\ e^k\cdot 3! \cdot b_3 = b_3 \cdot 3! \left(1+\frac{k}{1!}+\frac{k^2}{2!}\right) + b_3\cdot k^3\left(1 + \frac k4 + \frac{k^2}{4\cdot 5} + \ldots \right); \\ \hdotsfor[10]{1} \\ e^k\cdot s! \cdot b_s = b_s \cdot s!\left(1 + \frac{k}{1!} + \frac{k^2}{2!} + \ldots + \frac{k^{s-1}}{(s-1)!} \right) \\ \hfill + b_s\cdot k^s\left(1 + \frac{k}{s+1} + \frac{k^2}{(s+1)(s+2)} + \ldots \right). \end{array} \right\} \tag{2} \]} For the sake of convenience in notation the numbers $b_1\ldots b_s$ may be regarded as the coefficients of an arbitrary polynomial \[ \phi(x) + b_0 + b_1x + b_2x^2 + \ldots + \mbox{\correction{$b_sx^s$}{$b_sx_s$}}, \] the successive derivatives of which are \begin{gather*} \begin{array}{c} \phi'(x) = b_1 + 2\cdot b_2x + \ldots + s\cdot b_s \cdot x^{s-1}, \\ \hdotsfor[10]{1} \end{array} \\ \begin{array}{c} \phi^{(m)}(x) = b_m\cdot m! + b_{m+1} \cdot \frac{(m+1)!}{1!}\cdot x + \ldots + b_s\cdot \frac{s!}{(s-m)!} \cdot x^{s-m}; \\ \hdotsfor[10]{1} \end{array} \end{gather*} The diagonal in \hyperlink{eq2p20}{(2)} from $\text{\correction{$b_1$}{$b$}}\cdot 1!$ to $b_s\cdot s!\frac{k^{s-1}}{(s-1)!}$ is obviously $\phi'(k)$, the next lower diagonal is $\phi''(k)$, etc. Therefore by adding equations~\hyperlink{eq2p20}{(2)} in this notation we obtain %-----File: 033.png---Folio 21------- \hypertarget{eq3p21}{\begin{align*} e^k(1!\,b_1+2!\,b_2+\ldots+s!\,b_s)=\phi'(k) &+\phi''(k)+\ldots\\ &+\phi^{(s)}(k)+\sum_{m=1}^sb_m\cdot k^m\cdot R_{km}, \tag{3} \end{align*}} in which \[ R_{km}=1+\frac{k}{m+1}+\frac{k^2}{(m+1)(m+2)}+ \ldots. \] Remembering that $\phi(x)$ is perfectly arbitrary, we note that if it were so chosen that \[ \phi'(k)=0,\quad \phi''(k)=0,\ldots,\quad \phi^{(p-1)}(k)=0, \] for every $k$ ($k=1, 2, 3, \ldots, n$) then equations~\hyperlink{eq2p20}{(2)} and \hyperlink{eq3p21}{(3)} could be written in the form \hypertarget{eq4p21}{\begin{align*} e^k(1!\text{\correction{$\cdot$}{}} b_1+2!\cdot b_2+\ldots+s!\cdot b_s) & = \sum_{m=1}^sb_m\cdot k^m\cdot R_{km} \\ & + b_p\cdot p! \\ & + b_{p+1}\cdot (p+1)!\cdot \left(1+\frac{k}{1!}\right) \\ & + \ldots \\ & + b_s\cdot s!\left(1+\frac{k}{1!}+\frac{k^2}{2!}+\ldots +\frac{k^{s-p}}{\text{\correction{$(s-p)!$}{$(s-p)$}}}\right). \tag{4} \end{align*}} A choice of $\phi(x)$ satisfying the required conditions is \hypertarget{eq5p21}{\[ \phi(x)=(a_0+a_1x+a_2x^2+ \ldots +a_nx^n)^p \cdot \frac{x^{p-1}}{(p-1)!}=\frac{(f(x))^p\cdot x^{p-1}}{(p-1)!}, \tag{5} \]} where $f(x) = (x-1)(x-2)(x-3) \ldots (x-n)$. %-----File: 034.png---Folio 22------- Every $k$ ($k = 1, 2, \ldots, n$) is a $p$-tuple root of \hyperlink{eq5p21}{(5)}. Here $p$ is still perfectly arbitrary, but the degree $s$ of $\phi(x)$ is $np+p-1$. If $\phi(x)$ is expanded and the result compared with \[ \phi(x) = b_0 + b_1x + \ldots + b_s x^s, \] it is plain that \[ b_0 = 0,\ b_1=0,\ \ldots,\ b_{p-2}=0, \] on account of the factor $x^{p-1}$, and \[ b_{p-1} = \frac{a_0^p}{(p-1)!},\ b_p = \frac{I_p}{(p-1)!},\ \ldots,\ b_s = \frac{I_s}{(p-1)!}, \] where $I_p,I_{p+1},\ldots,I_s$, are all integers. The coefficient of $e^k$ in the left-hand member of \hyperlink{eq4p21}{(4)} is therefore \[ N_p = a_0^p + \frac{I_p}{(p-1)!}\cdot p! + \frac{I_{p+1}}{(p-1)!}\cdot (p+1)!+ \ldots + \frac{I_s }{(p-1)!}\cdot s! \] Whenever the arbitrary number $p$ is prime and greater than $a_0$, $N_p$ is the sum of $a_0^p$, which cannot contain $p$ as a factor, plus other integers each of which does contain the factor $p$. $N_p$ is therefore \emph{not zero and not divisible by $p$}. Further, since \[ \frac{(p+t)!}{(p-1)!\cdot r!}=p\frac{(p+1)(p+2)\ldots(p+t)}{r!} \] is an integer divisible by $p$ when $r\leqq t$, it follows that all the coefficients of the last block of terms in \hyperlink{eq4p21}{(4)} contain $p$ as a factor. Since $k$ is also an integer, \hyperlink{eq4p21}{(4)} evidently reduces to \[ N_p\cdot e^k=pW_{kp}+\sum_{m=1}^s b_m\cdot k^m\cdot R_{km}, \] %-----File: 035.png---Folio 23------- where $W_{kp}$ is an integer or zero, and this may be abbreviated to the form \hypertarget{eq6p23}{\[ \label{eq1onp23} \tag{6} N_p\cdot e^k = pW_{kp} + r_{kp}. \]} Before completing our proof we need to show that by choosing the arbitrary prime number $p$ sufficiently large, $r_{kp}$ can be made as small as we please. If $\alpha$ is a number greater than $n$, \begin{align*} |R_{km}| &= \left|1+ \frac{k}{m+1} + \frac{k^2}{(m+1)(m+2)} + \ldots \right| \\ &< \left|1+ \frac{\alpha}{m+1} + \frac{\alpha^2}{(m+1)(m+2)} + \ldots \right| \\ &< \left|1+ \frac\alpha1 + \frac{\alpha^2}{2!} + \ldots \right| \\ &< e^\alpha \end{align*} for all integral values of $m$ and of $k \qqle n$. \[ |r_{kp}| = \left|\sum_{m=1}^s b_m \cdot k^m \cdot R_{km} \right| \leqq \sum_{m=1}^s|b_m|\cdot k^m \cdot|R_{k,m}|. \] Since the number $b_m$ is the coefficient of $x^m$ in $\phi(x)$ and since each coefficient of $\phi(x)$ is numerically less than or equal to the corresponding coefficient of \[ \frac{x^{p-1}}{(p-1)!} \left(|a_0|+ |a_1|x + |a_2|x^2 + \ldots + |a_n|x^n \right)^p, \] it follows that \begin{align*} |r_{kp}| &< e^\alpha \cdot \frac{\alpha^{p-1}}{(p-1)!} \left(|a_0|+ |a_1|\alpha + \ldots + |a_n|\alpha^n \right)^p \\ &< \frac{Q^p}{(p-1)!} \cdot e^\alpha, \end{align*} %-----File: 036.png---Folio 24------- where \[ Q = \alpha (|a_0|+ |a_1|\alpha + \ldots + |a_n|\alpha^n) \] is a constant not dependent on $p$. The expression $ \frac{Q^p}{(p-1)!}$ is the $p$th term of the series for $Qe^Q$, and therefore by choosing $p$ sufficiently large $r_{kp}$, may be made as small as we please. If now $p$ is chosen as a prime number, greater than $\alpha$ and $\alpha_0$ and so great that for every $k$, \[ r_{kp} < \frac{1}{n\cdot d}, \] where $d$ is the greatest of the numbers \[ c,\; c_1,\; c_2,\; c_3,\; \ldots,\; c_n, \] the equations~\hyperlink{eq6p23}{(6)} evidently give \begin{align*} N_p(c + c_1 e + c_2 e^2 + \ldots &+ c_n e^n)\\ &= N_p c + p(c_1 W_{1p} + c_2 W_{2p} + \ldots + c_n W_{np}) \\ &\hspace*{2cm}+c_1 r_{1p} + c_2 r_{2p} + \ldots + c_n r_{np},\\ &= N_p c + pW + R,\tag{8} \end{align*} where $W$ is an integer or zero and $R$ is numerically less than unity. Since $N_p c$ is not divisible by $p$ and is not zero, while $pW$ is divisible by $p$, this sum is numerically greater than or equal to zero. Hence \[ N_p (c + c_1 e + c_2 e^2 + \ldots + c_n e^n ) \neq 0. \] Hence \[ c + c_1 e + c_2 e^2 + \ldots + c_n e^n \neq 0, \] and $e$ is a transcendental number. \end{proof} %-----File: 037.png---Folio 25------- \section{The Transcendence of $\pi$.}\hypertarget{chIsec9}{}%[9] The definition of the number $\pi$ is derived from \textsc{Euler}'s formula \[ e^{x \sqrt{-1}} = \cos x + \sqrt{-1} \sin x; \] by replacing $x$ by $\pi$, \hypertarget{eq1p25}{\[ \label{p25eq1} e^{\pi \sqrt{-1}} =-1.\tag{1} \]} If $\pi$ is assumed to be an algebraic number, $\pi\sqrt{-1}$ is also an algebraic number and is the root of an irreducible algebraic equation $F(x)=0$ whose coefficients are integers. If the roots of this equation are denoted by $z_1, z_2, z_3,\ldots, z_n$, then, since $\pi \sqrt{-1}$ is one of the $z$'s, it follows as a consequence of \hyperlink{eq1p25}{(1)} that \hypertarget{eq2p25}{\[ (e^{z_1}+1) (e^{z_2}+1) (e^{z_3}+1) \ldots (e^{z_n}+1) =0.\tag{2} \]} By expanding \hyperlink{eq2p25}{(2)} \[ 1 + \sum e^{z_i} + \sum e^{z_i+z_j} + \sum e^{z_i+z_j+z_k} + \ldots = 0. \] Among the exponents zero may occur a number of times e.g., $(c-1)$ times. If then \[ z_i, \quad z_i + z_j, \quad z_i + z_j + z_k,\quad \ldots, \] be designated by $x_1, x_2, x_3, \ldots, x_n$, the equation becomes \hypertarget{eq3p25}{\[ c + e^{x_1} + e^{x_2} + \ldots + e^{x_n} =0,\tag{3} \]} where $c$ is a positive number at least unity and the numbers $x_i$ are algebraic. These numbers, by an argument for which the reader is referred to \textsc{Weber} and \textsc{Wellstein}'s \emph{Encyclop\"{a}die der Elementarmathematik}, p.~427 et seq., may be shown to be the roots of an algebraic equation \[ f(x) = a_0 + a_1 x + a_2 x^2 + \ldots + a_n x^n = 0,\tag{$3'$} \] %-----File: 038.png---Folio 26------- the coefficients being integers and $a_0\neq0$ and $a_n\neq0$. The rest of the argument consists in showing that equation~\hyperlink{eq3p25}{(3)} is impossible when $x_1,x_2$, \ldots, $x_n$ are roots of ($3'$). The process is analogous to that in \hyperlink{chIsec8}{\S~8}. \hypertarget{eq4p26}{\[ \left. \begin{array}{l}\displaystyle e^{x_k}\cdot1!\,b_1 = b_1\cdot1! + b_1x_k \left(1 + \frac{x_k}{2} + \frac{x_k^2}{2\cdot3} +\ldots \right), \\ \displaystyle e^{x_k}\cdot2!\,b_2 = b_2\cdot2!\left(1 + \frac{x_k}{1!} \right) + b_2x_k^2 \left(1 + \frac{x_k}{3} + \frac{x_k^2}{3\cdot4} + \ldots\right), \\ \displaystyle e^{x_k}\cdot3!\,b_3 = b_3\cdot3!\left(1 + \frac{x_k}{1!} + \frac{x_k^2}{2!} \right) + b_3x_k^3\left(1 + \frac{x_k}{4} + \frac{x_k^2}{4\cdot5} + \ldots \right), \\ \hdotsfor[10]{1} \\ \displaystyle e^{x_k}\cdot s!\,b_s = b_s\cdot s!\left(1 + \frac{x_k}{1!} + \ldots + \frac{x_k^{s-1}}{(s-1)!} \right) \\ \displaystyle \hfill + b_s x_k^s\left(1 + \frac{x_k}{s+1} + \frac{x_k^2}{(s+1)(s+2)} + \ldots \right). \end{array} \right\} \tag{4} \]} The numbers $b_1,\ldots,b_{\text{\correction{$s$}{$n$}}}$ may be regarded as the coefficients of an arbitrary polynomial \[ \phi(x)=b_0+b_1x+b_2x^2+\ldots+b_s x^s, \] for which \[ \phi^{(m)}(x)=b_m\cdot m!+b_{m+1}\cdot\frac{(m+1)!}{1!}\cdot x + \ldots + b_s\frac{s!}{(s-m)!}\cdot x^{s-m}. \] The diagonal in equations~\hyperlink{eq4p26}{(4)} from $b_1\cdot1!$ to $b_s\cdot s!\frac{{x_k}^{s-1}}{(s-1)\text{\correction{$!$}{}}}$ is obviously $\phi'(x_k)$, and the next lower diagonal $\phi''(x_k)$, etc. Therefore, by adding equations~\hyperlink{eq4p26}{(4)}, \hypertarget{eq5p26}{\begin{multline*} e^{x_k}(1!\,b_1+2!\,b_2+\ldots+s!\,b_s) = \phi'(x_k)+\phi''(x_k)+\ldots \\ +\phi^{(s)}(x_k)+\sum_{m=1}^s b_m\cdot x_k^mR_{km}, \tag{5} \end{multline*}} %-----File: 039.png---Folio 27------- in which \[ R_{km} = 1 + \frac{x_k}{m+1} + \frac{x_k^2}{(m+1)(m+2)}+ \ldots \] Remembering that $\phi(x)$ is perfectly arbitrary, let it be so chosen that \[ \phi'(x_k) = 0,\; \phi''(x_k)=0,\; \phi'''(x_k)=0, \; \ldots, \phi^{(p-1)}(x_k)=0 \] for every $x_k$. Equation~\hyperlink{eq5p26}{(5)} may then be written as follows: \hypertarget{eq6p27}{\begin{align*} e^{x_k}(1!\,b_1 + 2!\,b_2 + \ldots + s!\,b_s) &= \sum_{m=1}^s b_m \cdot (x_k)^m \cdot R_{\text{\correction{$km$}{$k,m$}}} \\ &+ b_p \cdot p! \\ &+ b_{p+1} \cdot (p+1)! \left(1+\frac{x_k}{1!} \right) \\ &+\ldots \\ \tag{6} &+ b_s \cdot s! \left(1 + \frac{x_k}{1!} + \frac{x^2_k}{2!} + \ldots + \frac{x_k^{s-p}}{(s-p)!} \right). \end{align*}} A choice of $\phi(x)$ satisfying the required conditions is \begin{align*} \phi(x) &= \frac{a_n^{np-1} \cdot x^{p-1}}{(p-1)!} (a_0 + a_1 x + a_2 x^2 + \ldots + a_n x^n)^p \\ &= \frac{a_n^{np-1} \cdot x^{p-1}}{(p-1)!} (f(x))^p, \end{align*} of which every $x_k$ is a $p$-tuple root. If $\phi(x)$ is expanded and the result compared with \[ \phi(x) = b_0 + b_1 x + \ldots + b_s x^s, \] it is plain that $b_0=0$, $b_1=0$, \ldots, $b_{p-2}=0$, on account of the factor $x^{p-1}$; and \[ b_{p-1} = \frac{a_0^p a_n^{np-1}}{(p-1)!}, \quad b_p = \frac{I_p \cdot a_n^{np-1}}{(p-1)!} \quad \ldots, \quad b_s = \frac{I_s \cdot a_n^{np-1}}{(p-1)!}, \] %-----File: 040.png---Folio 28------- where $I_p,\ldots,I_s$, are all integers. The coefficient of $e^{x_k}$ in \hyperlink{eq6p27}{(6)} may now be written \[ N_p = a_n^{np-1} \left(a_0^p + \frac{I_p}{(p-1)!}\cdot p! + \frac{I_{p+1}}{(p-1)!}(p+1)! + \ldots + \frac{I_s}{(p-1)!} \cdot s! \right)\text{\correction{.}{}} \] If the arbitrary number $p$ is chosen as a prime number greater than $a_0$ and $a_n$, $N_p$ becomes the sum of $a_0^pa_n^{np-1}$, which cannot contain $p$ as a factor, and a number of other integers each of which is divisible by $p$. $N_p$ therefore is \textit{not zero and not divisible by $p$}. Further, since, $\dfrac{(p+t)!}{(p-1)!\cdot r!}$ is an integer divisible by $p$ when $r \leqq t$, it follows that all of the coefficients of the last block of terms in \hyperlink{eq6p27}{(6)} contain $p$ as a factor. If then \hyperlink{eq6p27}{(6)} is added by columns, \hypertarget{eq7p28}{\[ N_pe^{\text{\correction{$x_k$}{$xk$}}} = pa_n^{np-1} \left[ P_0 + P_1x_k + P_2x_k^2 + \ldots + P_{s-p}x_k^{s-p} \right] + \sum_{m=1}^s b_m\cdot x_k^m \cdot R_{km} \tag{7} \]} where $P_0,P_1,\ldots,P_{s-p}$ are integers. It remains to show that $\sum_{m=1}^s b_m \cdot x_k^m \cdot R_{km}$ can be made small at will by a suitable choice of the arbitrary $p$. As in the proof of the transcendence of $e$, it follows that \[ \left|r_{kp} \right| = \left|\sum_{m=1}^s b_m \cdot x_k^m \cdot R_{km} \right| < \frac{Q^p}{(p-1)!} \cdot e^\alpha, \] where \[ Q = |a_n^n| \alpha(|a_0|+ |a_1|\alpha + \ldots + |a_n|\alpha ), \] and $\alpha$ is the largest of the absolute values of $x_k$ ($k=1, \ldots, n$). If now $p$ is chosen as a prime number, greater than unity, greater than $a_0 \ldots a_n$ and greater than $c$, and so great also that $|r_{kp}|< \dfrac{1}{n}$, it follows directly from equation~\hyperlink{eq7p28}{(7)} that %-----File: 041.png---Folio 29------- \begin{multline*} \hypertarget{eq8p29}{\tag{8} N_p(c + e^{x_1} + e^{x_2} + \ldots + e^{x_n})} \\ = N_pc + p{a_n}^{np-1} (P_0S_0 + P_1S_1 + \ldots + P_{s-p}S_{s-p}) + \sum_{k=1}^n r_{kp}, \end{multline*} where \[ |r_{kp}| = \left|\sum_{m=1}^s b_m \cdot x_k^m \cdot R_{km} \right| < \frac1n, \] $S_0=n$, and $S_i=x_1^i + x_2^i + x_3^i + \ldots + x_n^i$, and therefore \begin{align*} S_1 &=-\frac{a_{n-1}}{a_n}, &S_2 &= \frac{a_{n-1}^2}{a_n^2}-\frac{2a_{n-2}}{a_n},\ldots, \footnotemark \end{align*} \footnotetext{% Cf.~\textsc{Burnside} and \textsc{Panton} \textit{Theory of Equations}, Chapter~VIII, Vol.~I.} and therefore it follows that ${a_n}^{np-1}S_1$, ${a_n}^{np-1}S_2$,\ldots, are all whole numbers or zero. The term \[ pa_n^{np-1} \cdot \sum_{i=0}^{s-p} P_iS_i \] is therefore an integer divisible by $p$, while, on the contrary, $N_p$ and $c$ are not divisible by $p$. The sum of these terms is therefore a whole number $\geqq +1$ or $\leqq-1$, and since $\displaystyle \sum_{k=1}^nr_{kp} < 1$, the entire right-hand member of \hyperlink{eq8p29}{(8)} is not zero, and hence \hyperlink{eq3p25}{(3)} is not zero. Therefore--- \begin{theorem}[9]\hypertarget{thm9}{} The number $\pi$ is transcendental. \end{theorem} %-----File: 042.png---Folio 30------- \chapter{SETS OF POINTS AND OF SEGMENTS.}\hypertarget{chapII}{}%[II] \section{Correspondence of Numbers and Points.}\hypertarget{chIIsec1}{}%[1] The system of real numbers may be set into \index{One-to-one correspondence}one-to-one correspondence with the points of a straight line. That is, a scheme may be devised by which every number corresponds to one and only one point of the line and vice versa. The point $0$ is chosen arbitrarily, and the points $1, 2, 3, 4, \ldots$ are at regular intervals to the right of $0$ in the order $1, 2, 3, 4, \ldots$ from left to right, while the points $-1,-2,-3,\ldots$ follow at regular intervals in the order $0, -1,-2,-3,\ldots$ from right to left. The points which correspond to fractional numbers are at intermediate positions as follows:\footnote{% It is convenient to think of numbers in this case as simply a notation for points. In view of the correspondence of points and numbers the numbers furnish a complete notation for all points.} To fix our ideas we obtain a point corresponding to a particular decimal of a finite number of digits, say $1.32$. \begin{figure}[!hbtp]\label{fig01}\hypertarget{fig01}{} \centering \setlength{\unitlength}{0.06\textwidth} \begin{picture}(10,1.4)(-5,-0.6) \scriptsize \put(-5,0){\line(1,0){10}} \put(-5,0){\line(0,1){0.25}} \put(-4,0){\line(0,1){0.25}} \put(-3,0){\line(0,1){0.25}} \put(-2,0){\line(0,1){0.25}} \put(-1,0){\line(0,1){0.25}} \put(0,0){\line(0,1){0.25}} \put(5,0){\line(0,1){0.25}} \put(-1.8,0){\line(0,1){0.65}} \put(-1.8,0.8){\makebox(0,0)[cc]{$1.32$}} \put(-5,0.5){\makebox(0,0)[cc]{$1$}} \put(-4,0.5){\makebox(0,0)[cc]{$.1$}} \put(-3,0.5){\makebox(0,0)[cc]{$.2$}} \put(-2,0.5){\makebox(0,0)[cc]{$.3$}} \put(-1,0.5){\makebox(0,0)[cc]{$.4$}} \put(0,0.5){\makebox(0,0)[cc]{$.5$}} \put(5,0.5){\makebox(0,0)[cc]{$1$}} \normalsize \put(0,-0.3){\makebox(0,0)[tc]{\sc Fig.~1}} \end{picture} \end{figure} Divide the segment $\overline{1\ 2}$ into ten equal parts. Then divide the segment \correction{$\overline{.3\ .4}$}{$\overline{3\ 4}$} of this division into ten equal parts. The point marked $2$ by the last division is the point corresponding to $1.32$. If the decimal is not terminating, we simply obtain an infinite sequence of points, such that any one is to the right of all that precede it, in case of a positive number, or to the %-----File: 043.png---Folio 31------- left in case of a negative number. The first few points of the sequence for the number $\pi$ are the points corresponding to the numbers $3$, $3.1$, $3.14$, $3.141$. This set of numbers is bounded, $4$, for instance, being an upper bound. Hence the points corresponding to these numbers all lie to the left of the point corresponding to the number $4$. To show that there exists a definite point corresponding to the least upper bound \correction{$\overline{B}$}{$B$} of the set of numbers $3$, $3.1$, $3.14$, $3.141$, etc., use is made of the following: \begin{other}[Postulate of Geometric Continuity]\index{Axioms!of continuity}\index{Continuity!axioms of}If a set $[x]$ of points of a line has a right bound, that is, if there exists a point $B$ on the line such that no point of the set $[x]$ is to the right of $B$, then there exists a leftmost right bound $\overline{B}$ of the set $[x]$. If the set has a left bound, it has a rightmost left bound. \end{other} The leftmost right bound of the set of points corresponding to the numbers $3.$, $3.1$, $3.14$, etc., is the point which corresponds to the number $\pi$. In the same manner it follows from the postulate that there is a definite point on the line corresponding to any decimal with an infinitude of digits.\footnote{% It is not implied here, of course, that it is possible to write a decimal with an infinitude of digits, or to mark the corresponding points. What is meant is that if an infinite sequence of digits is determined, a definite number and a definite point are thereby determined. Thus $\sqrt{2}$ determines an infinite sequence of digits, that is, it furnishes the law whereby the sequence can be extended at will.} Conversely, given any point on the line, e.g., a point $P$, to the right of $0$, there corresponds to it one and only one number. This is evident since, in dividing the line according to a decimal scale, either the point in question is one of the division-points, in which case the number corresponding to the point is a terminating decimal, or in case it is not a division-point we will have an infinite set of division\correction{-}{ }points to the left of it, the point in question being the leftmost right bound of the set. If now we pick out the rightmost point of this left set in every division and note the corresponding number, we have a set of numbers whose least upper bound corresponds to the point $P$. %-----File: 044.png---Folio 32------- The ordinary analytic geometry furnishes a scheme for setting all pairs of real numbers into correspondence with all points of a plane, and all triples of real numbers into correspondence with all points in space. Indeed, it is upon this correspondence that the analytic geometry is based. It should be noticed that the correspondence between numbers and points on the line preserves order, that is, if we have three numbers, $a$, $b$, $c$, so that $a < b < c$, then the corresponding points $A$, $B$, $C$ are under the ordinary conventions so arranged that $B$ is to the right of $A$, and $C$ to the right of $B$. It will be observed that we have not put this matter of the one-to-one correspondence between points and numbers into the form of a theorem. Rather than aiming at a rigorous demonstration from a body of sharply stated axioms, we have attempted to place the subject-matter before the reader in such a manner that he will understand on the one hand the necessity, and on the other the grounds, for the hypothesis. \section{Segments and Intervals. Theorem of Borel.}\hypertarget{chIIsec2}{}%[2] \begin{definition}\index{Segment} A \textit{segment} $\overline{a\ b}$ is the set of all numbers greater than $a$ and less than $b$. It does not include its end-points $a$ and $b$. An \index{Interval}\textit{interval} $\interval{a}{b}$ is the segment $\overline{a\ b}$ together with $a$ and $b$. For a segment plus its end point $a$ we use the notation $\linterval{a}{b}$, and when $a$ is absent and $b$ present $\rinterval{a}{b}$. All these notations imply that $a0$, $y=0$, $y=b>0$ determine the boundary of $P$. Let $0\leqq y_1\leqq b$. Upon the interval $i$ of the line %-----File: 049.png---Folio 37------- $y=y_1$, cut off by $P$, those parallelograms of $[p]$ that include points of $i$ as interior points determine a set of segments $[\pi]$ such that every point of $i$ is an interior point of one of these segments $\pi$. There is by Theorem~\hyperlink{thm10}{10} a finite subset of $[\pi]$, $\pi_1$ $\ldots$ $\pi_n$, including every point of $i$, and therefore a finite subset $p_1$ $\ldots$ $p_n$ of $[p]$, including as interior points every point of $i$. Moreover, since the number of $p_1$ $\ldots$ $p_n$ is finite, they include in their interior all the points of a definite strip, e.g., the points between the lines $y=y_1-e$ and $y=y_1+e$. \begin{figure}[!htpb]\label{fig04}\hypertarget{fig04}{} \centering \setlength{\unitlength}{0.008\textwidth} \begin{picture}(120,50)(-60,-25) \scriptsize \put(-50,-20){\line(0,1){40}} \put(40,-20){\line(0,1){40}} \put(-50,-20){\line(1,0){90}} \put(-50,20){\line(1,0){90}} \put(-50,0){\line(1,0){97}} \dashline{1}(-50,4)(47,4) \dashline{1}(-50,-4)(47,-4) \put(-56,-5){\line(0,1){9}} \put(-56,4){\line(1,0){16}} \put(-56,-5){\line(1,0){16}} \put(-40,-5){\line(0,1){9}} \put(-44,-7){\line(0,1){17}} \put(-44,10){\line(1,0){22}} \put(-44,-7){\line(1,0){22}} \put(-22,-7){\line(0,1){17}} \put(-30,-9){\line(0,1){16}} \put(-30,7){\line(1,0){19}} \put(-30,-9){\line(1,0){19}} \put(-11,-9){\line(0,1){16}} \put(-17,-4){\line(0,1){12}} \put(-17,8){\line(1,0){20}} \put(-17,-4){\line(1,0){20}} \put(3,-4){\line(0,1){12}} \put(-5,-5){\line(0,1){10}} \put(-5,5){\line(1,0){12}} \put(-5,-5){\line(1,0){12}} \put(7,-5){\line(0,1){10}} \put(6,-15){\line(0,1){30}} \put(6,15){\line(1,0){15}} \put(6,-15){\line(1,0){15}} \put(21,-15){\line(0,1){30}} \put(20,-9){\line(0,1){18}} \put(20,9){\line(1,0){25}} \put(20,-9){\line(1,0){25}} \put(45,-9){\line(0,1){18}} \put(-49,-21){\makebox(0,0)[tl]{$y=0$}} \put(-49,19){\makebox(0,0)[tl]{$y=b$}} \put(-54,-13){$x=0$} \put(36,-13){$x=a$} \put(48,4){$y_1+e$} \put(48,0){$y_1$} \put(48,-4){$y_1-e$} \normalsize \put(0,-22){\makebox(0,0)[tc]{\textsc{Fig.~4.}}} \end{picture} \end{figure} Thus for every $y_1$ $(0\leqq y_1\leqq b)$ we obtain a strip of the parallelogram $P$ such that every point of its interior is interior to one of a finite number of the parallelograms $[p]$. These strips intersect the $y$-axis in a set of segments that include every point of the interval $\interval{0}{b}$. There is therefore, by Theorem~\hyperlink{thm10}{10}, a finite set of strips which includes every point in $P$. Since each strip is included by a finite number of parallelograms $p$, the whole parallelogram $P$ is included by a finite subset of $[p]$. \end{proof} The generalization of Theorems \hyperlink{thm11}{11} and \hyperlink{thm12}{12} is left to the reader. \section{Limit Points. Theorem of Weierstrass.}\hypertarget{chIIsec3}{}%[3] \begin{definition} A \emph{neighborhood} or \emph{vicinity}\index{Vicinity} of a point $a$ in a line (or simply a line neighborhood of $a$) is a segment of this line such that $a$ lies within the segment. We denote a line neighborhood %-----File: 050.png---Folio 38------- of a point $a$ by $V(a)$\index{Vofa@$V(a)$}. The symbol $V^*(a)$\index{Vstarofa@$V^*(a)$} denotes the set of all points of $V(a)$ except $a$ itself. The symbols $V(\infty)$ and $V^*(\infty)$ are both used to denote infinite segments $\overline{a\ +\infty}$, and $V(-\infty)$ and $V^*(-\infty)$ to denote infinite segments $\overline{-\infty\ a}$.\footnote{% This notation is taken from \textsc{Pierpont's} \textit{Theory of Functions of Real Variables}. It is used here, however, with a meaning slightly different from that of \textsc{Pierpont}.} \index{Neighborhood}A neighborhood of a point in a plane (or a plane neighborhood of a point) is the interior of a parallelogram within which the point lies. A neighborhood of a point $(a,b)$ is denoted by $V(a,b)$ if $(a,b)$ is included and by $V^*(a,b)$ if $(a,b)$ is excluded. Instead of the three linear vicinities $V(a)$, $V(\infty)$, and $V(-\infty)$ we have the following nine in the case of the plane: \begin{figure}[!hbtp]\label{fig05}\hypertarget{fig05}{} \centering \includegraphics{images/fig05} %\correction{$V(-\infty,-\infty)$}{$V(-\infty,\infty)$} \end{figure} %-----File: 051.png---Folio 39------- \end{definition} It follows at once from a consideration of the scheme for setting the points on the line into correspondence with all numbers that in every neighborhood of a point there is a point whose corresponding number is rational. \begin{definition}\index{Limit!point} A point $a$ is said to be a \textit{limit point} of a set if there are points of the set, other than $a$, in every neighborhood of $a$. In case of a line neighborhood this says that there are points of the set in every $V^*(a)$. In the planar case this is equivalent to saying that $(a,b)$ is a limit point of the set $[x,y]$, either if for every $V^*(a)$ and $V(b)$ there is an $(x,y)$ of which $x$ is in $V^*(a)$ and $y$ in $V(b)$, or if for every $V(a)$ and $V^*(b)$ there is an $(x,y)$ of which $x$ is in $V(a)$ and $y$ in $V^*(b)$. \end{definition} Thus $0$ is a limit point of the set $\left[\tfrac{1}{2^k}\right]$, where $k$ takes all positive integral values. In this case the limit point is not a point of the set. On the other hand, in the set $1$, $1-\frac12$, $1-\frac{1}{2^2}$,\ldots, $1-\frac{1}{2^k}$, $1$ is a limit point of the set and also a point of the set. In this case $1$ is the least upper bound of the set. In case of the set $1$, $2$, $3$, the number $3$ is the least upper bound without being a limit point. The fundamental theorem about limit points is the following (due to \textsc{Weierstrass}): \begin{theorem}[15]\hypertarget{thm15}{} Every infinite bounded set $[p]$ of points on a line has at least one limit point. \end{theorem} \begin{proof} Since the set $[p]$ is bounded, every one of its points lies on a certain interval $\interval{a}{b}$. If the set $[p]$ has no limit point, then about every point of the interval $\interval{a}{b}$ there is a segment $\sigma$ which contains not more than one point of the set $[p]$. By Theorem~\hyperlink{thm10}{10} there is a finite set of the segments $[\sigma]$ such that every point of $\interval{a}{b}$ and hence of $[p]$ belongs to at least one of them, but each $\sigma$ contains at most one point of the set $[p]$, whence $[p]$ is a finite set of points. Since this is contrary to the hypothesis, the assumption that there is no limit point is not tenable. \end{proof} %-----File: 052.png---Folio 40------- It is customary to say that a set which has no finite upper bound has the upper bound \index{Infinity as a limit}$+\infty$, and that one which has no finite lower bound has the lower bound $-\infty$. In these cases, since the set has a point in every $V^*(+\infty)$ or in every $V^*(-\infty)$ $+\infty$ and $-\infty$ are also called limit points. With these conventions the theorem may be stated as follows: \begin{theorem}[16]\hypertarget{thm16}{} Every infinite set of points has a limit point, finite or infinite. \end{theorem} The theorem also generalizes in space of any number of dimensions. In the planar case we have: \begin{theorem}[17]\hypertarget{thm17}{} An infinite set of points lying entirely within a parallelogram has at least one limit point. \end{theorem} Theorem~\hyperlink{thm17}{17} is a corollary of the stronger theorem that follows: \begin{theorem}[18]\hypertarget{thm18}{} If $[(x,y)]$ is any set of number pairs and if $a$ is a limit point of the numbers $[x]$, there is a value of $b$, finite or $+\infty$ or $-\infty$, such that for every $V^*(a)$ and $V(b)$ there is an $(x,y)$ of which $x$ is in $V^*(a)$ and $y$ is in $V(b)$. \end{theorem} \begin{proof} Suppose there is no value $b$ finite or $+\infty$ or $-\infty$ such as is required by the theorem. Since neither $+\infty$ nor $-\infty$ possesses the property required of $b$, there is a $\overline{V^*}(a)$ and a $V(\infty)$ and a $V(-\infty)$ such that for every pair $(x,y)$ of $[(x,y)]$ whose $x$ lies in $\overline{V^*}(a)$ $y$ fails to lie in either $V(\infty)$ or $V(-\infty)$. This means that there exists a pair of numbers $M$ and $m$ such that for every $(x, y)$ whose $x$ is in $\overline{V^*}(a)$ the $y$ satisfies the condition $m0$, however small, there is some $n$, say $n_e$, such that $|b_{n_e}-a_{n_e}|< e$. \end{enumerate} \emph{Conclusion:} There is one and only one point $b$ which lies upon every interval $\interval{a_n}{b_n}$. \end{lemma} \begin{proof} Since the set of points $a_1\ldots a_n\ldots$ is bounded, we have at once, by the postulate of continuity, that this set has a leftmost right bound $\overline{B}_a$. Similarly, the set $b_1\ldots b_n\ldots$ has a rightmost left bound $\underline{B}_b$. It follows at once that $\overline{B}_a=\underline{B}_b$, for if not, we get either an $a$ point to the right of $\overline{B}_a$, or a $b$ point to the left of $\underline{B}_b$ when $n_e$ is so chosen that $|b_{n_e}-a_{n_e}|< \overline{B}_a-\underline{B}_b$. \end{proof} We now give another proof for Theorem~\hyperlink{thm11}{11}. Divide the interval $\interval{a}{b}$ on which all points of $[p]$ lie into two equal intervals. Then there is an infinite number of points $[p]$ on at least one of these intervals which we call $\interval{a_1}{b_1}$. Divide this interval %-----File: 055.png---Folio 43------- into two equal parts and so on indefinitely, always selecting for division an interval which contains an infinite number of points of the set $[p]$. We thus obtain an infinite sequence of intervals $\interval{a_1}{b_1}$, $\interval{a_2}{b_2}$, $\ldots$, $\interval{a_n}{b_n} \ldots$ which satisfies the hypothesis of the lemma. There is therefore a point $B$ which belongs to every one of the intervals $\interval{a_1}{b_1}$, $\interval{a_2}{b_2}$, $\ldots$, $\interval{a_n}{b_n} \ldots$, and therefore there is a point of the set $[p]$ in every neighborhood of $B$. It should be noticed that the intervals in this sequence may be such that all intervals after a certain one will have, say, the right extremities in common. In this case the right extremity is the point $B$. Such is the sequence, obtained by decimal division, representing the number $2=1.99999 \ldots$. %-----File: 056.png---Folio 44------- \chapter{FUNCTIONS IN GENERAL\@. SPECIAL CLASSES OF FUNCTIONS.}\hypertarget{chapIII}{}%[III] \section{Definition of a Function.}\hypertarget{chIIIsec1}{}%[1] \index{Function} \begin{definition}\index{Constant} A \emph{variable} is a symbol which represents any one of a set of numbers. A \emph{constant} is a special case of a variable where the set consists of but one number. \end{definition} \begin{definition} A variable $y$ is said to be a \index{Single-valued functions}\emph{single-valued function} of another variable $x$ if to every value of $x$ there corresponds one and only one value of $y$. The letter $x$ is called the \emph{independent}\index{Variable!independent}\index{Independent variable} variable and $y$ the \emph{dependent}\index{Variable!dependent}\index{Dependent variable} variable.\footnote{% \protect\hypertarget{fn}{}This definition of function is the culmination of a long development of the use of the word. The idea of function arose in connection with coordinate geometry, \textsc{Ren\'e Descartes} using the word as early as 1637. From this time to that of \textsc{Leibnitz} ``function'' was used synonymously with the word ``power,'' such as $x^2$, $x^3$, etc. \textsc{G.~W.~Leibnitz} regarded ``function'' as ``any expression standing for certain lengths connected with a curve, such as coordinates, tangents, radii of curvature, normals, etc.'' \textsc{Johann Bernoulli} (1718) defined ``function'' as ``an expression made up of one variable and any constants whatever.'' \textsc{Leonard Euler} (1734) called the expression described by \textsc{Bernoulli} an analytic function and introduced the notation $f(x)$. \textsc{Euler} also distinguished between algebraic and transcendental functions. He wrote the first treatise on ``The Theory of Functions.'' The problem of vibrating strings led to the consideration of trigonometric series. \textsc{J.~B.~Fourier} set the problem of determining what kind of relations can be expressed by trigonometric series. The possibility then under consideration that any relation might be so expressed led \textsc{Lejeune Dirichlet} to state his celebrated definition, which is the one given above. See the Encyclop\"adie der mathematischen Wissenschaften, II~A.~1, pp.~3--5; also \textsc{Ball}'s History of Mathematics, p.~378.} \end{definition} \begin{definition}\index{Many-valued function} A variable $y$ is said to be a many-valued function or multiple-valued function of another variable $x$ if to every value of $x$ there correspond one or more values of $y$. The class of multiple-valued functions thus includes the class of single-valued functions.\hyperlink{fn}{\footnotemark[1]} \end{definition} %-----File: 057.png---Folio 45------- It is sometimes convenient to think of special values taken by these two variables as arranged in two tables, one table containing values of the independent variable and the other containing the corresponding values of the dependent variable. \begin{center} \begin{tabular}{r|l} Independent Variable & Dependent Variable\\ \hline $x_1$ & $y_1$\\ $x_2$ & $y_2$\\ $\,\cdot\,$ & $\,\cdot\,$ \\ $\,\cdot\,$ & $\,\cdot\,$\\ $\,\cdot\,$ & $\,\cdot\,$\\ $x_n$ & $y_n$ \end{tabular} \end{center} If $y$ is a single-valued function of $x$, one and only one value of $y$ will appear in the table for each $x$. It is evident that functionality is a reciprocal relation; that is, if $y$ is a function of $x$, then $x$ is a function of $y$. It does not follow, however, that if $y$ is a single-valued function of $x$, then $x$ is a single-valued function of $y$, e.g., $y=x^2$. It is also to be noticed that such tables cannot exhibit the functional relation completely when the independent variable takes all values of the continuum, since no table contains all such values. \begin{definition}\label{dp45} That $y$ is a function of $x$ (and hence that $x$ is a function of $y$) is expressed by the equation $y=f(x)$ or by $x=f^{-1}(y)$. If $y$ and $x$ are connected by the equation $y=f(x)$, \index{Function!inverse}\index{Inverse function}$f^{-1}(y)$ is called the inverse function of $f(x)$. \end{definition} Thus $y=x^2$ has the inverse function $x=\pm\sqrt y$. In this case, while the first function $y=x^2$ is defined for all real values of $x$, the inverse function $x = \pm\sqrt y$ is defined only for positive values of $y$. The independent variable may or may not take all values between any two of its values. Thus $n!$ is a function of $n$ where $n$ takes only integral values. $S_n$, the sum of the first %-----File: 058.png---Folio 46------- $n$ terms of a series, is a function of $n$ where $n$ takes only integral values. Again, the amount of food consumed in a city is a function of the number of people in the city, where the independent variable takes on only integral values. Or the independent variable may take on all values between any two of its values, as in the formula for the distance fallen from rest by a body in time $t$, $s=\dfrac{gt^2}{2}$. It follows from the correspondence between pairs of numbers and points in a plane that the functional relation between two variables may be represented by a set of points in a plane. The points are so taken that while one of the two numbers which correspond to a point is a value of the independent variable, the other number is the corresponding value, or one of the corresponding values, of the dependent variable. Such representations are called \index{Function!graph of}\index{Graph of a function}graphs of the function. Cases in point where the function is single-valued are: the hyperbola referred to its asymptotes as axes $\left(y=\dfrac{1}{x}\right)$; a straight line not parallel to the $y$ axis $(y=ax+b)$; or a broken line such that no line parallel to the $y$ axis contains more than one of its points. In general, the graph of a single-valued function with a single-valued inverse is a set of points $[(x, y)]$ such that no two points have the same $x$ or the same $y$. Following is a graph of a function where the independent variable does not take all values between any two of its values. Consider $S_n$, the sum of the first $n$ terms as a function of $n$ in the series \[ S = 1+\frac12+\frac{1}{2^2}+\ldots+\frac{1}{2^{n-1}}+\ldots. \] The numbers on the $x$ axis are the values taken by the independent variable, while the functional relation is represented by the points within the small circles. Thus it is seen that the graph of this function consists of a discrete set of points. (Fig.~\hyperlink{fig06}{6}.) %-----File: 059.png---Folio 47------- The definition of a function here given is very general. It will permit, for instance, a function such that for all rational values of the independent variable the value of the function is unity, and for irrational values of the independent variable the value of the function is zero. \begin{figure}[!hbtp]\label{fig06}\hypertarget{fig06}{} \centering \setlength{\unitlength}{0.08\textwidth} \begin{picture}(10,6)(0,-0.5) \thicklines \put(0,0){\line(1,0){10}} \put(0,0){\line(0,1){5.5}} \thinlines \multiput(1,0)(1,0){5}{\line(0,1){3}} \put(1,1.5){\circle{0.2}} \put(2,2.25){\circle{0.2}} \put(3,2.625){\circle{0.2}} \put(4,2.8125){\circle{0.2}} \put(5,2.90625){\circle{0.2}} \dashline{0.1}(0,3)(6,3) \put(10,0.25){\makebox(0,0)[br]{$x$}} \put(0.25,5.5){\makebox(0,0)[tl]{$y$}} \put(5,-0.25){\makebox(0,0)[tc]{\textsc{Fig.~6.}}} \end{picture} \end{figure} \section{Bounded Functions.}\hypertarget{chIIIsec2}{}%[2] Since the definition of function is so general there are few theorems that apply to all functions. If the restriction that $f(x)$ shall be bounded is introduced, we have at once a very important theorem. \begin{definition}\index{Bounds!upper and lower} \index{Function!upper and lower bound of}\index{Upper bound!of a function}\index{Lower bound!of a function}A function, $f(x)$, has an \textit{upper bound for a set of values $[x]$} of the independent variable if there exists a finite number $M$ such that $f(x)m$ for every value of $x$ in $[x]$. A function which for a given set of values of $x$ has no \index{Infinity as a limit}finite upper bound is said to be \index{Function!unbounded}\index{Unbounded function}unbounded on that set, or to have an upper bound $+\infty$ on that set, and if it has %-----File: 060.png---Folio 48------- no lower bound on the set the function is said to have the lower bound $-\infty$ on the set. \end{definition} \begin{theorem}[19]\hypertarget{thm19}{} If on an interval $\interval{a}{b}$ a function has an upper bound $M$, then it has a least upper bound $\overline{B}$, and there is at least one value of $x$, $x_1$ on $\interval{a}{b}$ such that the least upper bound of the function on every neighborhood of $x_1$ contained in $\interval{a}{b}$ is $\overline{B}$. \end{theorem} \begin{proof} (1) The set of values of the function $f(x)$ form a bounded set of numbers. By Theorem~\hyperlink{thm4}{4} the set has a least upper bound $\overline{B}$. (2) Suppose there were no point $x_1$ on $\interval{a}{b}$ such that the least upper bound on every neighborhood of $x_1$ contained in \correction{$\interval{a}{b}$}{$\interval{a\text{---}}{!b}$} is $\overline{B}$. Then for every $x$ of $\interval{a}{b}$ there would be a segment $\sigma_x$ containing $x$ such that the least upper bound of $f(x)$ for values of $x$ common to $\sigma_x$ and $\interval{a}{b}$ is less than $\overline{B}$. The set $[\sigma_x]$ is infinite, but by Theorem~\hyperlink{thm10}{10} there exists a finite subset $[\sigma_n]$ of the set $[\sigma_x]$ covering $\interval{a}{b}$. Therefore, since the upper bound of $f(x)$ is less than $\overline{B}$ on that part of every one of these segments of $[\sigma_n]$ which lies on $\interval{a}{b}$, it follows that the least upper bound of $f(x)$ on $\interval{a}{b}$ is less than $\overline{B}$. Hence the hypothesis that no point $x_1$ exists is not tenable, and there is a point $x_1$ such that the least upper bound of the function on every one of its neighborhoods which lies in $\interval{a}{b}$ is $\overline{B}$. \end{proof} This argument applies to multiple-valued as well as to single-valued functions. As an exercise the reader may repeat the above argument to prove the following: \begin{corollary} If on an interval $\interval{a}{b}$ a function has an upper bound $+\infty$, then there is at least one value of $x$, $x_1$ on $\interval{a}{b}$ such that in every neighborhood of $x_1$ the upper bound of the function is $+\infty$. \end{corollary} %-----File: 061.png---Folio 49------- \section{Monotonic Functions; Inverse Functions.}\hypertarget{chIIIsec3}{}%[3] \begin{definitions}\index{Decreasing function}\index{Function!monotonic!increasing}\index{Increasing function}\index{Function!monotonic!decreasing}\index{Monotonic function} If a single-valued function $f(x)$ on an interval $\interval{a}{b}$ is such that $f(x_1) f(x_2)$ whenever $x_1f(x_1)$ and $f(x_2)>f(x_3)$ while $x_1 \dfrac{m_1}{n_1}$, then %-----File: 067.png---Folio 55------- $a^{x_1}1$ and $a^{x_1}>a^{x_2}$ if $a<1$. The proof of this follows at once from case ($a$), since $a^\frac{m_1}{n_1}=\left(a^\frac{1}{n_1}\right)^{m_1}$ (by definition and elementary algebra) and $a^\frac{m_2}{n_1}=\left({a^\frac{1}{n_1}}\right)^{m_2}$. \item[(\textit{c})] If $x_1=\dfrac{m_1}{n_1}$ and $x_2=\dfrac{m_2}{n_2}$, where $\dfrac{m_1}{n_1}<\dfrac{m_2}{n_2}$, we have $a^\frac{m_1}{n_1}=a^\frac{m_1{\cdot}n_2}{n_1{\cdot}n_2}$ and $a^\frac{m_2}{n_2}=a^\frac{m_2{\cdot}n_1}{n_2{\cdot}n_1}$, where $m_1{\cdot}n_2\text{\correction{$<$}{$>$}}m_2{\cdot}n_1$, which reduces case (\emph{c}) to case (\emph{b}).\qedhere \end{enumerate} \end{proof} This theorem makes it natural to define $a^x$, where $a>1$ and $x$ is a positive irrational number, as the least upper bound of all numbers of the form $\left[a^\frac mn\right]$, where \correction{$\left[\dfrac{m}{n}\right]$}{$\dfrac{m}{n}$} is the set of all positive rational numbers less than $x$, i.e., $a^x = \overline{B}\left[a^\frac mn\right]$. It is, however, equally natural to define $a^x$ as $\underline{B}\left[a^\frac pq\right]$, where $\left[\dfrac{p}{q}\right]$ is the set of all rational numbers greater than $x$. We shall prove that the two definitions are equivalent. \begin{lemma} If $[x]$ is the set of all positive rational numbers, then \begin{align*} \underline{B}[a^x]&=1 \qquad \text{if } a>1\\ \intertext{and} \overline{B}[a^x]&=1 \qquad \text{if } a<1. \end{align*} \end{lemma} \begin{proof} We prove the lemma only for the case $a>1$, the argument in the other case being similar. If $x$ is any positive rational number, $\dfrac{m}{n}$, then the number $\dfrac{1}{n}$ is less than or equal to $x$, and since $a^x$ is a monotonic function, $a^\frac1n \qqle a^\frac mn$. But $\left[\dfrac{1}{n}\right]$ is a subset of $\left[\dfrac{1}{n}\right]$. Hence \[ \underline{B}[a^x]=\underline{B}\left[a^\frac1n\right], \] where $[n]$ is the set of all positive integers. \end{proof} %-----File: 068.png---Folio 56------- If $\underline{B}\left[ a^{\frac1n} \right]$ were less than $1$, then there would be a value, $n_1$, of $n$ such that $a^{\frac{1}{n_1}}<1$. This implies that $a<1$, which is contrary to the hypothesis. On the other hand, if $\underline{B}\left[a^{\frac1n}\right] > 1$, there is a number of the form $1+e$, where $e>0$, such that $1+e1+ne, \] and the latter expression is clearly greater than $a$ if \[ n>\frac ae. \] Since $\underline{B}\left[a^{\frac1n}\right]$ cannot be either greater or less than $1$, \[ \underline{B}\left[a^{\frac1n}\right] = 1. \] \begin{theorem}[22]\hypertarget{thm22}{} If $x$ is any real number, and $\left[ \dfrac{m}{n} \right]$ the set of all rational numbers less than $x$, and $\left[\dfrac{p}{q}\right]$ the set of all rational numbers greater than $x$, then \begin{align*} \overline{B}\left[a^{\frac{m\vphantom{p}}{n}}\right] &= \underline{B}\left[a^{\frac pq}\right] &&\text{if $a>1$,} \\ \underline{B}\left[a^{\frac{m\vphantom{p}}{n}}\right] &= \overline{B}\left[a^{\frac pq}\right] &&\text{if $01$, the other case being similar. By the lemma, since $\underline{B}\left[\frac{p}{q}-\frac{m}{n}\right]$ is zero, \[ \underline{B}\left[ a^{\frac pq}-\text{\correction{$a^{\frac mn}$}{$a^m_n$}}\right] = \underline{B}\left[ a^\frac pq \left(1-a^{\frac mn-\frac pq} \right) \right] \] is also zero. Now if \[ \overline{B}\left[a^{\frac{m\vphantom{p}}{n}}\right] \neq \underline{B}\left[a^{\frac pq}\right], \] %-----File: 069.png---Folio 57------- since $a^{\frac pq}$ is always greater than $a^\frac mn$, \[ \underline{B}\left[a^\frac pq\right]- \overline{B}\left[a^\frac{m\vphantom{p}}{n}\right] = \varepsilon > 0. \] But from this it would follow that \[ a^\frac pq-a^\frac mn \] is at least as great as $\varepsilon$, whereas we have proved that \[ \underline{B}\left[a^\frac pq-a^\frac mn \right] = 0.\\ \] Hence \[ \overline{B}\left[a^\frac{m\vphantom{p}}{n}\right] = \underline{B}\left[a^\frac pq\right] \] if $a>1$. \end{proof} \begin{definition}\label{dp57}In case $x$ is a positive irrational number, and $\left[\dfrac{p}{q}\right]$ is the set of all rational numbers greater than $x$, and $\left[\dfrac{m}{n}\right]$ is the set of all rational numbers less than $x$, then \begin{alignat*}{2} a^x &= \underline{B}\left[a^\frac pq\right] = \overline{B}\left[a^\frac{m\vphantom{p}}{n}\right]&\qquad&\text{if $a> 1$}\\ \intertext{and} a^x &= \overline{B}\left[a^\frac pq\right] = \underline{B}\left[a^\frac{m\vphantom{p}}{n}\right]&&\text{if $01$, and a monotonic decreasing function if $00$) to the \textit{base} $a$ ($a>0$) is a number $y$ such that $a^y=x$, or $a^{\log_a x}=x$. That is, the function $\log_a x$ is the inverse of $a^x$. The identity \begin{align*} a^{x_1} \cdot a^{x_2} &= a^{x_1 + x_2}\\ \intertext{gives at once} \log_a x_1 + \log_a x_2 &= \log_a (x_1 \cdot x_2), \end{align*} and \[ (a^{x_1})^{x_2}=a^{x_1 \cdot x_2}\quad\text{gives}\quad x_1\cdot \log_a x_2 = \log_a x_2^{x_1}. \] \end{definition} By means of Theorem~\hyperlink{thm20}{20}, the logarithm $\log_a x$, being the inverse of a monotonic function, is also a monotonic function, increasing if $1 < a$ and decreasing if $00) \] is monotonic increasing for all values of $a$, $a>0$, that its lower bound is zero and its upper bound is $+\infty$, and that it takes on all values between these bounds. The proof of these statements is left to the reader. The general type of the argument required is exemplified in the following, by means of which we infer some of the properties of the function $x^x$. If $x_10)$ is a monotonic increasing function of $x$. Since the upper bound of $x\cdot\log_2x=\log_2x^x$ is $+\infty$, the upper bound of $x^x$ is $+\infty$. The lower bound of $x^x$ is not negative, since $x>0$, and must not be greater than the lower bound of $2^x$, since if $x<2$, $x^x<2^x$; since the lower bound of $2^x$ is zero\footnote{% The lower bound of $a^x$ is zero by Theorem~\hyperlink{thm23}{23}.} the lower bound of $x^x$ must also be zero. Further theorems about these functions are to be found on pages \pageref{logp64}, \pageref{logp81}, \pageref{s4p97}, \pageref{p123}, and \pageref{t101p160}. %-----File: 072.png---Folio 60------- \chapter{THEORY OF LIMITS.}\hypertarget{chapIV}{}%[IV] \section{Definitions. Limits of Monotonic Functions.}\hypertarget{chIVsec1}{}%[1] \begin{definition} If a point $a$ is a limit point of a set of values taken by a variable $x$, the variable is said \emph{to approach $a$ upon} the set; we denote this by the symbol $x\doteq a$. $a$ may be finite or $+\infty$ or $-\infty$. \end{definition} In particular the variable may approach $a$ from the left or from the right, or in the case where $a$ is finite, the variable may take values on each side of the limit point. Even when the variable takes all values in some neighborhood on each side of the limit point it may be important to consider it first as taking the values on one side and then those on the other. \begin{definition}\index{Function!limit of}\index{Limit!of a function} A value $b$ ($b$ may be \index{Infinity as a limit}$+\infty$ or $-\infty$ or a finite number) is a \emph{value approached}\index{Value approached by!a function}\index{Function!value approached by} by $f(x)$ as $x$ approaches\index{Value approached by!the independent variable} $a$ if for every $V^*(a)$ and $V(b)$ there is at least one value of $x$ such that $x$ is in $V^*(a)$ and $f(x)$ in $V(b)$. Under these conditions $f(x)$ is also said to approach $b$ as $x$ approaches $a$. \end{definition} \begin{definition}\index{Convergence!to a limit} If $b$ is the only value approached as $x$ approaches $a$, then $b$ is called \emph{the limit of $f(x)$} as $x$ approaches $a$. This is also indicated by the phrase ``\emph{$f(x)$ converges to a unique limit $b$} as $x$ approaches $a$,'' or \index{Approach to a limit}``\emph{$f(x)$ approaches $b$ as a limit},'' or by the notation \[ \mathop{L}_{\text{\correction{$x\doteq a$}{$x=a$}}} f(x)=b. \] \end{definition} The function $f(x)$ is sometimes referred to as the \index{Limitand function}\emph{limitand}. The set of values taken by $x$ is sometimes indicated by the symbol for a limit, as, for example, %-----File: 073.png---Folio 61------- \begin{align*} \mathop{L}_{\substack{x>a \\x\doteq a}} f(x)=b &&\text{or} &&\mathop{L}_{\substack{xb$; then since $\overline{B}f(x) =b$, there would be no value of $f(x)$ between $b$ and $b'$, that is, there would be a $V(b')$ which could contain no value of $f(x)$, whence $b'>b$ is not a value approached. Suppose $b'b''$. If $x_1a\\x\doteq a}} f(x), \] nor that either of these limits is equal to $f(a)$. A case in point is the following: Let the temperature of a cooling body of water be the independent variable, and the amount of heat given out in cooling from a certain fixed temperature be the dependent variable. When the water reaches the freezing-point %-----File: 075.png---Folio 63------- a great amount of heat is given off without any change in temperature. If the zero temperature is approached from below, the function approaches a definite limit point $k$, and if the temperature approaches zero from above, the function \begin{figure}[!htbp]\label{fig12}\hypertarget{fig12}{} \centering \setlength{\unitlength}{0.05\textwidth} \begin{picture}(10,10)(0,-1) \put(0,0){\line(1,0){10}} \put(0,0){\line(0,1){9}} \path(1,2)(4,5)(4,7)(8,9) \put(5,-0.5){\makebox(0,0)[tc]{\sc Fig.~12}} \put(0,9){\makebox(0,0)[tl]{Heat}} \put(8,0){\makebox(0,0)[bc]{Temp.}} \end{picture} \end{figure} approaches an entirely different point $k'$. \index{Discontinuity}This function, however, is multiple-valued at the zero point. A case where the limit fails to exist is the following: The function $y=\sin\ 1/x$; (see Fig.~\hyperlink{fig08}{8}, page~\pageref{fig08}) approaches an infinite number of values as x approaches zero. The value of the function will be alternately $1$ and $-1$, as $x=\dfrac{2}{\pi}$, $\dfrac{2}{3\pi}$, $\dfrac{2}{5\pi}$ etc., and for all values of $x$ between any two of these the function will take all values between $1$ and $-1$. Clearly every value between $1$ and $-1$ is a value approached as $x$ approaches zero. In like manner %-----File: 076.png---Folio 64------- $y = \dfrac{1}{x}\sin\dfrac{1}{x}$ approaches all values between and including $+\infty$ and $-\infty$, cf.\ Fig.~\hyperlink{fig13}{13}. \begin{figure}[!hbtp]\label{fig13}\hypertarget{fig13}{} \centering \includegraphics{images/fig13} \end{figure} The functions $a^x$, $\log_a x$\label{logp64}, $x^a$ defined in \hyperlink{chIIIsec4}{\S~4} of the \hyperlink{chapIII}{last chapter} are all monotonic and all satisfy the condition that \[ \mathop{L}_{\substack{x>a\\x\doteq a}}f(x) = f(a) = \mathop{L}_{\substack{x0$ there exists a ${V_\varepsilon}^*(a)$ such that for every $x$ in ${V_\varepsilon}^*(a)$, $|f(x)-b|< \varepsilon$.} In case $a$ also is finite, the condition may be stated in a form which is frequently used as the definition of a limit, namely: \textit{$\displaystyle\mathop{L}_{x \doteq a} f(x) =b$ means that for every $\varepsilon>0$ there exists a $\delta_\varepsilon >0$ such that if $|x-a|< \delta_\varepsilon$ and $x\neq a$, then $|f(x)-b|< \varepsilon$.}\footnote{% The $\varepsilon$ subscript to $\delta_\varepsilon$ or to ${V_\varepsilon}^*(a)$ denotes that $\delta_\varepsilon$ or ${V_\varepsilon}^*(a)$ is a function of $\varepsilon$. It is to be noted that inasmuch as any number less than $\delta_\varepsilon$ is effective as $\delta_\varepsilon$, $\delta_\varepsilon$ is a multiple-valued function of $\varepsilon$.} \begin{theorem}[27]\hypertarget{thm27}{} A necessary and sufficient condition that $f(x)$ shall converge to a finite limit as $x$ approaches $a$ is that for every $\varepsilon>0$ there shall exist a ${V_\varepsilon}^*(a)$ such that if $x_1$ and $x_2$ are any two values of $x$ in ${V_\varepsilon}^*(a)$, then \[ |f(x_1)-f(x_2)|< \varepsilon. \] \end{theorem} \begin{proof} (1) \textit{The condition is necessary.} If $\displaystyle\mathop{L}_{x\doteq a} f(x)=b$ and $b$ is finite, then by the preceding theorem for every $\frac{\varepsilon}{2}>0$ there exists a $V^*(a)$ such that if $x_1$ and $x_2$ are in $V^*(a)$, then \begin{align*} |f(x_1)-b|&< \frac\varepsilon2 \\ \intertext{and} |f(x_2)-b|&< \frac\varepsilon2, \end{align*} from which it follows that \[ |f(x_1)-f(x_2)|< \varepsilon. \] %-----File: 079.png---Folio 67------- (2) \textit{The condition is sufficient.} If the condition is satisfied, there exists a $\overline{V^*}(a)$ upon which the function $f(x)$ is bounded. For let $\overline{\varepsilon}$ be some fixed number. By hypothesis there exists a $\overline{V^*}(a)$ such that if $x$ and $x_0$ are on $\overline{V^*}(a)$, then \[ |f(x)-f(x_0)|< \overline{\varepsilon}. \] Taking $x_0$ as a fixed number, we have that \[ f(x_0)-\overline{\varepsilon} < f(x) < f(x_0) + \overline{\varepsilon} \] for every $x$ on $\overline{V^*}(a)$. Hence there is at least one \textit{finite} value, $b$, approached by $f(x)$. Now for every $\varepsilon>0$ there exists a $V_\varepsilon^*(a)$ such that if $x_1$ and $x_2$ are any two \correction{values}{valves} of $x$ in $V_\varepsilon^*(a)$, $|f(x_1)-f(x_2)|< \varepsilon$. Hence by the definition of value approached there is an $x_\varepsilon$ of $V_\varepsilon^*(a)$ for which \begin{align*} |f(x_\varepsilon)-b|&< \varepsilon\tag{\textit{a}}\\ \intertext{and} |f(x_\varepsilon)-f(x)|&< \varepsilon\tag{\textit{b}} \end{align*} for every $x$ of $V_\varepsilon^*(a)$. Hence, combining (\textit{a}) and (\textit{b}), for every $x$ of $V_\varepsilon^*(a)$ we have \[ |f(x)-b|< 2\varepsilon, \] and hence by the preceding theorem we have \[ \mathop{L}_{x \doteq a} f(x)=b.\qedhere \] \end{proof} In case $a$ as well as $b$ is finite, Theorem~\hyperlink{thm27}{27} becomes: \textit{A necessary and sufficient condition that \[ \mathop{L}_{x\doteq a}f(x) \] shall exist and be finite is that for every $\varepsilon>0$ there exists a $\delta_\varepsilon > 0$ such that \[ |f(x_1)-f(x_2)|<\varepsilon \] %-----File: 080.png---Folio 68------- for every $x_1$ and $x_2$ such that \[ x_1 \neq a,\quad x_2\neq a,\quad |x_1-a|< \delta_\varepsilon,\quad |x_2-a|< \delta_\varepsilon. \]} In case $a$ is $+\infty$ the condition becomes: \textit{For every $\varepsilon >0$ there exists a $N_\varepsilon>0$ such that} \[ |f(x_1)-f(x_2)|<\varepsilon \] \textit{for every $x_1$ and $x_2$ such that $x_1>N_\varepsilon$, $x_2>N_\varepsilon$.} The necessary and sufficient conditions just derived have the following evident corollaries: \begin{ncorollary}[1]\hypertarget{cor1th27}{} The expression \[ \mathop{L}_{x \doteq a}f(x)=b, \] where $b$ is finite, is equivalent to the expression \[ \mathop{L}_{x \doteq a}(f(x)-b)=0, \] and whether $b$ is finite or infinite \[ \mathop{L}_{x \doteq a} f(x) =b \text{ is equivalent to } \mathop{L}_{x \doteq a} (-f(x)) =-b. \] \end{ncorollary} \begin{ncorollary}[2]\hypertarget{cor2th27} The expressions \[ \mathop{L}_{x \doteq a} f(x) = 0 \text{ and } \mathop{L}_{x \doteq a} |f(x)|= 0 \] are equivalent. \end{ncorollary} \begin{ncorollary}[3] The expression \[ \mathop{L}_{x \doteq a} f(x)=b \] is equivalent to \[ \mathop{L}_{y \doteq 0} f(y+a)=b, \] where $y+a=x$. \end{ncorollary} %-----File: 081.png---Folio 69------- \begin{ncorollary}[4] The expression \[ \mathop{L}_{\stackrel{x < a}{x \doteq a}} f(x)=b \] is equivalent to \[ \mathop{L}_{z \doteq + \infty} f \left({a + \frac1z}\right) = b, \] where $z = \frac{1}{x-a}$. \end{ncorollary} The reader should verify these corollaries by writing down the necessary and sufficient condition for the existence of each limit. The following less obvious statement is proved in detail for the case when $b$ is finite, the case when $b$ is $+ \infty$ or $-\infty$ being left to the reader. \begin{ncorollary}[5] If \[ \mathop{L}_{x \doteq a} f(x) = b, \] then \[ \mathop{L}_{x \doteq a} |f(x)| = |b|. \] \end{ncorollary} \begin{proof} By the necessary condition of Theorem~\hyperlink{thm26}{26} for every $\varepsilon$ there exists a $V_{\varepsilon}^*(a)$ such that for every $x_1$ of $V_{\varepsilon}^*(a)$ \[ |f(x_1)-b|< \varepsilon. \] If $f(x_1)$ and $b$ are of the same sign, then \[ \bigl||f(x_1)|-|b|\bigr| = |f(x_1)-b|< \varepsilon, \] and if $f(x_1)$ and $b$ are of opposite sign, then \[ \bigl||f(x_1)|-|b|\bigr| < |f(x_1)-b|< \varepsilon. \] Hence, by the sufficient condition of Theorem~\hyperlink{thm26}{26}, \[ \mathop{L}_{x \doteq a} |f(x)| \] exists and is equal to $|b|$. \end{proof} %-----File: 082.png---Folio 70------- \begin{ncorollary}[6] If a function $f(x)$ is continuous at $x=a$, then $|f(x)|$ is continuous at $x=a$. \end{ncorollary} It should be noticed that \begin{align*} \mathop{L}_{x \doteq a} |f(x)|&= |b|\\ \intertext{is \textit{not equivalent} to} \mathop{L}_{x \doteq a} f(x)&=b. \end{align*} Suppose $f(x) = +1$ for all rational values of $x$ and $f(x) =-1$ for all irrational values of $x$. Then $\displaystyle\mathop{L}_{x \doteq a} |f(x)|= +1$, but $\displaystyle\mathop{L}_{x \doteq a} f(x)$ does not exist, since both $+1$ and $-1$ are values approached by $f(x)$ as $x$ approaches any value whatever. \begin{definition}\index{Numbers!sequence of}\index{Sequence of numbers} Any set of numbers which may be written $[x_n]$, where \begin{align*} n &= 0, 1, 2, \ldots, \kappa, \\ \text{or } \qquad n &= 0, 1, 2, \ldots, \kappa, \ldots, \end{align*} is called a \textit{sequence}. \end{definition} To the corollaries of this section may be added a corollary related to the definition of a limit. \begin{ncorollary}[7] If for every sequence of numbers $[x_n]$ having $a$ as a limit point, \[ \mathop{L}_{\substack{x|[x_n] \\ x \doteq a}} f(x)=b, \quad\text{then}\quad \mathop{L}_{x \doteq a} f(x)=b. \] \end{ncorollary} \begin{proof} In case two values $b$ and $b_1$ were approached by $f(x)$ as $x$ approaches $a$, then, as in the first part of the proof of Theorem~\hyperlink{thm26}{26}, two sequences could be chosen upon one of which $f(x)$ approached $b$ and upon the other of which $f(x)$ approached $b_1$. \end{proof} \section{Application to Infinite Series.}\hypertarget{chIVsec3}{}%[3] \index{Convergence!of infinite series}\index{Infinite series}\index{Series!infinite} The theory of limits has important applications to infinite series. An \textit{infinite series} is defined as an expression of the form %-----File: 083.png---Folio 71------- \[ \sum_{k=1}^\infty a_k = a_1 + a_2 + a_3 + \ldots + a_n + \ldots. \] If $S_n$ is defined as \[ a_1 + \ldots + a_n = \sum_{k=1}^n a_k, \] $n$ being any positive integer, then the sum of the series is defined\label{dp71} as \[ \mathop{L}_{n=\infty} S_n = S \] if this limit exists. If the limit exists and is finite, the series is said to be \index{Infinite series!convergence and divergence of}\index{Series!infinite!convergence and divergence of}\textit{convergent}. If $S$ is infinite or if $S_n$ approaches more than one value as $n$ approaches infinity, then the series is \index{Divergence}\textit{divergent}. For example, $S$ is infinite if \[ \sum_{k=1}^\infty a_k = 1 + 1 + 1 + 1 \ldots, \] and $S_n$ has more than one value approached if \[ \sum_{k=1}^\infty a_k = 1-1 + 1-1 + 1 \ldots. \] It is customary to write \[ R_n=S-S_n. \] A necessary and sufficient condition for the convergence of an infinite series is obtained from Theorem~\hyperlink{thm27}{27}. (1) \textit{For every $\varepsilon > 0$ there exists an integer $N_{\varepsilon}$, such that if $n > N_{\varepsilon}$ and $n' > N_{\varepsilon}$ then} \[ |S_n-S_{n'}|< \varepsilon. \] This condition immediately translates into the following form: %-----File: 084.png---Folio 72------- (2) \textit{For every $\varepsilon>0$ there exists an integer $N_\varepsilon$, such that if $n>N_\varepsilon$, then for every $k$} \[ |a_n + a_{n+1} + \ldots + a_{n+k}|< \varepsilon. \] \begin{corollary}\label{cp72} If $\sum\limits_{k=1}^\infty a_k$ is a convergent series, then $\displaystyle\mathop{L}_{k \doteq \infty} a_k=0$. \end{corollary} \begin{definition}\index{Absolute convergence of infinite series} A series \[ \sum_{k=0}^\infty a_k=a_0+a_1+ \ldots+a_n+ \ldots \] is said to be \textit{absolutely convergent} if \[ |a_0|+ |a_1|+ \ldots + |a_n| + \ldots \] is convergent. \end{definition} Since \[ |a_n + a_{n+1} +\ldots +a_{n+k}| < |a_n|+ |a_{n+1}|+ \ldots|a_{n+k}|, \] the above criteria give \begin{theorem}[28]\hypertarget{thm28}{} A series is convergent if it is absolutely convergent. \end{theorem} \begin{theorem}[29]\hypertarget{thm29}{} If $\sum\limits_{k=0}^\infty b_k$ is a convergent series all of whose terms are positive and $\sum\limits_{k=0}^\infty a_k$ is a series such that for every $k$, $|a_k|\leqq b_k$, then \[ \sum_{k=0}^\infty a_k \] is absolutely convergent. \end{theorem} \begin{proof} By hypothesis \[ \sum_{k=0}^n|a_k|\leqq \sum_{k=0}^n b_k. \] %-----File: 085.png---Folio 73------- Hence \[ \sum_{k=0}^n|a_k| \] is bounded, and being an increasing function of $n$, the series is convergent according to Theorem~\hyperlink{thm25}{25}. \end{proof} This theorem gives a useful method of determining the convergence or divergence of a series, namely, by comparison with a known series. Such a known series is the \index{Geometric series}\index{Series!geometric}geometric series \[ a+ar+ar^2 + \ldots +ar^n+ \ldots, \] where $0 < r < 1$ and $a > 0$. In this series \[ \sum_{k=0}^n ar^k = a\frac{1-r^{n+1}}{1-r} < \frac{a}{1-r}, \] which shows that the series is convergent. Moreover, it can easily be seen to have the sum $\dfrac{a}{1-r}$. If $r \qqge 1$, the geometric series is evidently divergent. This result can be used to prove the ``ratio-test'' for convergence. \begin{theorem}[30]\index{Ratio test for convergence of infinite series} \hypertarget{thm30}{}If there exists a number, $r$, $00$ there exists an integer $N_\varepsilon$, such that if $n>N_\varepsilon$ then $|R_n|< \varepsilon$.} \end{proof} \begin{definition}\label{dp75}A function $f(x)$ such that \[ \mathop{L}_{x \doteq a} f(x)=0 \] is called an \index{Infinitesimals}\textit{infinitesimal} as $x$ approaches $a$.\footnote{% No constant, however small if not zero, is an infinitesimal, the essence of the latter being that it varies so as to approach zero as a limit. Cf.\ Goursat, Cours d'Analyse, tome~I, p.~21, etc.} \end{definition} \begin{theorem}[32]\hypertarget{thm32}{} The sum, difference, or product of two infinitesimals is an infinitesimal. \end{theorem} \begin{proof} Let the two infinitesimals be $f_1(x)$ and $f_2(x)$. For every $\varepsilon$, $1> \varepsilon >0$, there exists a $V_1^*(a)$ for every $x$ of which \[ |f_1(x)|< \frac\varepsilon2, \] and a $V_2^*(a)$ for every $x$ of which \[ |f_2(x)|< \frac\varepsilon2. \] Hence in any $V^*(a)$ common to $V_1^*(a)$ and $V_2^*(a)$ \begin{align*} & |f_1(x) + f_2(x)|\leqq |f_1(x)|+ |f_2(x)|< \varepsilon, \\ & |f_1(x)-f_2(x)|\leqq |f_1(x)|+ |f_2(x)|< \varepsilon, \\ & |f_1(x) \cdot f_2(x)|= |f_1(x)|\cdot|f_2(x)|< \varepsilon. \end{align*} From these inequalities and Theorem~\hyperlink{thm26}{26} the conclusion follows. \end{proof} \begin{theorem}[33]\hypertarget{thm33}{} If $f(x)$ is bounded on a certain $\overline{V^*}(a)$ and $\varepsilon(x)$ is an infinitesimal as $x$ approaches $a$, then $\varepsilon(x)\cdot f(x)$ is also an infinitesimal as $x$ approaches $a$. \end{theorem} %-----File: 088.png---Folio 76------- \begin{proof} By hypothesis there are two numbers $m$ and $M$, such that $M>f(x)>m$ for every $x$ on $\overline{V^*}(a)$. Let $k$ be the larger of $|m|$ and $|M|$. Also by hypothesis there exists for every $\varepsilon$ a ${V_\varepsilon}^*(a)$ within $\overline{V^*}(a)$ such that if $x$ is in ${V_\varepsilon}^*(a)$, then \begin{align*} |\varepsilon(x)|&< \frac{\varepsilon}{k} \\ \intertext{or} k|\varepsilon(x)|&< \varepsilon. \end{align*} But for such values of $x$ \[ |f(x)\cdot\varepsilon(x)| < k\cdot|\varepsilon(x)|< \varepsilon, \] and hence for every $\varepsilon$ there is a ${V_\varepsilon}^*(a)$ such that for $x$ an ${V_\varepsilon}^*(a)$ \[ |f(x)\cdot\varepsilon(x)|< \varepsilon.\qedhere \] \end{proof} \begin{corollary} If $f(x)$ is an infinitesimal and $c$ any constant, then $c \cdot f(x)$ is an infinitesimal. \end{corollary} \begin{theorem}[34]\hypertarget{thm34}{} If $\displaystyle \mathop{L}_{x \doteq a} f_1(x)=b_1$ and $\displaystyle \mathop{L}_{x \doteq a} f_2(x)=b_2$, $b_1$ and $b_2$ being finite, then \begin{align*} \tag{$\alpha$} &\mathop{L}_{x\doteq a} \{f_1(x) \pm f_2(x)\} = b_1 \pm b_2, \\ \tag{$\beta$} &\mathop{L}_{x\doteq a} \{f_1(x) \cdot f_2(x)\} = b_1 \cdot b_2; \\ \intertext{and if $b_2\neq 0$,} \tag{$\gamma$} &\mathop{L}_{x\doteq a} \frac{f_1(x)}{f_2(x)} = \frac{b_1}{b_2} \end{align*} \end{theorem} \begin{proof} According to Theorem~\hyperlink{thm31}{31}, we write \begin{align*} f_1(x) &= b_1 + \varepsilon_1(x),\\ f_2(x) &= b_2 + \varepsilon_2(x), \end{align*} %-----File: 089.png---Folio 77------- where $\varepsilon_1(x)$ and $\varepsilon_2(x)$ are infinitesimals. Hence \begin{gather*} \tag{$\alpha'$} f_1(x) + f_2(x) = b_1 + b_2 + \varepsilon_1(x) + \varepsilon_2(x), \\ \tag{$\beta'$} f_1(x)\cdot f_2(x) = b_1 \cdot b_2 + b_1 \cdot \varepsilon_2(x) + b_2 \cdot \varepsilon_1(x) + \varepsilon_1(x) \cdot \varepsilon_2(x). \end{gather*} But by the preceding theorem the terms of $(\alpha')$ and $(\beta')$ which involve $\varepsilon_1(x)$ and $\varepsilon_2(x)$ are infinitesimals, and hence the conclusions $(\alpha)$ and $(\beta)$ are established. To establish ($\gamma$), observe that by Theorem~\hyperlink{thm26}{26} there exists a $V^*(a)$ for every $x$ of which $|f_2(x)-b_2|< |b_2|$ and hence upon which $f_2(x)\neq 0$. Hence \[ \frac{f_1(x)}{f_2(x)} = \frac{b_1 + \varepsilon_1(x)}{b_2 + \varepsilon_2(x)} = \frac{b_1}{b_2} + \frac{b_2 \varepsilon_1(x)-b_1 \varepsilon_2(x)} {b_2 \{ b_2 + \varepsilon_2(x) \}}, \] the second term of which is infinitesimal according to Theorems \hyperlink{thm32}{32} and \hyperlink{thm33}{33}. \end{proof} Some of the cases in which $b_1$ and $b_2$ are $\pm\infty$ are covered by the following theorems. The other cases ($\infty-\infty$, $\dfrac{\infty}{\infty}$, $\dfrac{0}{0}$, etc.), are treated in Chapter~\hyperlink{chapVI}{VI}. \begin{theorem}[35]\hypertarget{thm35}{} If $f_2(x)$ has a lower bound on some $V^*(a)$, and if \[ \mathop{L}_{x \doteq 0} f_1(x) = +\infty, \] then \[ \mathop{L}_{x \doteq 0} \{f_2(x) + f_1(x)\} = +\infty. \] \end{theorem} \begin{proof} Let $M$ be the lower bound of $f_2(x)$. By hypothesis, for every number $E$ there exists a $V_E^*(a)$ such that for $x$ on $V_E^*(a)$ \[ f_1(x) > E-M. \] Since \[ f_2(x) > M,\\ \] this gives \[ f_1(x) + f_2(x) > E, \] which means that $ f_1(x) + f_2(x)$ approaches the limit $+\infty$. \end{proof} %-----File: 090.png---Folio 78------- \begin{theorem}[36]\hypertarget{thm36}{} If $\displaystyle \mathop{L}_{x \doteq a} f_1(x) = + \infty$ or $-\infty$, and if $f_2(x)$ is such that for a $\overline{V^*}(a)$\correction{,}{} $f_2(x)$ has a lower bound greater than zero or an upper bound less than zero, then $\displaystyle \mathop{L}_{x \doteq a} \{ f_1(x) \cdot f_2(x)\}$ is definitely infinite; i.e., if $f_2(x)$ has a lower bound greater than zero and $\displaystyle \mathop{L}_{x \doteq a} f_1(x) = +\infty$, then $\displaystyle\mathop{L}_{x\doteq a}\{f_1(x)\cdot f_2(x)\} = +\infty$, etc. \end{theorem} \begin{proof} Suppose $f_2(x)$ has a lower bound greater than zero, say $M$, and that $\displaystyle \mathop{L}_{x \doteq a} f_1(x) = +\infty$. Then for every $E$ there exists a $V_E^*(a)$ within $\overline{V^*}(a)$ such that for every $x_1$ of $V_E^*(a)$, $f_1(x_1) > \dfrac{E}{M}$, and therefore $f_1(x_1)\cdot f_2(x_1)\qqge f_1(x_1)\cdot M>E$. Hence by the definition of limit of a function $\displaystyle\mathop{L}_{x\doteq a}\{f_1(x)\cdot f_2(x)\} = +\infty$. If we consider the case where $f_2(x)$ has an upper bound less than zero, we have in the same manner $L \{f_1(x)\cdot f_2(x)\} =-\infty$. Similar statements hold for the cases in which $\displaystyle \mathop{L}_{x \doteq a} f_1(x) =-\infty$. \end{proof} \begin{corollary} If $f_2(x)$ is positive and has a finite upper bound and $\displaystyle \mathop{L}_{x \doteq a} f_1(x) = +\infty$, then \[ \mathop{L}_{x \doteq a} \frac{f_1(x)}{f_2(x)} = +\infty. \] \end{corollary} \begin{theorem}[37]\hypertarget{thm37}{} If $\displaystyle \mathop{L}_{x \doteq a} f(x)= +\infty$, then $\displaystyle \mathop{L}_{x \doteq a} \frac{1}{f(x)} = 0$, and there is a vicinity $V^*(a)$ upon which $f(x)>0$. Conversely, if $\displaystyle \mathop{L}_{x \doteq a} f(x) =0$ and there is a $V^*(a)$ upon which $f(x) > 0$, then $\displaystyle \mathop{L}_{x \doteq a} \frac{1}{f(x)} = +\infty$. \end{theorem} \begin{proof} If $\displaystyle \mathop{L}_{x \doteq a} f(x) = +\infty$, then for every $\varepsilon$ there exists a ${V_\varepsilon}^*(a)$ such that if $x$ is in ${V_\varepsilon}^*(a)$, then \[ f(x) > \frac{1}{\varepsilon} \] %-----File: 091.png---Folio 79------- and \[ \frac{1}{f(x)} < \varepsilon. \] \[ \therefore \mathop{L}_{x \doteq a} \frac{1}{f(x)} = 0, \] since both $f(x)$ and $\dfrac{1}{f(x)}$ are positive. Again, if $\displaystyle \mathop{L}_{x \doteq a} f(x) =0$, then for every $\varepsilon$ there is a $\overline{V_\varepsilon^*}(a)$ such that for $x$ in $\overline{V_\varepsilon^*}(a)$, $|f(x)|<\varepsilon$ or $\dfrac{1}{f(x)}>\dfrac{1}{\varepsilon}$ ($f(x)$ being positive). Hence \[ \mathop{L}_{x \doteq a} \frac{1}{f(x)} = + \infty.\qedhere \] \end{proof} \begin{ncorollary}[1] If $f_1(x)$ has finite upper and lower bounds on some $V^*(a)$ and $\displaystyle \mathop{L}_{x \doteq a} f_2(x) = +\infty$ or $-\infty$, then \[ \mathop{L}_{x \doteq a} \frac{f_1(x)}{f_2(x)} = 0. \] \end{ncorollary} \begin{ncorollary}[2] If $f_2(x)$ is positive and $f_1(x)$ has a positive lower bound on some $V^*(a)$ and $\displaystyle \mathop{L}_{x \doteq a} f_2(x)=0$, then \[ \mathop{L}_{x \doteq a} \frac{f_1(x)}{f_2(x)} = +\infty. \] \end{ncorollary} \begin{theorem}[38]\index{Change of variable}\emph{(change of variable).}\hypertarget{thm38}{} If \begin{enumerate} \item[\textnormal{(1)}] $\displaystyle\mathop{L}_{x \doteq a } f_1(x) = b_1$ and $\displaystyle\mathop{L}_{x \doteq b_1} f_2(y) = b_2$ when $y$ takes all valves of $f_1(x)$ corresponding to values of $x$ on some $\overline{V^*}(a)$, and if \item[\textnormal{(2)}]\hypertarget{item2p79} $\displaystyle f_1(x) \neq b_1 \text{ for } x \text{ on } \overline{V^*}(a)$, \end{enumerate} then \[ \mathop{L}_{x \doteq a} f_2(f_1(x)) = b_2. \] \end{theorem} %-----File: 092.png---Folio 80------- \begin{proof} ($\alpha$) Since $\displaystyle \mathop{L}_{y \doteq b_1} f_2(y) =b_2$, for every $V(b_2)$ there exists a $V^*(b_1)$ such that if $y$ is in $V^*(b_1)$, $f_2(y)$ is in $V(b_2)$. Since $\displaystyle \mathop{L}_{x \doteq a}f_1(x) =b_1$, for every $V(b_1)$ there exists a $V^*(a)$ in $\overline{V^*}(a)$ such that if $x$ is in $V^*(a)$, $f_1(x)$ is in $V(b_1)$. But by \hyperlink{item2p79}{(2)} if $x$ is in $V^*(a)$, $f_1(x)\neq b$. Hence ($\beta$) for every $V^*(b_1)$ there exists a $V^*(a)$ such that for every $x$ in $V^*(a)$, $f_1(x)$ is in $V^*(b_1)$. Combining statements ($\alpha$) and ($\beta$): for every $V(b_2)$ there exists a $V^*(a)$ such that for every $x$ in $V^*(a)$ $f_1(x)$ is in $V^*(b_1)$, and hence $f_2(f(x))$ is in $V(b_2)$. This means, according to Theorem~\hyperlink{thm26}{26}, that \[ \mathop{L}_{x \doteq a} f_2(f_1(x)) = b_2.\qedhere \] \end{proof} \begin{theorem}[39]\hypertarget{thm39}{} If $\displaystyle \mathop{L}_{x \doteq a} f_1(x) =b$ and $\displaystyle \mathop{L}_{y \doteq b} f_2(y) =f_2(b)$, where $y$ takes all values taken by $f_1(x)$ for $x$ on some $\overline{V^*}(a)$, then \[ \mathop{L}_{x \doteq a} f_2(f_1(x)) = f_2(b). \] \end{theorem} \begin{proof} The proof of the theorem is similar to that of Theorem~\hyperlink{thm38}{38}. In this case the notation $f_2(b)$ implies that $b$ is a finite number. Thus for every $\varepsilon_1$ there exists a ${V_{\varepsilon_1}}^*(a)$ entirely within $\overline{V^*}(a)$ such that if $x$ is in ${V_{\varepsilon_1}}^*(a)$, \[ |f_1(x)-b|< \varepsilon_1. \] Furthermore, for every $\varepsilon_2$ there exists a $\delta_{\varepsilon_2}$ such that for every $y$, $y \neq b$, $|y-b|<\delta_{\varepsilon_2}$, \[ |f_2(y)-f_2(b)|< \varepsilon_2. \] But since $|f_2(y)-f_2(b)|= 0$ when $y = b$, this means that for all values of $y$ (equal or unequal to $b$) such that $|y-b|< \delta_{\varepsilon_2}$, $|f_2(y)-f_2(b)|< \varepsilon_2$. Now let $\varepsilon_1 = \delta_{\varepsilon_2}$; then, if $x$ is in ${V_{\varepsilon_1}}^*(a)$, it follows that $|f_1(x)-b|< \delta_{\varepsilon_2}$ and therefore that \[ |f_2(f_1(x))-f_2(b)|< \varepsilon_2. \] Hence \[ \mathop{L}_{x \doteq a} f_2(f_1(x)) = f_2(b).\qedhere \] \end{proof} %-----File: 093.png---Folio 81------- \begin{ncorollary}[1]\hypertarget{cor1p81}{} If $f_1(x)$ is continuous at $x=a$, and $f_2(y)$ is continuous at $y=f_1(a)$, then $f_2(f_1(x))$ is continuous at $x=a$. \end{ncorollary} \begin{ncorollary}[2]\hypertarget{cor2p81}{} If $k \neq 0$, $f(x) \geqq 0$, and $\displaystyle \mathop{L}_{x \doteq a} f(x) =b$, then \[ \mathop{L}_{x \doteq a} (f(x))^k = b^k, \] under the convention that $\infty^k = \infty$ if $k>0$ and $\infty^k=0$ if $k<0$. \end{ncorollary} \begin{ncorollary}[3]\label{logp81} If $c>0$ and $f(x)>0$ and $b>0$ and $\displaystyle \mathop{L}_{x \doteq a} f(x)=b$, then \[ \mathop{L}_{x \doteq a} \log_c f(x) = \log_c b, \] under the convention that $\log_c (+\infty) = +\infty$ and $\log_c 0 =-\infty$. \end{ncorollary} The conclusions of the last two corollaries may also be expressed by the equations \[ \mathop{L}_{x \doteq a} (f(x))^k = (\mathop{L}_{x \doteq a} f(x))^k \] and \[ \log_c \mathop{L}_{x \doteq a} f(x) = \mathop{L}_{x \doteq a} \log_c f(x). \] \begin{ncorollary}[4] If $\displaystyle \mathop{L}_{x \doteq a} (f(x))^k$ or $\displaystyle \mathop{L}_{x \doteq a} \log f(x)$ fails to exist, then $\displaystyle \mathop{L}_{x \doteq a} f(x)$ does not exist. \end{ncorollary} \section{Further Theorems on Limits.}\hypertarget{chIVsec5}{}%[5] \begin{theorem}[40]\hypertarget{thm40}{} If $f(x) \leqq b$ for all values of a set $[x]$ on a certain $V^*(a)$, then every value approached by $f(x)$ as $x$ approaches $a$ is less than or equal to $b$. Similarly if $f(x) \geqq b$ for all values of a set $[x]$ on a certain $V^*(a)$, then every value approached by $f(x)$ as $x$ approaches $a$ is greater than or equal to $b$. \end{theorem} \begin{proof} If $f(x) \leqq b$ on $V^*(a)$, then if $b'$ is any value greater than $b$, and $V(b')$ any vicinity of $b'$ which does not include $b$, there is no value of $x$ on $V^*(a)$ for which $f(x)$ is in $V(b')$. Hence $b'$ is not a value approached. A similar argument holds for the case where $f(x) \geqq b$. \end{proof} %-----File: 094.png---Folio 82------- \begin{ncorollary}[1]\hypertarget{cor1p82}{} If $f(x)\geqq 0$ in the neighborhood of $x=a$, then if \[ \mathop{L}_{x\doteq a} f(x)\text{ exist, } \mathop{L}_{x\doteq a} f(x) \geqq 0. \] \end{ncorollary} \begin{ncorollary}[2]\hypertarget{cor2p82}{} If $f_1(x)\geqq f_2(x)$ in the neighborhood of $x=a$, then \[ \mathop{L}_{x\doteq a} f_1(x) \geqq \mathop{L}_{x\doteq a} f_2(x) \] if both these limits exist. \end{ncorollary} \begin{proof} Apply Corollary~\hyperlink{cor1p82}{1} to $f_1(x)-f_2(x)$. \end{proof} \begin{ncorollary}[3]\hypertarget{cor3p82}{} If $f_1(x)\geqq f_2(x)$ in the neighborhood of $x=a$, then the largest value approached by $f_1(x)$ is greater than or equal to the largest value approached by $f_2(x)$. \end{ncorollary} \begin{ncorollary}[4]\hypertarget{cor4p82}{} If $f_1(x)$ and $f_2(x)$ are both positive in the neighborhood of $x=a$, and if $f_1(x)\geqq f_2(x)$, then if $\displaystyle\mathop{L}_{x\doteq a} f_1(x)=0$, it follows that \[ \mathop{L}_{x\doteq a} f_2(x)=0. \] \end{ncorollary} \begin{theorem}[41]\hypertarget{thm41}{} If $[x']$ is a subset of $[x]$, $a$ being a limit point of $[x']$, and if $\displaystyle\mathop{L}_{x\doteq a} f(x)$ exists, then $\displaystyle\mathop{L}_{\text{\correction{$x'$}{$x$}}\doteq a} f(x')$ exists and \[ \mathop{L}_{x\doteq a} f(x)= \mathop{L}_{x'\doteq a} f(x').% \footnote{The notation $f(x')$ is used to indicate that $x$ takes the values of the set $[x']$.} \] \end{theorem} \begin{proof} By hypothesis there exists for every $V(b)$ a $V^*(a)$ such that for every $x$ of the set $[x]$ which is in $V^*(a)$, $f(x)$ is in $V(b)$. Since $[x']$ is a subset of $[x]$, the same $V^*(a)$ is evidently efficient for $x$ on $[x']$. \end{proof} In the statement of necessary and sufficient conditions for the existence of a limit we have made use of a certain positive multiple-valued function of $\varepsilon$ denoted by $\delta_\varepsilon$. If a given value is effective as a $\delta_\varepsilon$, then every positive value smaller than this is also effective. \begin{theorem}[42]\hypertarget{thm42}{} For every $\varepsilon$ for which the set of values of $\delta_\varepsilon$ has an upper bound there is a greatest $\delta_\varepsilon$. \end{theorem} %-----File: 095.png---Folio 83------- \begin{proof} Let $\overline{B}[\delta_\varepsilon]$ be the least upper bound of the set of values of $\delta_\varepsilon$, for a particular $\varepsilon$. If $x$ is such that $|x-a|< \overline{B}[\delta_\varepsilon]$, then there is a $\delta_\varepsilon$ such that $|x-a|< \delta_\varepsilon$. But if $|x-a|< \delta_\varepsilon$, $|f(x)-b|< \varepsilon$. Hence, if $|x-a|< \overline{B}[\delta_\varepsilon]$, $|f(x)-b|< \varepsilon$. \end{proof} \begin{theorem}[43]\hypertarget{thm43}{} The limit of the least upper bound of a function $f(x)$ on a variable segment $\overline{a\ x}$, $a < x$, as the end point approaches $a$, is the least upper bound of the values approached by the function as $x$ approaches $a$ from the right. \end{theorem} \begin{proof} Let $l$ be the least upper bound of the values approached by the function as $x$ approaches $a$ from the right, and let $b(x)$ represent the upper bound of $f(x)$ for all values of $x$ on $\overline{a\ x}$. Since $\overline{B}f(x)$ on the segment $\overline{a\ x_1}$ is not greater than $\overline{B}f(x)$ on a segment $\overline{a\ x_2}$ if $x_1$ lies on $\overline{a\ x_2}$, $b(x)$ is a non-oscillating function decreasing as $x$ decreases. Hence $\displaystyle \mathop{L}_{x \doteq a} b(x)$ exists by Theorem~\hyperlink{thm21}{21}; and by Corollary~\hyperlink{cor3p82}{3}, Theorem~\hyperlink{thm40}{40}, $\displaystyle \mathop{L}_{x \doteq a} b(x) \geqq l$. If $\displaystyle \mathop{L}_{x \doteq a} b(x) = k > l$, then there are two vicinities of $k$, $V_1(k)$ contained in $V_2(k)$ and $V_2(k)$ not containing $l$. By Theorem~\hyperlink{thm26}{26} a $V_1^*(a)$ exists such that if $x$ is in $V_1^*(a)$, $b(x)$ is in $V_1(k)$. Furthermore, by the definition of $b(x)$, if $x_1$ is an arbitrary value of $x$ on $V_1^*(a)$, then there is a value of $x$ in $\overline{a\ x_1}$ such that $f(x)$ is in $V(k)$. Hence $k$ would be a value approached by $f(x)$ contrary to the hypothesis $k>l$. \end{proof} \section{Bounds of Indetermination. Oscillation.}\hypertarget{chIVsec6}{}%[6] It is a corollary of Theorem~\hyperlink{thm43}{43} that in the approach to any point $a$ from the right or from the left the least upper \correction{bounds}{bound} and the greatest lower bounds of the values approached by $f(x)$ are themselves values approached by $f(x)$. The four numbers thus indicated may be denoted by \[\label{limp84} \overline{f(a+0)} = \mathop{\overline{L}}_{x \doteq a+0} f(x) = \stackrel{\leftarrow}{\mathop{L}_{x \doteq a}} f(x), \] %-----File: 096.png---Folio 84------- the least upper bound of the values approached from the right: \[ \overline{f(a-0)} = \mathop{\overline{L}}_{x \doteq a-0} f(x) = \mathop{\stackrel{\rightarrow}{L}}_{x \doteq a} f(x), \] the least upper bound of the values approached from the left: \[ \underline{f(a+0)} = \mathop{\underline{L}}_{x \doteq a + 0} f(x) = \mathop{L}_{\stackrel{\leftarrow}{x \doteq a}} f(x), \] the greatest lower bound of the values approached from the right: \[ \underline{f(a-0)} = \mathop{\underline{L}}_{x \doteq a-0} f(x) = \mathop{L}_{\stackrel{\rightarrow}{x \doteq a}} f(x), \] the greatest lower bound of the values approached from the left. If all four of these values coincide, there is only one value approached and $\displaystyle \mathop{L}_{x \doteq a} f(x)$ exists. If $\overline{f(a+0)}$ and $\underline{f(a+0)}$ coincide, this value is denoted by $f(a+0)$ and is the same as $\displaystyle \mathop{L}_{\stackrel{x>a}{x \doteq a}} f(x)$. Similarly if $\overline{f(a-0)}$ and $\underline{f(a-0)}$ coincide, their common value, $\displaystyle \mathop{L}_{\stackrel{xc_2 \text{ and } f(x'')c_2$ and $f(x'')c_2$, or no $x''$ such that $f(x'')c_2$ and $f(x'')b_2$ is the least upper bound of the values approached; $a_2$ may then be so chosen that $b_2 < a_2 < B$, so that by hypothesis for $x$ on $V^*(a)$ $B$ cannot be a value approached. Again, suppose $BB$. Therefore $B$ cannot be the least upper bound. Since the least upper bound may not be either less than $b_2$ or greater than $b_2$, it must be equal to $b_2$. A similar argument will prove $b_1$ to be the greatest lower bound of the values approached. \end{proof} %-----File: 099.png---Folio 87------- \chapter{CONTINUOUS FUNCTIONS.}\hypertarget{chapV}{}%[V] \section{Continuity at a Point.}\hypertarget{chVsec1}{}%[1] The notion of continuous functions will in this chapter, as in the definition on page~\pageref{dp61}, be confined to single-valued functions. It has been shown in Theorem~\hyperlink{thm34}{34} that if $f_1(x)$ and $f_2(x)$ are continuous at a point $x=a$, then \[ f_1(x) \pm f_2(x), \quad f_1(x) \cdot f_2(x), \quad f_1(x)/f_2(x), \quad (f_2(x) \neq 0) \] are also continuous at this point. Corollary~\hyperlink{cor1p81}{1} of Theorem~\hyperlink{thm39}{39} states that a continuous function of a continuous function is continuous. The definition of continuity at $x=a$, namely, \[ \mathop{L}_{x \doteq a} f(x) = f(a), \] is by Theorem~\hyperlink{thm26}{26} equivalent to the following proposition: \emph{For every $\varepsilon>0$ there exists a $\delta_\varepsilon>0$ such that if $|x-a|< \delta_\varepsilon$, then $|f(x)-f(a)|< \varepsilon$.} It should be noted that the restriction $x \neq a$ which appears in the general form of Theorem~\hyperlink{thm26}{26} is of no significance here, since for $x=a$, $|f(x)-f(a)|= 0 < \varepsilon$. In other words, we may deal with vicinities of the type $V(a)$ instead of $V^*(a)$. The difference of the least upper and the greatest lower bound of a function on an interval $\interval{a}{b}$ has been called in Chapter~\hyperlink{chapIV}{IV}, page~\pageref{chIVp85}, the oscillation of $f(x)$ on that interval, and denoted by $O_a^b(x)$. The definition of continuity and Theorem~\hyperlink{thm27}{27}, Chapter~\hyperlink{chapIII}{III}, give the following necessary and sufficient condition for the continuity of a function $f(x)$ at the %-----File: 100.png---Folio 88------- \textit{For every $\varepsilon>0$ there exists a $\delta_\varepsilon>0$ such that if $|x_1-a|< \delta_\varepsilon$, and $|x_2-a|< \delta_\varepsilon$ then $|f(x_1)-f(x_2)|< \dfrac{\varepsilon}{2}$. This means that for all values of $x_1$ and $x_2$ on the segment $\overline{(a-\delta_\varepsilon)\ (a + \delta_\varepsilon)}$} \[ \overline{B} |f(x_1)-f(x_2)|\leqq \frac\varepsilon2 < \varepsilon, \] and this means \[ \overline{B}f(x)-\underline{B}f(x) < \varepsilon, \] or \[ O^{a + \delta_\varepsilon}_{a-\delta_\varepsilon} f(x) < \varepsilon. \] Then we have \begin{theorem}[45]\hypertarget{thm45}{} If $f(x)$ is continuous for $x=a$, then for every $\varepsilon>0$ there exists a $V_\varepsilon(a)$ such that on $V_\varepsilon(a)$ the oscillation of $f(x)$ is less than $\varepsilon$. \end{theorem} \begin{theorem}[46]\hypertarget{thm46}{} If $f(x)$ is continuous at a point $x=a$ and if $f(a)$ is positive, then there is a neighborhood of $x=a$ upon which the function is positive. \end{theorem} \begin{proof} If there were values of $x$, $[x']$ within every neighborhood of $x=a$ for which the function is equal to or less than zero, then by Theorem~\hyperlink{thm24}{24} there would be a value approached by $f(x')$ as $x'$ approaches $a$ on the set $[x']$. That is, by Theorem~\hyperlink{thm40}{40}, there would be a negative or zero value approached by $f(x)$, which would contradict the hypothesis. \end{proof} \section{Continuity of a Function on an Interval.}\hypertarget{chVsec2}{}%[2] \begin{definition}\index{Continuity!over an interval}\index{Function!continuity of!over an interval} A function is said to be continuous on an interval $\interval{a}{b}$ if it is continuous at every point on the interval. \end{definition} \begin{theorem}[47]\hypertarget{thm47}{} If $f(x)$ is continuous on a finite interval $\interval{a}{b}$, then for every $\varepsilon > 0$, $\interval{a}{b}$ can be divided into a finite number of equal intervals upon each of which the oscillation of $f(x)$ is less than $\varepsilon$.\footnote{% The importance of this theorem in proving the properties of continuous functions seems first to have been recognized by \textsc{Goursat}. See his \textit{Cours d'Analyse}, Vol.~1, page~161.} \end{theorem} %-----File: 101.png---Folio 89------- \begin{proof} By Theorem~\hyperlink{thm45}{45} there is about every point of $\interval{a}{b}$ a segment $\sigma$ upon which the oscillation is less than $\varepsilon$. This set of segments $[\sigma]$ covers $\interval{a}{b}$, and by Theorem~\hyperlink{thm11}{11} $\interval{a}{b}$ can be divided into a finite number of equal intervals each of which is interior to a $\sigma$; this gives the conclusion of our theorem. \end{proof} \begin{theorem}[48]\hypertarget{thm48}{} (Uniform continuity.)\label{t48p89}\index{Uniform continuity}\index{Continuity!uniform}\index{Function!uniform continuity of} If a function is continuous on a finite interval $\interval{a}{b}$, then for every $\varepsilon>0$ there exists a $\delta_{\varepsilon}>0$ such that for any two values of $x$, $x_1$, and $x_2$, on $\interval{a}{b}$ where $|x_1-x_2|< \delta_{\varepsilon}$, $|f(x_1)-f(x_2)|< \varepsilon$. \end{theorem} \begin{proof} This theorem may be inferred in an obvious way from the preceding theorem, or it may be proved directly as follows: By Theorem~\hyperlink{thm27}{27}, for every $\varepsilon$ there exists a neighborhood $V_{\varepsilon}(x')$ of every $x'$ of $\interval{a}{b}$ such that if $x_1$ and $x_2$ are on \correction{$V_\varepsilon(x')$}{$V (x')$}, then $|f(x_1)-f(x_2)|< \varepsilon$. The $V_{\varepsilon}(x)$'s constitute a set of segments which cover $\interval{a}{b}$. Hence, by Theorem~\hyperlink{thm12}{12}, there is a $\delta_{\varepsilon}$ such that if $|x_1-x_2|\text{\correction{$<$}{$>$}} \delta_{\varepsilon}$, $x_1$ and $x_2$ are on the same \correction{$V_\varepsilon(x')$}{$V (x')$} and consequently $|f(x_1)-f(x_2)|< \varepsilon$. \end{proof} The uniform continuity theorem is due to \textsc{E.~Heine}.\footnote{% \textsc{E.~Heine:} \textit{Die Elemente der Functionenlehre}, Crelle, Vol.~74 (1872), p.~188.} The proof given by him is essentially that given above. In 1873 \textsc{L\"uroth}\footnote{% \textsc{L\"uroth:} \textit{Bemerkung \"uber Gleichm\"assige Stetigkeit}, Mathematische Annalen, Vol.~6, p.~319.} gave another proof of the theorem which is based on the following definition of continuity: A single-valued function is continuous at a point $x=a'$ if for every positive $\varepsilon$ there exists a $\delta_{\varepsilon}$, such that for every $x_1$ and $x_2$ on the interval $\interval{a-\delta_{\varepsilon}}{a + \delta_{\varepsilon}}$, $|f(x_1)-f(x_2)|< \varepsilon$ (Theorem~\hyperlink{thm45}{45}). By Theorem~\hyperlink{thm42}{42} there exists a greatest $\delta$ for a given point and for a given $\varepsilon$. Denote this by $\Delta_{\varepsilon}(x)$. If the function is continuous at every point of $\interval{a}{b}$, then for every $\varepsilon$ there will be a value of $\Delta_{\varepsilon}(x)$ for every point of the interval, i.e., $\Delta_{\varepsilon}(x)$, for any particular $\varepsilon$, will be a single-valued function of $x$. %-----File: 102.png---Folio 90------- The essential part of \textsc{L\"uroth's} proof consists in establishing the following fact: If $f(x)$ is continuous at every point of its interval, then for any particular value of $\varepsilon$ the function $\Delta_\varepsilon(x)$ is also a continuous function of $x$. From this it follows by Theorem~\hyperlink{thm50}{50} that the function $\Delta_\varepsilon(x)$ will actually reach its greatest lower bound, that is, will have a minimum value; and this minimum value, like all other values of $\delta_\varepsilon$, will be positive.\footnote{% It is interesting to note that this proof will not hold if the condition of Theorem~\hyperlink{thm26}{26} is used as a definition of continuity. On this point see \textsc{N.~J. Lennes}: The Annals of Mathematics, second series, Vol.~6, p.~86.} This minimum value of \correction{$\Delta_\varepsilon(x)$}{$\Delta$} on the interval under consideration will be effective as a $\delta_\varepsilon$, independent of $x$. The property of a continuous function exhibited above is called uniform continuity, and Theorem~\hyperlink{thm48}{48} may be briefly stated in the form: \emph{Every function continuous on an interval is uniformly continuous on that interval.}\footnote{% It should be noticed that this theorem does not hold if ``segment'' is substituted for ``interval,'' as is shown by the function $\dfrac1x$ on the segment $\overline{0\ 1}$, which is continuous but not uniformly continuous. The function is defined and continuous for every value of $x$ on this \textit{segment}, but not for every value of $x$ on the \emph{interval} $\interval{0}{1}$.} This theorem is used, for example, in proving the integrability of continuous functions. See page~\pageref{t98p157}. \begin{theorem}[49]\hypertarget{thm49}{} If a function is continuous on an interval $\interval{a}{b}$, it is bounded on that interval. \end{theorem} \begin{proof} By Theorem~\hyperlink{thm46}{46} the interval $\interval{a}{b}$ can be divided into a finite number of intervals, such that the oscillation on each interval is less than a given positive number $\varepsilon$. If the number of intervals is $n$, then the oscillation on the interval $\interval{a}{b}$ is less than $n\varepsilon$. Since the function is defined at all points of the interval, its value being $f(x_1)$ at some point $x_1$, it follows that every value of $f(x)$ on $\interval{a}{b}$ is less than $f(x_1) +n\varepsilon$ and greater than $f(x_1)-n\varepsilon$; which proves the theorem. \end{proof} \begin{theorem}[50]\hypertarget{thm50}{} If a function $f(x)$ is continuous on an interval %-----File: 103.png---Folio 91------- $\interval{a}{b}$, then the function assumes as values its least upper and its greatest lower bound. \end{theorem} \begin{proof} By the preceding theorem the function is bounded and hence the least upper and greatest lower bounds are finite. By Theorem~\hyperlink{thm19}{19} there is a point $k$ on the interval $\interval{a}{b}$ such that the least upper bound of the function on every neighborhood of $x=k$ is the same as the least upper bound on the interval $\interval{a}{b}$. Denote the least upper bound of $f(x)$ on $\interval{a}{b}$ by $B$. It follows from Theorem~\hyperlink{thm43}{43} that $B$ is a value approached by $f(x)$ as $x$ approaches $k$. But since $\displaystyle \mathop{L}_{x\doteq k} f(x) =f(k)$, the function being continuous at $x=k$, we have that $f(k) = B$. In the same manner we can prove that the function reaches its greatest lower bound. \end{proof} \begin{corollary} If $k$ is a value not assumed by a continuous function on an interval $\interval{a}{b}$, then $f(x)-k$ or $k-f(x)$ is a continuous function of $x$ and assumes its least upper and greatest lower bounds. That is, there is a definite number $\Delta$ which is the least difference between $k$ and the set of values of $f(x)$ on the interval $\interval{a}{b}$. \end{corollary} \begin{theorem}[51]\hypertarget{thm51}{} If a function is continuous on an interval $\interval{a}{b}$, then the function takes on all values between its least upper and its greatest lower bound. \end{theorem} \begin{proof} If there is a value $k$ between these bounds which is not assumed by a continuous function $f(x)$, then by the corollary of the preceding theorem there is a value $\Delta$ such that no values of $f(x)$ are between $k-\Delta$ and $k+\Delta$. With $\varepsilon$ less than $\Delta$ divide the interval $\interval{a}{b}$ into subintervals according to Theorem~\hyperlink{thm47}{47}, such that the oscillation on every interval is less than $\varepsilon$. No interval of this set can contain values of $f(x)$ both greater and less than $k$, and no two consecutive intervals can contain such values. Suppose the values of $f(x)$ on the first interval of this set are all greater than $k$, then the same is %-----File: 104.png---Folio 92------- true of the second interval of the set, and so on. Hence it follows that all values of $f(x)$ on $\interval{a}{b}$ are either greater than or less than $k$, which is contrary to the hypothesis that $k$ lies between the least upper and the greatest lower bounds of the function on $\interval{a}{b}$. Hence the hypothesis that $f(x)$ does not assume the value $k$ is untenable. \end{proof} By the aid of Theorem~\hyperlink{thm51}{51} we are enabled to prove the following: \begin{theorem}[51a]\hypertarget{thm51a}{} If $f_1(x)$ is continuous at every point of an interval $\interval{a'}{b'}$ except at a certain point $a$, and if \[ \mathop{L}_{x \doteq a} f_1(x) = +\infty \text{ \textit{and} } \mathop{L}_{x \doteq a} f_2(x) =-\infty, \] then for every $b$, finite or $+\infty$ or $-\infty$, there exist two sequences of points, $[x_i]$ and $[x'_i]$ ($i=0, 1, 2, \ldots$), each sequence having a as a limit point, such that \[ \mathop{L}_{i \doteq \infty} \{ f_1(x_i) + f_2(x'_i) \} = b. \] \end{theorem} \begin{proof} Let $[x'_i]$ be any sequence whatever on $\interval{a'}{b'}$ having $a$ as a limit point, and let $x_0$ be an arbitrary point of $\interval{a'}{b'}$. Since $f_1(x)$ assumes all values between $f_1(x_0)$ and $+\infty$, and since $\displaystyle\mathop{L}_{x \doteq a} f_2(x) =-\infty$, it follows, in case $b$ is finite, that for every $i$ greater than some fixed value there exists an $x_i$ such that \[ f_1(x_i) + f_2(x'_i) = b. \] In case $b = +\infty$, $x_i$ is chosen so that \[ f_1(x_i) + f_2(x'_i) > i.\qedhere \] \end{proof} \begin{corollary} Whether $f_1(x)$ and $f_2(x)$ are continuous or not, if $\displaystyle\mathop{L}_{x \doteq a} f_1(x) = +\infty$ and $\displaystyle\mathop{L}_{x \doteq a} f_2(x) =-\infty$, there exists a pair of %-----File: 105.png---Folio 93------- sequences $[x_i]$ and $[x_i']$ such that \[ \mathop{L}_{i\doteq\infty} \{f_1(x_i)+f_2(x_i)\} \] is $+\infty$ or $-\infty$. \end{corollary} \begin{theorem}[52]\hypertarget{thm52}{} If $y$ is a function, $f(x)$, of $x$, monotonic and continuous on an interval $\interval{a}{b}$, then $x=f^{-1}(y)$ is a function of $y$ which is monotonic and continuous on the interval $\interval{f(a)}{f(b)}$. \end{theorem} \begin{proof} By Theorem~\hyperlink{thm20}{20} the function $f^{-1}(y)$ is monotonic and has as upper and lower bounds $a$ and $b$. By Theorems~50 and 51 the function is defined for every value of $y$ between and including $f(a)$ and $f(b)$ and for no other values. We prove the function continuous on the interval $\interval{f(a)}{f(b)}$ by showing that it is continuous at any point $y=y_1$ on this interval. As $y$ approaches $y_1$ on the interval $\interval{f(a)}{{y_1}}$, $f^{-1}(y)$ approaches a definite limit $g$ by Theorem~\hyperlink{thm25}{25}, and by Theorem~\hyperlink{thm40}{40} $a0$ there exists a $ \delta_\varepsilon > 0$ such that for any two values of $x'$, $x_1'$, and $x_2'$ an $\interval{a}{b}$, for which $ |x_1'-x_2'| < \delta_\varepsilon$, $|f(x_1')-f(x_2')| < \varepsilon$. \end{definition} \begin{theorem}[55]\hypertarget{thm55}{} If a function $f(x')$ is defined on a set everywhere dense on the interval $\interval{a}{b}$ and is uniformly continuous over that set, then there exists one and only one function $f(x)$ defined on the full interval $\interval{a}{b}$ such that: \begin{enumerate} \item[\textnormal{(1)}] $f(x)$ is identical with $f(x')$ where $f(x')$ is defined. \item[\textnormal{(2)}] $f(x)$ is continuous on the interval $\interval{a}{b}$. \end{enumerate} \end{theorem} \begin{proof} Let $x''$ be any point on the interval $\interval{a}{b}$, but not of the set $[x']$. We first prove that \[ \mathop{L}_{x' \doteq x''} f(x') \] exists and is finite. By the definition of uniform continuity, for every $\varepsilon$ there exists a $\delta_\varepsilon$ such that for any two values of $x'$, $x_1'$, and $x_2'$, where $|x_1'-x_2'| < \delta_\varepsilon$, $|f(x_1')-f(x_2)| < \varepsilon $. Hence we have for every pair of values $x_1'$ and $x_2'$ where $|x_1'-x''|< \dfrac {\delta_\varepsilon}{2}$ and $|x_2'-x''| < \dfrac{\delta_\varepsilon}{2}$ that $|f(x_1')-f(x_2')|< \varepsilon$. By Theorem~\hyperlink{thm23}{23} this is a sufficient condition that \[ \mathop{L}_{x' \doteq x''}f(x') \] shall exist and be finite. Let $f(x)$ denote a function identical with $f(x')$ on the set $[x']$ and equal to \[ \mathop{L}_{x' \doteq x''} f(x') \] at all points $x''$. This function is defined upon the continuum, %-----File: 108.png---Folio 96------- since all points $x''$ on $\interval{a}{b}$ are limit points of the set $[x']$. Hence the function has the property that $\displaystyle \mathop{L}_{x_1\doteq x} f(x')=f(x)$ for every $x$ of $\interval{a}{b}$. We next prove that $f(x)$ is continuous at every point on the interval $\interval{a}{b}$, in other words that $f(x)$ cannot approach a value $b$ different from $f(x_1)$ as $x$ approaches $x_1$. We already know that $f(x)$ approaches $f(x_1)$ on the set $[x']$. If $b$ is another value approached, then for every positive $\varepsilon$ and $\delta$ there is an $x_{\varepsilon\delta}$ such that \hypertarget{eq1p97}{\[ |x_{\varepsilon\delta}-x_1|<\delta, \qquad|f(x_{\varepsilon\delta})-b|<\varepsilon.\tag{1} \]} Since $f(x_{\varepsilon\delta}) =\displaystyle\mathop{L}_{x'\doteq x_{\varepsilon\delta}} f(x')$ we have that for every $\varepsilon>0$ there exists a $\delta_\varepsilon>0$ such that for every $x'$ for which $|x'-x_{\varepsilon\delta}|<\delta_\varepsilon$, \hypertarget{eq2p97}{\[ |f(x')-f(x_{\varepsilon\delta})|<\varepsilon. \tag{2} \]} From \hyperlink{eq1p97}{(1)} and \hyperlink{eq2p97}{(2)} we have \hypertarget{eq3p97}{\[ |f(x')-b|<2\varepsilon. \tag{3} \]} Since the $\delta$ of \hyperlink{eq1p97}{(1)} is any positive number, there is an $x_{\varepsilon\delta}$ on every neighborhood of $x_1$ and hence by \hyperlink{eq2p97}{(2)} and \hyperlink{eq3p97}{(3)} an $x'$ on every neighborhood of $x_1$ such that $|f(x')-b| <2\varepsilon$, $\varepsilon$ being arbitrary and $b$ a constant different from $f(x_1'')$. But this is contrary to the fact proved above, that $\displaystyle \mathop{L}_{x'\doteq x_1}f(x')$ exists and is equal to $f(x_1)$. Hence the function is continuous at every point of the interval $\interval{a}{b}$. The uniqueness of the function follows directly from Theorem~\hyperlink{thm54}{54}. \end{proof} This theorem can be applied, for example, to give an elegant definition of the exponential function (see Chap.~\hyperlink{chapIII}{III}). We first show that the function $a^\frac mn$ is uniformly continuous on the set of all rational values between $x_1$ and $x_2$, and then define %-----File: 109.png---Folio 97------- $a^x$ on the continuum as that continuous function which coincides with $a^\frac mn$ for the rational values $\dfrac mn$. The properties of the function then follow very easily. It will be an excellent exercise for the reader to carry out this development in detail. \section{The Exponential Function.}\hypertarget{chVsec4}{}%[4] \label{s4p97} Consider the function defined by the infinite series \[ 1+x+\frac{x^2}{2!}+\frac{x^3}{3!}+\ldots+\frac{x^n}{n!}+\ldots. \tag{1} \] Applying the ratio test for the convergence of infinite series we have \[ \frac{x^n}{n!}\div\frac{x^{n-1}}{(n-1)!}=\frac xn. \] If $n'$ is a fixed integer larger than $x$, this ratio is always less than $\dfrac x{n'}<1$. The series~(1) therefore converges absolutely for every value of $x$, and we may denote its sum by \[ e(x). \] From Chap.~\hyperlink{chapI}{I}, page~\pageref{t7p17}, we have that \[ e(1)= \mathop{L}_{n\doteq\infty} \left(1+\frac1n\right)^n =e. \] \begin{theorem}[56]\hypertarget{thm56}{} \[ \mathop{L}_{n\doteq\infty} \left(1+\frac xn\right)^n, \] where $[n]$ is the set of all positive integers, exists and is equal to $e(x)$ for all values of $x$. \end{theorem} %-----File: 110.png---Folio 98------- \begin{proof} Let \[ E_n(x) = \sum_{k=0}^n\frac{x^k}{k!} \] (where $0! = 1$). Then, since \[ \left(1+\frac xn\right)^n = 1 + \frac{n!}{(n-1)!} \cdot\frac xn + \frac{n!}{(n-2)!\cdot 2!} \left(\frac xn\right)^2 + \ldots + \frac{n!}{n!} \left(\frac xn\right)^n, \] it follows that \begin{align*} \left|E_n(x)-\left(1+\frac xn\right)^n\right| &= \left|\sum_{k=2}^n\left(\frac1{k!}- \frac{n!}{(n-k)!\cdot k!\,n^k} \right)x^k\right| \\ &\leqq \sum_{k=2}^n\left(\frac1{k!}- \frac{n(n-1)\ldots(n-k+1)}{k!\,n^k} \right)\cdot|x^k| \\ &<\sum_{k=2}^n \frac{n^k-(n-k+1)^k}{k!\,n^k}\cdot|x^k|. \end{align*} Now, since \begin{multline*} n^k-(n-k+1)^k = (k-1)\{n^{k-1}+n^{k-2}\cdot(n-k+1)+\ldots \\ +(n-k+1)^{k-1}\} < (k-1)k\cdot n^{k-1}, \end{multline*} it follows that \[ \left|E_n(x)-\left(1+\frac xn\right)^n\right| < \sum_{k=2}^n \frac{|x|^k}{(k-2)!\cdot n} < \frac{x^2\cdot e(|x|)}{n}. \] For a fixed value of $x$, therefore, we have \[ \left(1+\frac xn\right)^n = E_n(x)+\varepsilon_1(n), \] where $\varepsilon_1(n)$ is an infinitesimal as $n\doteq\infty$. At the same time \[ e(x) = E_n(x) + \varepsilon_2(n), \] where $\varepsilon_2(n)$ is an infinitesimal as $n\doteq\infty$. Hence \[ \mathop{L}_{n\doteq\infty} \left(1+\frac xn\right)^n = e(x).\qedhere \] \end{proof} %-----File: 111.png---Folio 99------- \begin{theorem}[57]\hypertarget{thm57}{} \[ \mathop{L}_{z\doteq \infty} \left(1+\frac xz\right), \] where $[z]$ is the set of all real numbers, exists and is equal to $e(x)$. \end{theorem} \begin{proof} If $z$ is any number greater than $1$, let $n_z$ be the integer such that \[ n_z\leqq z0$, \[ 1+\frac x{n_z}\geqq1+\frac xz >1+\frac x{n_z+1}. \tag{1} \] Hence \[ \left(1+\frac x{n_z}\right)^{n_z+1}\geqq \left(1+\frac xz\right)^z > \left(1+\frac x{n_z+1}\right)^{n_z}, \tag{2} \] or \[ \left(1+\frac x{n_z}\right) \left(1+\frac x{n_z}\right)^{n_z} \geqq \left(1+\frac xz\right)^z > \left(1+\frac x{n_z+1}\right)^{n_z+1}\cdot \frac{1}{1+\frac{x}{n_z+1}}. \tag{3} \] Since \begin{alignat*}{2} \mathop{L}_{z\doteq\infty} \left(1+\frac x{\text{\correction{$n_z$}{$n$}}}\right) &=1, &\text{ and } \mathop{L}_{z\doteq\infty}\left(1+\frac x{n_z+1}\right) &=1,\\ \intertext{and} \mathop{L}_{z\doteq\infty}\left(1+\frac x{n_z}\right)^{n_z} &=e(x), &\text{and} \mathop{L}_{z\doteq\infty} \left(1+\frac x{n_z+1}\right)^{n_z+1}&=e(x), \end{alignat*} the inequality~(3), together with Corollary~\hyperlink{cor3p82}{3}, Theorem~\hyperlink{thm40}{40}, leads to the result: \[ \mathop{L}_{z\doteq\infty} \left(1+\frac xz\right)^z=e(x). \] The argument is similar if $x<0$. \end{proof} \begin{corollary} \[ \mathop{L}_{z\doteq\infty} \left(1+\frac xz\right)^z=e(x), \] where $[z]$ is any set of numbers with limit point $+\infty$. \end{corollary} \begin{theorem}[58]\hypertarget{thm58}{}\label{t58p99} The function $e(x)$ is the same as $e^x$ where \[ e=1+1+\frac1{2!}+\frac1{3!}+\ldots \] \end{theorem} %-----File: 112.png---Folio 100------ \begin{proof} By the continuity of $z^x$ as a function of $z$ (see Corollary~\hyperlink{cor2p81}{2} of Theorem~\hyperlink{thm39}{39}), it follows that, since \begin{align*} \mathop{L}_{n\doteq \infty}\left(1+\frac1n\right)^n &= e,\\ \mathop{L}_{n\doteq \infty}\left(1+\frac1n\right)^{nx} &= e^x. \end{align*} But \[ \left(1+\frac1n\right)^{nx} = \left(1+\frac x{nx}\right)^{nx} = \left(1+\frac xz\right)^z, \] where $z=nx$. Hence by Theorem~\hyperlink{thm39}{39} \[ e^x = \mathop{L}_{z\doteq \infty}\left(1+\frac xz\right)^z, \] and by the corollary of Theorem~\hyperlink{thm57}{57} the latter expression is equal to $e(x)$. Hence we have \hypertarget{eq1p100}{\[ e^x = 1+x+\frac{x^2}{2!}+\frac{x^3}{3!}+\ldots.\tag{1} \]} \hyperlink{eq1p100}{(1)} is frequently used as the definition of $e^x$, $a^x$ being defined as $e^x\cdot\log_e a$. \end{proof} %-----File: 113.png---Folio 101------ \chapter{INFINITESIMALS AND INFINITES.}\hypertarget{chapVI}{}%[VI] \section{The Order of a Function at a Point.}\hypertarget{chVIsec1}{}%[1] An infinitesimal has been defined (page~\pageref{dp75}) as a function $f(x)$ such that \[ \mathop{L}_{x \doteq a} f(x)=0. \] A function which is unbounded in every vicinity of $x=a$ is said to have an \index{Function!infinite at a point}\textit{infinity} at $a$, to be or become \index{Infinite}infinite at $x=a$, or to have an \index{Singularity}\textit{infinite singularity} at $x=a$.\footnote{% It is perfectly compatible with these statements to say that while $f(x)$ has an infinite singularity at $x=a$, $f(a)=0$ or any other finite number. For example, a function which is $\dfrac{1}{x}$ for all values of $x$ except $x=0$ is left undefined for $x=0$ and hence at this point the function may be defined as zero or any other number. This function illustrates very well how a function which has a finite value at every point may nevertheless have infinite singularities.} The reciprocal of an infinitesimal at $x=a$ is infinite at this point. A function may be infinite at a point in a variety of ways: \begin{enumerate} \item[(\textit{a})] It may be monotonic and approach $+\infty$ or $-\infty$ as $x \doteq a$; for example, $\dfrac{1}{x}$ as $x$ approaches zero from the positive side. \item[(\textit{b})] It may oscillate on every neighborhood of $x=a$ and still approach $+\infty$ or $-\infty$ as a unique limit; for example, \[ \frac{\sin\dfrac{1}{x}+2}{x} \] as $x$ approaches zero. %-----File: 114.png---Folio 102------ \item[(\textit{c})] It may approach any set of real numbers or the set of all real numbers; an example of the latter is \[ \frac{\sin\dfrac1x}{x} \] as $x$ approaches zero. See Fig.~\hyperlink{fig13}{13}, page~\pageref{fig13}. \item[(\textit{d})] $+\infty$ and $-\infty$ may both be approached while no other number is approached; for example, $\frac1x$ as $x$ approaches zero from both sides. \end{enumerate} \begin{defnorder}\index{Order of function} If $f(x)$ and $\phi(x)$ are two functions such that in some neighborhood $V^*(a)$ neither of them changes sign or is zero, and if \[ \mathop{L}_{x \doteq a} \frac{f(x)}{\phi(x)} = k, \] where $k$ is finite and not zero, then $f(x)$ and $\phi(x)$ are said to be of the \textit{same order} at $x=a$. If \[ \mathop{L}_{x \doteq a} \frac{f(x)}{\phi(x)} = 0, \] then $f(x)$ is said to be \textit{infinitesimal with respect to} $\phi(x)$, and $\phi(x)$ is said to be \index{Infinite}\textit{infinite with respect to} $f(x)$. If \[ \mathop{L}_{x \doteq a} \frac{f(x)}{\phi(x)} = +\infty \text{ or }-\infty, \] then, by Theorem~\hyperlink{thm37}{37}, $\phi(x)$ is infinitesimal with respect to $f(x)$, and $f(x)$ infinite with respect to $\phi(x)$. If $f(x)$ and $\phi(x)$ are both infinitesimal at $x=a$, and $f(x)$ is infinitesimal with respect to $\phi(x)$, then $f(x)$ is infinitesimal of a \textit{higher order} than $\phi(x)$, and $\phi(x)$ of \textit{lower order} than $f(x)$. If $\phi(x)$ and $f(x)$ are both infinite at $x=a$, and $f(x)$ is infinite with respect to $\phi(x)$, then $f(x)$ is %-----File: 115.png---Folio 103------ infinite of higher order than $\phi(x)$, and $\phi(x)$ is infinite of lower order than $f(x)$.\footnote{% This definition of order is by no means as general as it might possibly be made. The restriction to functions which are not zero and do not change sign may be partly removed. The existence of \[ \underset{x\doteq a}L\frac{f(x)}{\phi(x)} \] is dispensed with for some cases in \hyperlink{chVIsec4}{\S~4} on Rank of Infinitesimals and Infinites. For an account of still further generalizations (due mainly to \textsc{Cauchy}) see \textsc{E.~Borel}, \textit{S\'eries \correction{\`a}{a} Termes Positifs}, Chapters III and IV, Paris, 1902. An excellent treatment of the material of this section together with extensions of the concept of order of infinity is due to \label{borlottip103}\textsc{E.~Borlotti}, {\it Calcolo degli Infinitesimi}, Modena, 1905 (62 pages).} \end{defnorder} The independent variable $x$ is usually said to be an infinitesimal of the first order as $x$ approaches zero, $x^2$ of the second order, etc. Any constant $\neq 0$ is said to be infinite of zero order, $\dfrac{1}{x}$ is of the first order, $\dfrac{1}{x^2}$ of the second order, etc. This usage, however, is best confined to analytic functions. In the general case there are no two infinitesimals of consecutive order. Evidently there are as many different orders of infinitesimals between $x$ and $x^2$ as there are numbers between $1$ and $2$; i.e., $x^{1+k}$ is of higher order than $x$ for every positive value of $k$. Since $\displaystyle\mathop{L}_{x\doteq a}\frac{f_1(x)}{f_2(x)}=\frac1k$ whenever $\displaystyle\mathop{L}_{x\doteq a}\frac{f_2(x)}{f_1(x)}=k$, we have \begin{theorem}[59]\hypertarget{thm59}{} If $f_1(x)$ is of the same order as $f_2(x)$, then $f_2(x)$ is of the same order as $f_1(x)$. \end{theorem} \begin{theorem}[60]\hypertarget{thm60}{} The function $cf(x)$ is of the same order as $f(x)$, $c$ being any constant not zero. \end{theorem} \begin{proof} By Theorem~\hyperlink{thm34}{34}, $\displaystyle\mathop{L}_{x\doteq a}\frac{cf(x)}{f(x)}=c$. \end{proof} \begin{theorem}[61]\hypertarget{thm61}{} If $f_1(x)$ is of the same order as $f_2(x)$, and $f_2(x)$ is of the same order as $f_3(x)$, then $f_1(x)$ and $f_3(x)$ are of the same order. \end{theorem} %-----File: 116.png---Folio 104------ \begin{proof} By hypothesis $\displaystyle\mathop{L}_{x \doteq a} \frac{f_1(x)}{f_2(x)}=k_1$ and $\displaystyle\mathop{L}_{x \doteq a} \frac{f_2(x)}{f_3(x)}=k_2$. By Theorem~\hyperlink{thm34}{34}, \[ \displaystyle\mathop{L}_{x \doteq a} \frac{f_1(x)}{f_2(x)} \cdot \mathop{L}_{x \doteq a} \frac{f_2(x)}{f_3(x)} = \mathop{L}_{x \doteq a} \frac{f_1(x)}{f_3(x)}. \] (By definition, $f_2(x) \neq 0$ and $f_3(x)\neq 0$ for some neighborhood of $x=a$.) Hence \[ \mathop{L}_{x \doteq a} \frac{f_1(x)}{f_3(x)} = k_1 \cdot k_2.\qedhere \] \end{proof} \begin{theorem}[62]\hypertarget{thm62}{} If $f_1(x)$ and $f_2(x)$ are infinitesimal (infinite) and neither is zero or changes sign on some $V^*(a)$, then $f_1(x)\cdot f_2(x)$ is infinitesimal (infinite) of a higher order than either. \end{theorem} \begin{proof} \[ \mathop{L}_{x \doteq a} \frac{f_1(x) \cdot f_2(x)}{f_2(x)} = \mathop{L}_{x \doteq a} f_1(x) = 0.\ (\pm \infty.)\qedhere \] \end{proof} \begin{theorem}[63]\hypertarget{thm63}{} If $f_1(x)$, $\ldots$, $f_n(x)$ have the same sign on some $V^*(a)$ and if $f_2(x)$, $\ldots$, $f_n(x)$ are infinitesimal (infinite) of the same or higher (lower) order than $f_1(x)$, then \[ f_1(x) + f_2(x) + f_3(x) + \ldots + f_n(x) \] is of the same order as $f_1(x)$, and if $f_2(x)$, $f_3(x)$, $\ldots$, $f_n(x)$ are of higher (lower) order than $f_1(x)$, then $f_1(x) \pm f_2(x) \pm f_3(x) \pm \ldots \pm f_n(x)$ is of the same order as $f_1(x)$. \end{theorem} \begin{proof} We are to show that \[ \mathop{L}_{x \doteq a} \frac{f_1(x) + f_2(x) + \ldots + f_n(x)}{f_1(x)} = k \neq 0. \] By hypothesis, \[ \mathop{L}_{x \doteq a} \frac{f_2(x)}{f_1(x)} = k_2, \; \mathop{L}_{x \doteq a} \frac{f_3(x)}{f_1(x)} = k_3, \; \ldots, \; \mathop{L}_{x \doteq a} \frac{f_n(x)}{f_1(x)} = k_n, \] and \[ \mathop{L}_{x \doteq a} \frac{f_1(x)}{f_1(x)} = 1. \] %-----File: 117.png---Folio 105------ Hence, by Theorem~\hyperlink{thm30}{30}, \[ \mathop{L}_{x \doteq a} \left\{% \frac{f_1(x)}{f_1(x)} + \frac{f_2(x)}{f_1(x)} + \frac{f_3(x)}{f_1(x)} + \ldots + \frac{f_n(x)}{f_1(x)} \right\} = 1 + k_2 \text{\correction{$+\ldots+$}{$\ldots$}} k_n = k \neq 0, \] since all the $k$'s are positive or zero. Similarly, under the second hypothesis, \begin{align*} \mathop{L}_{x \doteq a} \frac{f_1(x) \pm f_2(x) \pm \ldots \pm f_n(x)}{f_1(x)} & = \mathop{L}_{x \doteq a} \left\{% \frac{f_1(x)}{f_1(x)} \pm \frac{f_2(x)}{f_1(x)} \pm \ldots \pm \frac{f_n(x)}{f_1(x)} \right\}\\ & = 1 + 0 + \ldots + 0 = 1.\qedhere \end{align*} \end{proof} \begin{theorem}[64]\hypertarget{thm64}{} If $f_3(x)$ and $f_4(x)$ are infinitesimals with respect to $f_1(x)$ and $f_2(x)$, then \[ \mathop{L}_{x \doteq a} \frac{\{f_1(x) + f_3(x)\} \cdot \{f_2(x) + f_4(x)\}}{f_1(x)\cdot f_2(x)}=1. \] \end{theorem} \begin{proof} \begin{align*} &\mathop{L}_{x \doteq a} \frac{\{f_1(x) + f_3(x)\} \cdot \{f_2(x) + f_4(x)\}}{f_1(x)\cdot f_2(x)} \\ =&\mathop{L}_{x \doteq a} \frac{f_1(x)\cdot f_2(x) + f_1(x)\cdot f_4(x) + f_3(x)\cdot f_2(x) + f_3(x)\cdot f_4(x)}{f_1(x)\cdot f_2(x)} \\ =&\mathop{L}_{x \doteq a} \frac{f_1(x)\cdot f_2(x)}{f_1(x)\cdot f_2(x)} + \mathop{L}_{x \doteq a} \frac{f_1(x)\cdot f_4(x)}{f_1(x)\cdot f_2(x)} + \mathop{L}_{x \doteq a} \frac{f_3(x) \cdot f_2(x)} {f_1(x)\cdot f_2(x)} + \mathop{L}_{x \doteq a} \frac{f_3(x) \cdot f_4(x)} {f_1(x)\cdot f_2(x)} = 1.\qedhere \end{align*} \end{proof} \section{The Limit of a Quotient.}\hypertarget{chVIsec2}{}%[2] \begin{theorem}[65]\hypertarget{thm65}{} If as $x \doteq a$, $\varepsilon_1(x)$ is an infinitesimal with respect to $f_1(x)$ and $\varepsilon_2(x)$ with respect to $f_2(x)$, then the values approached by \[ \frac{f_1(x) + \varepsilon_1(x)}{f_2(x) + \varepsilon_2(x)} \quad \text{and} \quad \frac{f_1(x)}{f_2(x)} \] as $x$ approaches $a$ are identical. \end{theorem} %-----File: 118.png---Folio 106------ \begin{proof} This follows from the identity \[ \frac{f_1(x) + \varepsilon_1(x)}{f_2(x) + \varepsilon_2(x)} = \frac{f_1(x)}{f_2(x)} \cdot \frac{\left(1 + \dfrac{\varepsilon_1(x)}{f_1(x)}\right)}% {\left(1 + \dfrac{\varepsilon_2(x)}{f_2(x)}\right)}, \] \correction{since}{Since} $\dfrac{\varepsilon_1(x)}{f_1(x)} $ and $\dfrac{\varepsilon_2(x)}{f_2(x)}$ are infinitesimal. \end{proof} \begin{corollary} If $f_1(x)$ and $f_2(x)$ are infinite at $x=a$, then \[ \frac{f_1(x) + c} {f_2(x) + d} \quad \text{and} \quad \frac{f_1(x)}{f_2(x)} \] approach the same values. \end{corollary} \begin{theorem}[66]\hypertarget{thm66}{} If $\displaystyle \mathop{L}_{x \doteq a} \dfrac{f_1(x)}{\phi_1(x)} = \mathop{L}_{x \doteq a} \dfrac{f_2(x)}{\phi_2(x)} = k$, and if $\displaystyle \mathop{L}_{x \doteq a} \frac{\phi_1(x)}{\phi_2(x)} = l$\\ is finite, then \[ k = \mathop{L}_{x \doteq a} \frac{f_1(x) + f_2(x)}{\phi_1(x) + \phi_2(x)} = \mathop{L}_{x \doteq a_1} \frac{f_1(x)}{\phi_1(x)}, \] provided $l \neq-1$ if $k$ is finite, and provided $l>0$ if $k$ is infinite. \end{theorem} \begin{proof} \begin{align*} &\frac{f_1(x) + f_2(x)}{\phi_1(x) + \phi_2(x)}-\frac{f_2(x)}{\phi_2(x)} = \frac{f_1(x)\phi_2(x)-f_2(x)\phi_1(x)}{\phi_2(x)(\phi_1(x) + \phi_2(x))},\\ &\frac{f_1(x) + f_2(x)}{\phi_1(x) + \phi_2(x)} = \frac{f_2(x)}{\phi_2(x)} + \left(\frac{f_1(x)}{\phi_1(x)}-\frac{f_2(x)}{\phi_2(x)}\right) \cdot \left(\frac{1}{1 + \dfrac{\phi_2(x)}{\phi_1(x)}} \right). \end{align*} In case $k$ is finite, the second term of the right-hand member is evidently infinitesimal if $l \neq-1$ and the theorem is proved. In the case where $k$ is infinite we write the above identity in the following form: \[ \frac{f_1(x) + f_2(x)}{\phi_1(x) + \phi_2(x)} = \frac{f_1(x)}{\phi_1(x)} \cdot \frac{1}{1 + \dfrac{\phi_2(x)}{\phi_1(x)}} + \frac{f_2(x)}{\phi_2(x)} \cdot \frac{1}{1 + \dfrac{\phi_1(x)}{\phi_2(x)}}. \] %-----File: 119.png---Folio 107------ Both terms of the second member approach $+\infty$ or both $-\infty$ if $l>0$. \end{proof} \begin{corollary} If $\phi_1(x)$ and $\phi_2(x)$ are both positive for some $V^*(a)$, and if $\displaystyle k=\mathop{L}_{x\doteq a} \frac{f_1(x)}{\phi_1(x)} = \mathop{L}_{x\doteq a} \frac{f_2(x)}{\phi_2(x)}$, then $\displaystyle\mathop{L}_{\text{\correction{$x\doteq a$}{$x=a$}}} \frac{f_1(x)+f_2(x)}{\phi_1(x)+\phi_2(x)} = k$ whenever $k$ is finite. If $k$ is infinite, the condition must be added that $\dfrac{\phi_1(x)}{\phi_2(x)}$ has a finite upper and a non-zero lower bound. \end{corollary} \begin{theorem}[67]\hypertarget{thm67}{} If $f_1(x)$ and $f_2(x)$ are both infinitesimals as $x\doteq a$, then a necessary and sufficient condition that \[ \mathop{L}_{x\doteq a} \frac{f_1(x)}{f_2(x)} =k\qquad \text{($k$ finite and not zero)} \] is that in the equation $f_1(x)=k\cdot f_2(x) + \varepsilon(x)$, $\varepsilon(x)$ is an infinitesimal of higher order than $f_1(x)$ or $f_2(x)$. \end{theorem} \begin{proof} (1) \emph{The condition is necessary.}---Since $\displaystyle\mathop{L}_{x\doteq a} \frac{f_1(x)}{f_2(x)}=k$, \[ \frac{f_1(x)}{f_2(x)}=k+\varepsilon'(x), \] or $f_1(x)=f_2(x)\cdot k+f_2(x)\cdot\varepsilon'(x)$, where $\displaystyle\mathop{L}_{x\doteq a} \varepsilon'(x)=0$ (Theorem~\hyperlink{thm31}{31}). By Theorems \hyperlink{thm60}{60} and \hyperlink{thm61}{61}, $f_1(x)$ and $f_2(x)\cdot k$ are of the same order, since $k\neq0$, while by Theorem~\hyperlink{thm62}{62} $\varepsilon'(x)\cdot f_2(x)$ is of higher order than either $f_1 (x)$ or $f_2(x)$. Hence the function $\varepsilon(x) = \varepsilon'(x)\cdot f_2(x)$ is infinitesimal. (2) \emph{The condition is sufficient.}---By hypothesis $f_1(x) = f_2(x)\cdot k + \varepsilon(x)$, where $f_1(x)$ and $f_2(x)$ are of the same order as $x\doteq a$, while $\varepsilon(x)$ is of higher order than these. Let $\varepsilon'(x) =\dfrac{\varepsilon(x)}{f_2(x)}$, which by hypothesis is an infinitesimal. We then have $\dfrac{f_1(x)}{f_2(x)} =k+ \varepsilon'(x)$. Hence, by Theorem~\hyperlink{thm31}{31}, $\displaystyle \mathop{L}_{x\doteq a} \dfrac{f_1(x)}{f_2(x)}=k$. \end{proof} %-----File: 120.png---Folio 108------ \section[Indeterminate Forms]{Indeterminate Forms.\footnotemark}\hypertarget{chVIsec3}{}%[3] \footnotetext{% The theorems of this section are to be used in \hyperlink{chVIIsec6}{\S~6} of Chap.~\hyperlink{chapVII}{VII}.} \begin{lemma} If $\dfrac ab$ and $\dfrac cd$ are any two fractions such, that $b$ and $d$ are both positive or both negative, then the value of \[ \frac{a + c}{b+d} \] lies on the interval $\interval{\dfrac ab}{\dfrac cd}$. \end{lemma} \begin{proof} Suppose $b$ and $d$ both positive and \[ \frac ab \geqq \frac{a+c}{b+d}, \] then \begin{gather*} ab+ad \geqq ab+bc.\\ \therefore ad \geqq bc;\\ \therefore cd+ad \geqq cd+bc;\\ \therefore \frac{a+c}{b+d} \geqq \frac cd. \end{gather*} The other cases follow similarly. \end{proof} \begin{theorem}[68]\hypertarget{thm68}{} If $f(x)$ and $\phi(x)$, defined on some $V(+\infty)$, are both infinitesimal as $x$ approaches $+\infty$, and if for some positive number $h$, $\phi(x+h)$ is always less than $\phi(x)$ and \[ \mathop{L}_{x\doteq\infty} \frac{f(x+h)-f(x)}{\phi(x+h)-\phi(x)}=k, \] then \[ \mathop{L}_{x\doteq\infty} \frac{f(x)}{\phi(x)} \] exists and is equal to $k$.\footnote{% This and the following theorem are due to \textsc{O.~Stolz}, who generalized them from the special cases (stated in our corollaries) due to \textsc{Cauchy}. See \textsc{Stolz} und \textsc{Gmeiner}, Functionentheorie, Vol.~1, p.~31. See also the reference to \textsc{Bortolotti} given on page~\pageref{borlottip103}.} \end{theorem} %-----File: 121.png---Folio 109------ \begin{proof} Let $V_1(k)$ and $V_2(k)$ be a pair of vicinities of $k$ such that $V_2(k)$ is entirely within $V_1(k)$. By hypothesis there exists an $h$ and an $X_2$ such that if $x>X_2$, \hypertarget{eq1p109}{\[ \frac{f(x+h)-f(x)}{\phi(x+h)-\phi(x)} \tag{1} \]} is in $V_2(k)$. Since this is true for every $x>X_2$, \hypertarget{eq2p109}{\[ \frac{f(x+2h)-f(x+h)}{\phi(x+2h)-\phi(x+h)}\tag{2} \]} is also in $V_2(k)$. From this it follows by means of the lemma that \[ \frac{f(x+2h)-f(x)}{\phi(x+2h)-\phi(x)},\tag{3} \] whose value is between the values of \hyperlink{eq1p109}{(1)} and \hyperlink{eq2p109}{(2)}, is also in $V_2(k)$. By repeating this argument we have that for every integral value of $n$, and for every $x>X_2$, \[ \frac{f(x+nh)-f(x)}{\phi(x+nh)-\phi(x)} \] is in $V_2(k)$. By Theorem~\hyperlink{thm65}{65}, for any $x$ \[ \mathop{L}_{n\doteq\infty} \frac{f(x+nh)-f(x)}{\phi(x+nh)-\phi(x)} = \frac{f(x)}{\phi(x)}. \] Hence for every $x$ and for every $\varepsilon$ there exists a value of $n$, $N_{x\varepsilon}$, such that if $n>N_{x\varepsilon}$, \[ \left| \frac{f(x+nh)-f(x)}{\phi(x+nh)-\phi(x)}-\frac{f(x)}{\phi(x)} \right| < \varepsilon. \] Taking $\varepsilon$ less than the distance between the nearest end-points of $V_1(k)$ and $V_2(k)$ it is plain that for every $x>X_2$, $\dfrac{f(x)}{\phi(x)}$ is %-----File: 122.png---Folio 110------ on $V_1(k)$, which, according to Theorem~\hyperlink{thm26}{26}, proves that \[ \mathop{L}_{x\doteq\infty} \frac{f(x)}{\phi(x)} = k.\qedhere \] \end{proof} \begin{corollary} If $[n]$ is the set of all positive integers and $\phi(n+1)<\phi(n)$ and $f(n)$ and $\phi(n)$ are both infinitesimal as $n\doteq\infty$, then if \[ \mathop{L}_{n\doteq\infty} \frac{f(n+1)-f(n)}{\phi(n+1)-\phi(n)} =k, \] it follows that $\displaystyle \mathop{L}_{n\doteq\infty} \dfrac{f(n)}{\phi(n)}$ exists and is equal to $k$. \end{corollary} \begin{theorem}[69]\hypertarget{thm69}{} If $f(x)$ is bounded on every finite interval of a certain $V(+\infty)$, and if $\phi(x)$ is monotonic on the same $V(+\infty)$ and $\displaystyle \mathop{L}_{x\doteq\infty} \phi(x) = +\infty$, and if for some positive number $h$ \[ \mathop{L}_{x\doteq\infty} \frac{f(x+h)-f(x)}{\phi(x+h)-\phi(x)} =k, \] then \[ \mathop{L}_{x\doteq\infty} \frac{f(x)}{\phi(x)} \] exists and is equal to $k$. \end{theorem} \begin{proof} By hypothesis, for every pair of vicinities $V_1(k)$ and $V_2(k)$, $V_2(k)$ entirely within $V_1(k)$, there exists an $X_2$ such that if $x>X_2$, then \[ \frac{f(x+h)-f(x)}{\phi(x+h)-\phi(x)} \] is in $V_2(k)$. From this it follows as in the last theorem that \[ \frac{f(x+nh)-f(x)}{\phi(x+nh)-\phi(x)} \] is in $V_2(k)$. Now make use of the identity %-----File: 123.png---Folio 111------ \hypertarget{eq1p111}{\begin{align*} \frac{f(x+nh)}{\phi(x+nh)} &= \frac{f(x+nh)-f(x)}{\phi(x+nh)}+\frac{f(x)}{\phi(x+nh)}\\ &= \frac{f(x+nh)-f(x)}{\phi(x+nh)-\phi(x)} \left(1-\frac{\phi(x)}{\phi(x+nh)} \right) + \frac{f(x)}{\phi(x+nh)}. \tag{1} \end{align*}} Let $[x']$ be the set of all points on the interval \correction{$\interval{X_2}{X_2+h}$}{$\interval{X_2}{X_2}+h$}, and for this interval let $A_2$ be an upper bound of $|f(x')|$ and $B_2$ an upper bound of $\phi(x')$. Then \begin{alignat*}{2} \frac{\phi(x')}{\phi(x'+nh)} &= \varepsilon_1(x', n) &&< \frac{B_2}{\phi(X_2+nh)} \\ \intertext{and} \frac{|f(x')|}{\phi(x'+nh)} &= \varepsilon_2(x', n) &&< \frac{A_2}{\phi(X_2+nh)}. \end{alignat*} Hence for every $\varepsilon$ there exists a value of $n$, $N_{\varepsilon_V}$, such that if $n > N_{\varepsilon_V}$ \hypertarget{eq2p111}{\[ \varepsilon_1(x', n) < \varepsilon \qquad \text{ and } \qquad \varepsilon_2(x', n) < \varepsilon \tag{2} \]} independently of $x'$ so long as $x'$ is on \correction{$\interval{X_2}{X_2+h}$}{$\interval{X_2}{X_2}+h$}. There are then three cases to discuss: \begin{align*} (1)&\ k \text{ finite.} & (2)&\ k = +\infty. & (3)&\ k =-\infty. \end{align*} (1) $k$ {\em finite}. By the preceding argument, for $x > X_2$, \[ \frac{f(x+nh)-f(x)}{\phi(x+nh)-\phi(x)} \] is in $V_2(k)$, and hence \[ \frac{|f(x'+nh)-f(x')|}{\phi(x'+nh)-\phi(x')} < K + \varepsilon_{V_2}, \] where $\varepsilon_{V_2}$, is the length of the interval $V_2(k)$ and $K$ the absolute value of $k$. Then, in view of \hyperlink{eq1p111}{(1)}, \[ \left|\frac{f(x'+nh)}{\phi(x'+nh)} -\frac{f(x'+nh)-f(x')}{\phi(x'+nh)-\phi(x')} \right| < (K+\varepsilon_{V_2}) \varepsilon_1(x',n) + \varepsilon_2(x',n). \] %-----File: 124.png---Folio 112------ Now take $\varepsilon_V$ smaller in absolute value than the length of the interval between the closer end-points of $V_1(k)$ and $V_2(k)$. By \hyperlink{eq2p111}{(2)} there exists a value of $n$, $N_{\varepsilon_V}$, such that if $n>N_{\varepsilon_V}$, \begin{align*} \varepsilon_1(x', n) &< \frac{\varepsilon_V}{2(K+\varepsilon_{V_2})}\\ \intertext{and} \varepsilon_2(x', n) &< \frac{\varepsilon_V}{2} \end{align*} for all values of $x'$ on \correction{$\interval{X_2}{X_2+h}$}{$\interval{X_2}{X_2}+h$}. Hence for $n > N_{\varepsilon_V}$ \[ \left|\frac{f(x'+nh)}{\phi(x'+nh)} -\frac{f(x'+nh)-f(x')}{\phi(x'+nh)-\phi(x')} \right| < (K + \varepsilon_{V_2}) \frac{\varepsilon_V}{2(K+\varepsilon_{V_2})} + \frac{\varepsilon_V}{2} = \varepsilon_V, \] and since for $x > X_2 + N_{\varepsilon_V}h$ there is an $n>N_{\varepsilon_V}$ and an $x'$ between $X_2$ and $X_2 + h$ such that \[ x' + nh = x, \] it follows that if $x > X_2 + N_{\varepsilon_V}$, \[ \left| \frac{f(x)}{\phi(x)} -\frac{f(x'+nh)-f(x')}{\phi(x'+nh)-\phi(x)} \right| < \varepsilon_V, \] and therefore, $\dfrac{f(x)}{\phi(x)}$ is on $V_1(k)$. This means, according to Theorem~\hyperlink{thm26}{26}, that \[ \mathop{L}_{x\doteq\infty} \frac{f(x)}{\phi(x)} = k. \] (2) $k = +\infty$. If the numbers $m_1$ and $m_2$ are the lower end points of $V_1(k)$ and $V_2(k)$, then \[ \frac{f(x'+nh)-f(x')}{\phi(x'+nh)-\phi(x')} > m_2 \quad \text{for} \quad x' > X_2. \] %-----File: 125.png---Folio 113------ If $\varepsilon_V$ is then chosen less than $m_2-m_1$, there will exist a value of $N_{\varepsilon_V}$ such that \[ \varepsilon_1(x', n) < \frac{\varepsilon_V}{2m_2} \qquad \text{and} \qquad \varepsilon_2(x', n) < \frac{\varepsilon_V}{2m_1} \] for all values of $n > N_{\varepsilon_V}$ independently of $x'$ so long as $x'$ is in \correction{$\linterval{X_2}{X_2+h}$}{$\linterval{X_2}{X_2}+h$}. Then, in view of \hyperlink{eq1p111}{(1)}, \[ \frac{f(x'+nh)}{\phi(x'+nh)} > m_2 \left(1-\frac{\varepsilon_V}{2m_2} \right) -\frac{\varepsilon_V}{2m_2} > m_2-\frac{\varepsilon_V}{2} \left(1+\frac{1}{m_2} \right). \] Since there is no loss of generality if $m_2 > +1$, this proves that for $x > X_2 + N_{\varepsilon_V} n$, \[ \frac{f(x)}{\phi(x)} > m_2-\varepsilon_V > m_1, \] and hence $\dfrac{f(x)}{\phi(x)}$ is on $V_1(k)$. (3) $k =-\infty$ is treated in an analogous manner. \end{proof} \begin{ncorollary}[1] If $[n]$ is the set of all positive integers and if \[ \phi(n+1) > \phi(n) \qquad \text{and} \qquad \mathop{L}_{n=\infty} \phi(n) = \infty, \] then if \[ \mathop{L}_{n=\infty}{L} \frac{f(n+1)-f(n)}{\phi(n+1)-\phi(n)} = k, \] it follows that $\displaystyle{\mathop{L}_{n = \infty}} \dfrac{f(n)}{\phi(n)}$ exists and is equal to $k$\correction{.}{} \end{ncorollary} \begin{ncorollary}[2] If $f(x)$ is bounded on every interval, \correction{$\interval{x}{(x+1)}$}{$\interval{x}{(x}+1)$}, and if \[ \mathop{L}_{x=\infty} f(x+1)-f(x) = k, \] then \[ \mathop{L}_{x=\infty} \frac{f(x)}{x} \] exists and is equal to $k$. \end{ncorollary} %-----File: 126.png---Folio 114------ \section{Rank of Infinitesimals and Infinites.}\hypertarget{chVIsec4}{}%[4] \index{Rank of infinitesimals and infinites} \begin{definition} If on some $V^*(a)$ neither $f_1(x)$ nor $f_2(x)$ vanishes, and $\displaystyle\left|\frac{f_1(x)}{f_2(x)} \right|$ and $\displaystyle\left|\frac{f_2(x)}{f_1(x)} \right|$ are both bounded as $x$ approaches $a$, then $f_1(x)$ and $f_2(x)$ are of the same \textit{rank} whether $\displaystyle{\mathop{L}_{x \doteq a}} \frac{f_1(x)}{f_2(x)}$ exists or not.\footnote{% $x$ and $x \cdot (\sin\dfrac1{x}+2)$ are of the same rank but not of the same order as $x$ approaches zero.} \end{definition} The following theorem is obvious. \begin{theorem}[70]\hypertarget{thm70}{} If $f_1(x)$ and $f_2(x)$ are of the same order, they are of the same rank, and if $f_1(x)$ and $f_2(x)$ are of different orders, they are not of the same rank. If $f_1(x)$ and $f_2(x)$ are of the same rank, they may or may not be of the same order. \end{theorem} \begin{theorem}[71]\hypertarget{thm71}{} If $f_1(x)$ and $f_2(x)$ are of the same rank as $x$ approaches $a$, then $c\cdot f_1(x)$ and $f_2(x)$ are of the same rank, $c$ being any constant not zero. \end{theorem} \begin{proof} By hypothesis for some positive number $M$, \begin{gather*} \left|\frac{f_1(x)}{f_2(x)} \right|< M \text{ and } \left|\frac{f_2(x)}{f_1(x)} \right|< M,\\ \intertext{hence} \left|\frac{c \cdot f_1(x)}{f_2(x)} \right|< M \cdot|c| \text{ and } \left|\frac{f_2(x)}{c \cdot f_1(x)} \right|< \frac{M}{|c|}.\qedhere \end{gather*} \end{proof} \begin{theorem}[72]\hypertarget{thm72}{} If $f_1(x)$ and $f_2(x)$ are of the same rank and $f_2(x)$ and $f_3(x)$ are of the same rank as $x$ approaches $a$, then $f_1(x)$ and $f_3(x)$ are of the same rank as $x$ approaches $a$. \end{theorem} \begin{proof} By hypothesis, \[ \left|\frac{f_1(x)}{f_2(x)} \right|< M_1 \text{ and } \left|\frac{f_2(x)}{f_3(x)} \right|< M_2 \] in some neighborhood of $x=a$. Therefore \[ \left|\frac{f_1(x)}{f_2(x)} \right|\cdot \left|\frac{f_2(x)}{f_3(x)} \right|< M_1 \cdot M_2 \text{ or } \left|\frac{f_1(x)}{f_3(x)} \right|< M_1 \cdot M_2.\qedhere \] %-----File: 127.png---Folio 115------ In the same manner \[ \left|\frac{f_2(x)}{f_1(x)} \right|< M_1 \text{ and } \left|\frac{f_3(x)}{f_2(x)} \right|< M_2, \text{ whence } \left|\frac{f_3(x)}{f_1(x)} \right|< M_1 \cdot M_2. \] \end{proof} \begin{theorem}[73]\hypertarget{thm73}{} If $f_1(x)$ is infinitesimal (infinite) and does not vanish on some $V^*(a)$, and if $f_2(x)$ and $f_3(x)$ are infinitesimal (infinite) of the same rank as $x$ approaches $a$, then $f_1(x) \cdot f_2(x)$ is of higher order than $f_3(x)$, and $f_1(x) \cdot f_3(x)$ is of higher order than $f_2(x)$. Conversely, if for every function, $f_1(x)$, infinitesimal (infinite) at $a$, $f_1(x) \cdot f_2(x)$ is of higher order than $f_3(x)$, and $f_1(x) \cdot f_3(x)$ is of higher order than $f_2(x)$, then $f_2(x)$ and $f_3(x)$ are of the same rank. \end{theorem} \begin{proof} Since $\displaystyle\left|\frac{f_1(x)}{f_3(x)} \right|$ is bounded as $x$ approaches $a$, it follows by Theorem~\hyperlink{thm33}{33} that \[ \mathop{L}_{x\doteq a} \frac{f_1(x)\cdot f_2(x)}{f_3(x)} = 0, \] which proves the first part of the theorem. Since likewise $\displaystyle\left|\frac{f_3(x)}{f_2(x)} \right|$ is bounded, we have that \[ \mathop{L}_{x\doteq a} \frac{f_1(x)\cdot f_3(x)}{f_2(x)} = 0. \] Suppose that for every $f_1(x)$ \[ \mathop{L}_{x\doteq a} \frac{f_1(x)\cdot f_2(x)}{f_3(x)} = 0 \text{ and } \mathop{L}_{x\doteq a} \frac{f_1(x)\cdot f_3(x)}{f_2(x)} = 0, \] and that $f_2(x)$ and $f_3(x)$ are not of the same rank. Then, on a certain subset $[x']$, $\displaystyle \mathop{L}_{x\doteq a} \frac{f_2(x')}{f_3(x')} = 0$, or on some other subset $[x'']$, $\displaystyle \mathop{L}_{x\doteq a} \frac{f_3(x'')}{f_2(x'')} = 0$. Let $f_1(x) = \dfrac{f_2(x)}{f_3(x)}$ on the set $[x']$ for which $\displaystyle \mathop{L}_{x\doteq a} \frac{f_2(x)}{f_3(x)} = 0$, and $x-a$ on the other points of the continuum; %-----File: 128.png---Folio 116------ then $f_1(x)$ is an infinitesimal as $x$ approaches $a$, while for the set $[x']$ \[ \mathop{L}_{x \doteq a} \frac{f_1{(x')} \cdot f_3(x')}{f_3(x')} = \mathop{L}_{x \doteq a} \frac{f_2(x')}{f_3(x')} \cdot \frac{f_3(x')}{f_2(x')} = 1, \] which contradicts the hypothesis that \[ \mathop{L}_{x \doteq a} \frac{f_1(x) \cdot f_3(x)}{f_2(x)} = 0. \] Similarly if on a certain subset $\displaystyle{\mathop{L}_{x \doteq a}} \dfrac{f_3(x)}{f_2(x)} = 0$, we obtain a contradiction by putting $f_1(x) = \dfrac{f_3(x)}{f_2(x)}$. \end{proof} %-----File: 129.png---Folio 117------ \chapter{DERIVATIVES AND DIFFERENTIALS.}\hypertarget{chapVII}{}%[VII] \section{Definition and Illustration of Derivatives.}\hypertarget{chVIIsec1}{}%[1] \begin{definition}\index{Derivative} If the ratio $\frac{f(x)-f(x_1)}{x-x_1}$ approaches a definite limit, finite or infinite, as $x$ approaches $x_1$, the \textit{derivative} of $f(x)$ at the point $x_1$ is the limit \[ \mathop{L}_{x \doteq x_1} \frac{f(x)-f(x_1)}{x-x_1}. \] \end{definition} \begin{figure}[!hbtp]\label{fig14}\hypertarget{fig14}{} \centering \setlength{\unitlength}{0.15\textwidth} \begin{picture}(6,4)(0,0) \put(0,0.25){\line(1,0){6}} \put(0,0.25){\line(0,1){3.5}} \qbezier(1,1.25)(1.5,2)(2,2.5) \qbezier(2,2.5)(2.5,3)(3,3.25) \qbezier(3,3.25)(4,3.75)(5,2.5) \path(2,0.25)(2,2.5)(3,3.25)(3,0.25) \path(2,2.5)(3,2.5) \put(2,0.23){\makebox(0,0)[tc]{$x_1$}} \put(3,0.23){\makebox(0,0)[tc]{$x$}} \put(1.9,2.5){\makebox(0,0)[br]{$A$}} \put(2.9,3.25){\makebox(0,0)[br]{$B$}} \put(3.1,3.25){\makebox(0,0)[tl]{$f(x)$}} \put(3.1,2.5){\makebox(0,0)[lc]{$f(x_1)$}} \put(3,0){\makebox(0,0)[tc]{\sc Fig.~14.}} \end{picture} \end{figure} It is implied that the function $f(x)$ is a single-valued function of $x$. $x-x_1$ is sometimes denoted by $\Delta x_1$, and $f(x)-f(x_1)$ by $\Delta f(x_1)$, or, if $y=f(x)$, by $\Delta y_1$. An obvious illustration of a derivative occurs in Cartesian geometry when the function is represented by a graph (Fig.~\hyperlink{fig14}{14}). %-----File: 130.png---Folio 118------ $\dfrac{f(x)-f(x_1)}{x-x_1}$ is the slope of the line $AB$. If we suppose that the line $AB$ approaches a fixed direction (which in this figure would obviously be the case) as $x$ approaches $x_1$, then $\displaystyle\mathop{L}_{x \doteq x_1} \frac{f(x)-f(x_1)}{x-x_1}$ will exist and will be equal to the slope of the limiting position of $AB$. If the point $x$ were taken only on one side of $x_1$, we should have two similar limiting processes. It is quite conceivable, however, that limits should exist on each side, but that they should differ. That case occurs if the graph has a cusp as in Fig.~\hyperlink{fig15}{15}. \begin{figure}[!hbtp]\label{fig15}\hypertarget{fig15}{} \centering \setlength{\unitlength}{0.15\textwidth} \begin{picture}(5,3)(0,-0.5) \put(0,0){\line(1,0){5}} \put(2,0){\line(0,1){2.5}} \qbezier(1,1)(1.85,1.55)(2,2.5) \qbezier(2,2.5)(2.5,1.5)(3,1.25) \qbezier(3,1.25)(4,0.75)(4.5,0.6) \path(1,0)(1,1)(2,1)(2,1.25)(3,1.25)(3,0) \put(1,-0.1){\makebox(0,0)[tc]{$x$}} \put(2,-0.1){\makebox(0,0)[tc]{$x_1$}} \put(3,-0.1){\makebox(0,0)[tc]{$x$}} \put(1,1){\makebox(0,0)[br]{$B$}} \put(2,2.5){\makebox(0,0)[cb]{$A$}} \put(3,1.25){\makebox(0,0)[lb]{$B$}} \put(2.1,1){\makebox(0,0)[lc]{$f(x)$}} \put(1.9,1.25){\makebox(0,0)[rc]{$f(x)$}} \put(2.1,2.5){\makebox(0,0)[cl]{$f(x_1)$}} \put(2.5,-0.5){\makebox(0,0)[bc]{\sc Fig.~15.}} \end{picture} \end{figure} These\index{Derivative!progressive and regressive}\index{Progressive derivative}\index{Regressive derivative} two cases are distinguished by the terms progressive and regressive derivatives. When the independent variable approaches its limit from below we speak of the progressive derivative, and when from above we speak of the regressive derivative. It follows from the definition of derivative that, except in one singular case, it exists only when both these limits exist and are equal. The exception is the case of a derivative of a function at an end-point of an interval upon which the function is defined. Obviously both the progressive and the regressive derivative cannot exist at such a point. In %-----File: 131.png---Folio 119------ this case we say the derivative exists if either the progressive or the regressive derivative exists. Whether the progressive and regressive derivatives exist or not, there exist always four so-called derived numbers (which may be $\pm\infty$), namely, the upper and lower bounds of indetermination of \[ \frac{f(x)-f(x_1)}{x-x_1}, \] as $x \doteq x_1$ from the right or from the left. (Compare page~\pageref{limp84}, Chapter~\hyperlink{chapIV}{IV}.) The derived numbers are denoted by the symbols. \[ \overrightarrow{D},\ \underrightarrow{D},\ \overleftarrow{D},\ \underleftarrow{D}, \] analogous to the symbols on page~\pageref{limp84}. Of course, in every case, \[ \overrightarrow{D}\geqq \underrightarrow{D} \text{ and } \overleftarrow{D} \geqq \underleftarrow{D}. \] If we consider the curve representing the function \[ y=x \cdot \sin \frac1x \] at the point $x=0$, it is apparent that the limiting position of $AB$ does not exist, although the function is continuous at the point $x=0$ if defined as zero for $x=0$. For at every maximum and minimum of the curve $\sin\dfrac{1}{x}$, $x \cdot \sin\dfrac{1}{x} = \pm x$, and the curve touches the lines $x=y$ and $x=-y$. That is, $\dfrac{f(x)-f(x_1)}{x-x_1}$ approaches every value between $1$ and $-1$ inclusive, as $x$ approaches zero. The notion \textit{derivative} is fundamental in physics as well as in geometry. If, for instance, we consider the motion of a body, we may represent its distance from a fixed point as a function of time, $f(t)$. At a certain instant of time $t_1$ its distance from the fixed point is $f(t_1)$, and at another instant $t_2$ it is $f(t_2)$; then \[ \frac{f(t_1)-f(t_2)}{t_1-t_2} \] %-----File: 132.png---Folio 120------ is the average velocity of the body during the interval of time $t_1-t_2$ in a direction from or toward the assumed fixed point. Whether the motion be from or toward the fixed point is of course indicated by the sign of the expression $\dfrac{f(t_1)-f(t_2)}{t_1-t_2}$. If we consider this ratio as the time interval is taken shorter and shorter, that is, as $t_2$ approaches $t_1$, it will in ordinary physical motion approach a perfectly definite limit. This limit is spoken of as the velocity of the body at the instant $t_1$. \begin{definition}\index{Derived function} The derivative of a function $y=f(x)$ is denoted by $f'(x)$ or by $D_xf(x)$ or $\dfrac{df(x)}{dx}$ or $\dfrac{dy}{dx}$. $f'(x)$ is also referred to as the \emph{derived function} of $f(x)$. \end{definition} \section{Formulas of Differentiation.}\hypertarget{chVIIsec2}{}%[2] \begin{theorem}[74]\hypertarget{thm74}{} The derivative of a constant is zero. More precisely: If there exists a neighborhood of $x_1$ such that for every value of $x$ on this neighborhood $f(x) =f(x_1)$, then $f'(x_1) =0$. \end{theorem} \begin{proof} In the neighborhood specified $\dfrac{f(x)-f(x_1)}{x-x_1}=0$ for every value of $x$. \end{proof} \begin{corollary} If $f'(x_1)$ exists and if in every $V^*(x_1)$ there is a value of $x$ such that $f(x) =f(x_1)$, then $f'(x_1) = 0$. \end{corollary} \begin{theorem}[75]\hypertarget{thm75}{} When for two functions $f_1(x)$ and $f_2(x)$ the derived functions $f_1'(x)$ and $f_2'(x)$ exist at $x_1$ it follows that, except in the indeterminate case $\infty-\infty$, \begin{enumerate} \item[\textnormal{(\textit{a})}] If $f_3(x) = f_1(x) + f_2(x)$, then $f_3(x)$ has a derivative at $x_1$ and \[ f_3'(x_1) =f_1'(x_1) + f_2'(x_1). \] \item[\textnormal{(\textit{b})}] If $f_3(x) = f_1(x) \cdot f_2(x)$, then $f_3(x)$ has a derivative at $x_1$ and \[ f_3'(x_1) = f_1'(x_1) \cdot f_2(x_1) + f_1(x_1) \cdot f_2'(x_1). \] \item[\textnormal{(\textit{c})}]If $f_3(x) = \dfrac{f_1(x)}{f_2(x)}$, then, provided there is a $V(x_1)$ upon which $f_2(x) \neq 0$, $f_3(x)$ has a derivative and \[ f_3'(x_1) = \frac{f_1'(x_1) \cdot f_2(x_1)-f_1(x_1) \cdot f_2'(x_1)} {\{f_2(x_1)\}^2}. \] \end{enumerate} \end{theorem} %-----File: 133.png---Folio 121------ \begin{proof} By definition and the theorems of Chapter~\hyperlink{chapIV}{IV} (which exclude the case $\infty-\infty$), \begin{enumerate} \item[(\textit a)] \begin{align*} f_1'(x_1) + f_2'(x_1) &= \mathop{L}_{x\doteq x_1} \frac{f_1(x)-f_1(x_1)}{x-x_1} + \mathop{L}_{x\doteq x_1} \frac{f_2(x)-f_2(x_1)}{x-x_1} \tag{1} \\ &= \mathop{L}_{x\doteq x_1} \left\{ \frac{f_1(x)-f_1(x_1)}{x-x_1} + \frac{f_2(x)-f_2(x_1)}{x-x_1} \right\} \tag{2} \\ &= \mathop{L}_{x\doteq x_1} \frac{f_1(x)+f_2(x)-f_1(x_1)-f_2(x_1)}{x-x_1} \tag{3} \\ &= \mathop{L}_{x\doteq x_1} \frac{f_3(x)-f_3(x_1)}{x-x_1}. \end{align*} But by definition, \[ f_3'(x_1) = \mathop{L}_{x\doteq x_1} \frac{f_3(x)-f_3(x_1)}{x-x_1}. \tag{4} \] Hence $f_3'(x_1)$ exists, and $f_3'(x_1) =f_1'(x_1) +f_2'(x_1)$. \item[(\textit b)] $f_3(x)=f_1(x)\cdot f_2(x)$.\\ Whenever $x\neq x_1$ we have the identity \begin{align*} &\frac{f_3(x)-f_3(x_1)}{x-x_1} = \frac{f_1(x)\cdot f_2(x)-f_1(x_1)\cdot f_2(x_1)}{x-x_1} \\ =\,&\frac{f_1(x )\cdot f_2(x)-f_1(x_1)\cdot f_2(x ) + f_1(x_1)\cdot f_2(x)-f_1(x_1)\cdot f_2(x_1) }{x-x_1} \\ =\, &f_2(x) \left\{ \frac{f_1(x)-f_1(x_1)}{x-x_1} \right\} + f_1(x) \left\{ \frac{f_2(x)-f_2(x_1)}{x-x_1} \right\}. \end{align*} But the limit of the last expression exists as $x\doteq x_1$ (except perhaps in the case $\infty-\infty$) and is equal to \[ f_2(x_1)\cdot f_1'(x_1) + f_1(x_1)\cdot f_2'(x_1). \] %-----File: 134.png---Folio 122------ Hence \[ \mathop{L}_{x \doteq x_1} \frac{f_3(x)-f_3(x_1)}{x-x_1} \] exists and \[ f_3'(x_1) = f_2(x_1)\cdot f_1'(x_1) + f_2'(x_1) \cdot f_1(x_1).\] \item[(\textit c)] \[ f_3(x)= \frac {f_1(x)}{f_2(x)}. \] The argument is based on the identity \[ \frac{\frac{f_1(x)}{f_2(x)}-\frac{f_1(x_1)}{f_2(x_1)} }{x-x_1} = \frac{f_1(x) \cdot f_2(x_1)-f_2(x) \cdot f_1(x_1) } { f_2(x) \cdot f_2(x_1) \cdot (x-x_1) }, \] which holds when $x \neq x_1$ and when $f_2(x) \neq 0$. But \begin{align*} &\frac{f_1(x)\cdot f_2(x_1)-f_2(x) \cdot f_1(x_1)} {f_2(x)\cdot f_2(x_1) (x-x_1)} \\ &= \frac{f_1(x )\cdot f_2(x_1)-f_1(x_1)\cdot f_2(x_1) + f_1(x_1)\cdot f_2(x_1)-f_2(x )\cdot f_1(x_1)} {f_2(x)\cdot f_2(x_1) (x-x_1)} \\ &= \frac{f_2(x_1) \left\{f_1(x)-f_1(x_1) \right\} -f_1(x_1) \left\{ f_2(x)-f_2(x_1) \right\}} {f_2(x)\cdot f_2(x_1) (x-x_1)}. \end{align*} As before (excluding the case $\infty-\infty$) we have \[ f_3'(x_1) = \frac{f_2(x_1)\cdot f_1'(x_1)-f_2'(x_1) \cdot f_1(x_1)} {\left\{ f_2(x_1) \right\}^2 }\text{\correction{.}{,}} \] \end{enumerate} \end{proof} \begin{corollary} It follows from Theorems \hyperlink{thm74}{74} and \hyperlink{thm75}{75} of this chapter that if $f_2(x)=a\cdot f_1(x)$ where $f_1'(x)$ exists, then \[ f_2'(x)=a\cdot f_1'(x). \] \end{corollary} \begin{theorem}[76]\hypertarget{thm76}{}\label{p122t76} If $x>0$, then $\dfrac{d}{dx}x^k =k\cdot x^{k-1}$. \end{theorem} %-----File: 135.png---Folio 123------ \label{p123} \begin{enumerate} \item[(\textit{a})] If $k$ is a positive integer, we have \begin{align*} \mathop{L}_{x\doteq x_1} \frac{x^k-x_1^k}{x-x_1} &= \mathop{L}_{x\doteq x_1} \bigl\{ x^{k-1} + x^{k-2}\cdot x_1 + \ldots + x^k\cdot x_1^{k-2}+x_1^{k-1} \bigr\} \\ &= k\cdot x_1^{k-1}. \end{align*} \item[(\textit{b})] If $k$ is a positive rational fraction $\dfrac{m}{n}$, we have \begin{gather*} \mathop{L}_{x\doteq x_1} \frac{x^{\frac mn}-{x_1}^{\frac mn}}{x-x_1} = \mathop{L}_{x\doteq x_1} \frac{\bigl(x^{\frac1n}\bigr)^m -\bigl({x_1}^{\frac1n}\bigr)^m} {\bigl(x^{\frac1n}\bigr)^n -\bigl({x_1}^{\frac1n}\bigr)^n} \\ = \mathop{L}_{x\doteq x_1} \frac{1}{\bigl(x^{\frac1n}\bigr)^{n-1} + \bigl(x^{\frac1n}\bigr)^{n-2}\cdot \bigl({x_1}^{\frac1n}\bigr) + \ldots + \bigl({x_1}^{\frac1n}\bigr)^{n-1}} \cdot \frac{\bigl(x^{\frac1n}\bigr)^m -\bigl({x_1}^{\frac1n}\bigr)^m} {x^{\frac1n}-{x_1}^{\frac1n}} \\ = \frac{1}{n\cdot \bigl({x_1}^{\frac1n}\bigr)^{n-1}} \cdot m \bigl({x_1}^{\frac1n}\bigr)^{m-1}, \end{gather*} by the preceding case.\\ But \[ \frac{1}{n\cdot \bigl({x_1}^{\frac1n}\bigr)^{n-1}} \cdot m \bigl({x_1}^{\frac1n}\bigr)^{m-1} = \frac mn{x_1}^{\frac mn-1} = k\cdot {x_1}^{k-1}. \] \item[(\textit{c})] If $k$ is a negative rational number and equal to $-m$, then, by the two preceding cases, \begin{align*} \mathop{L}_{x\doteq x_1} \frac{x^{-m}-{x_1}^{-m}}{x-x_1} =-\mathop{L}_{x\doteq x_1} \cdot \frac{1}{x^m \cdot x_1^m} \cdot \frac{x^m-x_1^m}{x-x_1} &=-\frac{1}{x_1^{2m}} \cdot mx_1^{m-1} \\ &=-m{x_1}^{-m-1}\text{\correction{.}{}} \end{align*} But \[ -m{x_1}^{-m-1} = k\cdot x^{k-1}. \] \item[(\textit{d})] If $k$ is a positive irrational number, we proceed as follows: %-----File: 136.png---Folio 124------ Consider values of $x$ greater than or equal to unity. Let $x$ approach $x_1$ so that $x>x_1$. Since, by Theorem~\hyperlink{thm23}{23}, $x^k$ is a monotonic increasing function of $k$ for $x > 1$, it follows that \[ \frac{x^k-x_1^k}{x-x_1} = x_1^k \cdot \frac{\left(\dfrac{x}{x_1}\right)^k-1}{x-x_1} > x_1^{k'} \cdot \frac{\left(\dfrac{x}{x_1}\right)^{k'}-1}{x-x_1} \] for all values of $k'$ less than $k$, and all values of $x$ greater than $x_1$. If $k'$ is a rational number, we have by the preceding cases that \[ \mathop{L}_{x\doteq x_1} x_1^{k'} \cdot \frac{\left(\dfrac{x}{x_1}\right)^{k'}-1}{x-x_1} = k'x_1^{k'-1}. \] Since $x_1^{k-1}$ is a continuous function of $k$, it follows that for every number $N$ less than $kx_1^{k-1}$ there exists a rational number $k_1'$ less than $k$ such that \[ N < k_1'\cdot x_1^{k'-1} < k\cdot x_1^{k-1}. \] Hence, by Theorem~\hyperlink{thm40}{40}, \[ x_1^k \cdot \frac{\left(\dfrac{x}{x_1}\right)^k-1}{x-x_1} \] cannot approach a value $N$ less than $kx_1^{k-1}$ as $x$ approaches $x_1$. By a precisely similar argument we show that a number greater than $kx_1^{k-1}$ cannot be a value approached. Since there is always at least one value approached, we have that \[ \mathop{L}_{x\doteq x_1} \frac{x^k-x_1^k}{x-x_1} = k\cdot x_1^{k-1}. \] If $x x_1$ \correction{and}{\textit{and}} $f(x_3) < f(x_1)$. \end{enumerate} Similarly we define a \index{Minimum of a function}\textit{minimum} of a function. \end{definition} This definition allows any point of a constant stretch like $a$, Fig.~\hyperlink{fig17}{17}, to be a maximum, but does not allow any point of $b$ to be either a maximum or a minimum. \begin{figure}[!hbtp]\label{fig17}\hypertarget{fig17}{} \centering \setlength{\unitlength}{0.08\textwidth} \begin{picture}(10,7)(0,-0.5) \path(0,6.5)(0,0)(10,0) \path(0.25,2.5)(1.5,3.5)(3.5,3.5)(5,2) \path(5.25,2.25)(6,3.5)(8.5,3.5)(9.5,5.75) \put(2.5,3.55){\makebox(0,0)[bc]{$a$}} \put(7.25,3.55){\makebox(0,0)[bc]{$b$}} \put(5,-0.5){\makebox(0,0)[bc]{\sc Fig.~17.}} \end{picture} \end{figure} \begin{theorem}[83]\hypertarget{thm83}{}\label{t83p131} If $f'(x_1)$ exists and if $f(x)$ has a maximum or a minimum at $x = x_1$, then $f'(x_1) = 0$. \end{theorem} \begin{proof} In case of a maximum at $x_1$, it follows directly from the hypothesis that \[ \mathop{L}_{\stackrel{x\doteq x_1}{x > x_1}} \frac{f(x)-f(x_1)}{x-x_1} \qqle 0, \text{ and also } \mathop{L}_{\stackrel{x\doteq x_1}{x < x_1}} \frac{f(x)-f(x_1)}{x-x_1} \qqge 0, \] Since $f'(x_1)$ exists these limits are equal, that is, the derivative is equal to zero. Similarly in case of a minimum. \end{proof} \begin{theorem}[84]\index{Rolle's theorem}\hypertarget{thm84}{} If $f(x_1) = f(x_2)$, $f(x)$ being continuous on the %-----File: 144.png---Folio 132------ interval $\interval{x_1}{x_2}$, and if the derivative exists\footnote{% Not necessarily finite.} at every point between $x_1$ and $x_2$, then there is a value $\xi$ between $x_1$ and $x_2$ such that $f'(\xi) =0$. The derivative need not exist at $x_1$ and $x_2$. \end{theorem} \begin{proof} \begin{enumerate} \item[(\textit{a})] The function may be a constant between $x_1$ and $x_2$, in which case $f'(x)=0$ for all values of $x$ between $x_1$ and $x_2$ by Theorem~\hyperlink{thm74}{74}. \item[(\textit b)] There may be values of the function between $x_1$ and $x_2$ which are greater than $f(x_1)$ and $f(x_2)$. Since the function is continuous on the interval $\interval{x_1}{x_2}$, it reaches a least upper bound on this interval at some point $x_3$ (different from $x_1$ and $x_2$). By Theorem~\hyperlink{thm83}{83}, \[ f'(x_3)=0. \] \item[(\textit{c})] In case there are values of the function on the interval $\interval{x_1}{x_2}$ less than $f(x_1)$, the derivative is zero at the minimum point in precisely the same manner as under case (\textit{b}). \end{enumerate} \end{proof} \begin{figure}[!hbtp]\label{fig18}\hypertarget{fig18}{} \centering \setlength{\unitlength}{0.06\textwidth} \begin{picture}(7,6.5)(-1,-1.5) \put(-1,0){\line(1,0){7}} \put(0,0){\line(0,1){5}} \put(5,0){\line(0,1){4}} \put(0,5){\line(5,-1){5}} \put(1.5,3){\line(5,-1){3}} \dashline{0.1}(3,0)(3,2.7) \qbezier(0,5)(1.5,3)(3,2.7) \qbezier(3,2.7)(4,2.5)(5,4) \put(0,-0.25){\makebox(0,0)[tc]{$x_1$}} \put(0,5.25){\makebox(0,0)[bc]{$A$}} \put(3,-0.25){\makebox(0,0)[tc]{$\xi$}} \put(5,-0.25){\makebox(0,0)[tc]{$x_2$}} \put(5,4.25){\makebox(0,0)[bc]{$B$}} \put(2.5,-1){\makebox(0,0)[tc]{\textsc{Fig.~18.}}} \end{picture} \end{figure} This theorem is called \textsc{Rolle's} Theorem. The restriction that $f(x)$ shall be continuous is unnecessary if the derivative %-----File: 145.png---Folio 133------ exists, but simplifies the argument. The proof without this restriction is suggested as an exercise for the reader. The geometric interpretation is that any curve representing a continuous function, $f(x)$, such that $f(x_1) = f(x_2)$, and having a tangent at every point \correction{between}{betweeen} $x_1$ and $x_2$ has a horizontal tangent at some point between them. An immediate generalization of this is that between any two points $A$ and $B$ on a curve which satisfies the hypothesis of this theorem there is a tangent to the curve which is parallel to the line $AB$. The following theorem is a corresponding analytical generalization: \begin{theorem}[85]\index{Mean-value theorem!of the differential calculus} \hypertarget{thm85}{}If $f(x)$ is continuous on the interval $\interval{x_1}{x_2}$, and if the derivative exists at every point between $x_1$ and $x_2$, then there is a value of $x$, $x = \xi$, between $x_1$ and $x_2$ such that \[ f'(\xi) = \frac{f(x_1)-f(x_2)}{x_1-x_2}. \] \end{theorem} \begin{proof} Consider a function $f_1(x)$ such that \[ f_1(x) = f(x)-(x-x_2)\cdot \frac{f(x_1)-f(x_2)}{x_1-x_2}; \] then $f_1(x_1)= f(x_2)$ and $f_1(x_2) = f(x_2)$. Therefore $f_1(x_1) = f_1(x_2)$. Hence, by Theorem~\hyperlink{thm84}{84}, there is an $x$, $x = \xi$ on the segment $\overline{x_1\ x_2}$ such that $f_1'(\xi) = 0$. That is, \[ f_1'(\xi) = f'(\xi)-\frac{f(x_1)-f(x_2)}{x_1-x_2} = 0. \] Therefore \[ f'(\xi) = \frac{f(x_1)-f(x_2)}{x_1-x_2}.\qedhere \] \end{proof} This is the ``mean-value theorem.'' Its content may also be expressed by the equation \[ f(x_2) = f(x_1) + (x_2-x_1) f'(\xi). \] %-----File: 146.png---Folio 134------ Denoting $x_1-x$ by $dx$ and $\xi$ by $x+\theta dx$, where $0<\theta<1$, it takes the form \[ f(x_1 +dx) = f(x_1) + f'(x_1+\theta dx)dx. \] \begin{theorem}[86]\hypertarget{thm86}{} If $f_1(x)$ and $f_2(x)$ are continuous on an interval $\interval{a}{b}$, and if $f_1'(x)$ and $f_2'(x)$ exist between $a$ and $b$, $f_2'(x)\neq\pm\infty$, and $f_2'(x)\neq 0$, $f_2(a)\neq f_2(b)$, then there is a value of $x$, $x=\xi$ between $a$ and $b$ such that \[ \frac{f_1(a)-f_1(b)}{f_2(a)-f_2(b)} = \frac{f_1'(\xi)}{f_2'(\xi)}. \] \end{theorem} \begin{proof} Consider a function \[ f_3(x)= \frac{f_1(a)-f_1(b)}{f_2(a)-f_2(b)} \{ f_2(x)-f_2(b) \}-\{ f_1(x)-f_1(b) \}. \] Since $f_3(a)=0$ and $f_3(b)=0$, we have as before $f_3'(\xi)=0$.\\ But \[ f_3'(\xi)= \frac{f_1(a)-f_1(b)}{f_2(a)-f_2(b)} \cdot f'_2(\xi)-f'_1(\xi). \] Therefore \[ \frac{f_1(a)-f_1(b)}{f_2(a)-f_2(b)} = \frac{f_1'(\xi)}{f_2'(\xi)}.\qedhere \] \end{proof} This is called the second mean-value theorem. The first mean-value theorem has a very important extension to ``Taylor's series with a remainder,'' which follows as Theorem~\hyperlink{thm87}{87}. \section{Taylor's Series.}\hypertarget{chVIIsec5}{}%[5] \index{Taylor's series}\index{Series!Taylor's} The derivative of $f'(x)$ is denoted by $f''(x)$ and is called the second \correction{derivative}{derviative} of $f(x)$. In general the $n$th derivative is the derivative of the $n-1$st derivative and is denoted by $f^{(n)}(x)$. \begin{theorem}[87]\hypertarget{thm87}{} If the first $n$ derivatives of the function $f(x)$ exist and are finite upon the interval $\interval{a}{b}$, there is a value of $x$, $x_n$ on the interval $\interval{a}{b}$ such that %-----File: 147.png---Folio 135------ \begin{multline*} f(b) = f(a) + \frac{(b-a)}{1!} f'(a) + \frac{(b-a)^2}{2!} f''(a) + \ldots \\ + \frac{(b-a)^{n-1}}{(n-1)!}\cdot f^{(n-1)}(a) + \frac{(b-a)^n}{n!} f^{(n)}(x_n). \end{multline*} \end{theorem} \begin{proof} Let $R_n$ be a constant such that \begin{multline*} F(x) = f(x)-f(a)-(x-a)f'(a)-\frac{(x-a)^2}{2!}f''(a)-\ldots \\ -\frac{(x-a)^{n-1}}{(n-1)!}f^{(n-1)}(a)-\frac{(x-a)^n}{n!}R_n \end{multline*} is equal to zero for $x=b$. Since $F(x)=0$ for $x=a$, there is, by Theorem~\hyperlink{thm84}{84}, some value of $x$, $x_1$, $a0$. \end{enumerate} \end{theorem} \begin{proof} By Taylor's theorem, for every $x$ in the vicinity of \correction{$a$}{a} \[ f(x)=f(a) + (x-a)^nf^{(n)}(a)+(x-a)^{n+1}\cdot f^{(n+1)}(\xi_x), \] where $\xi_x$ is between $x$ and $a$. Hence \[ f(x)-f(a) = (x-a)^n\{f^{(n)}(a)+(x-a)f^{(n+1)}(\xi_x)\}. \] But since $f^{(n+1)}(\xi_x)$ is bounded and $x-a$ is infinitesimal, there exists a $\overline{V^*}(a)$ such that if $x$ is in $\overline{V^*}(a)$, \[ f(x)-f(a) \] is positive or negative according as \[ (x-a)^n\cdot f^{(n)}(a) \] is positive or negative. \begin{enumerate} \item[(1)] If $n$ is odd, $(x-a)^n$ is of the same sign as $x-a$, and hence for $f^{(n)}(a)>0$ \begin{gather*} f(x)-f(a)>0 \quad \text{if } x>a,\\ f(x)-f(a)<0 \quad \text{if } x0 \quad \text{if } xa. \end{gather*} \item[(2)] If $n$ is even, $(x-a)^n$ is always positive, and hence if $f^{(n)}(a) >0$, \[ \left. \begin{aligned} f(x)-f(a)>0 \quad &\text{if } x>a,\\ f(x)-f(a)>0 \quad &\text{if } xa, \\ f(x)-f(a)<0 &\quad \text{if } xa$. Consider only such values of $x$. Then if \begin{align*} z=\frac{1}{x-a},\ &f(x)=f(a+\frac1z)=F(z) \\ \intertext{and} &\phi(x) = \phi(a+\frac1z) = \Phi(z), \end{align*} by hypothesis and Theorem~\hyperlink{thm79}{79}, $F'(z)$ and $\Phi'(z)$ exist and \begin{align*} &F'(z) = f'(x)\frac{dx}{dz}, \\ &\Phi'(z)=\phi'(x)\frac{dx}{dz}. \end{align*} Hence if \[ \mathop{L}_{x\doteq a} \frac{f'(x)}{\phi'(x)}=K, \] then, according to Theorem~\hyperlink{thm38}{38}, \[ \mathop{L}_{x\doteq \infty} \frac{F'(z)}{\Phi'(z)} \] exists and is equal to $K$. Hence, by Theorem~\hyperlink{thm90}{90}, \[ \mathop{L}_{x\doteq \infty} \frac{F(z)}{\Phi(z)} \] exists and is equal to $K$. Hence, by Theorem~\hyperlink{thm38}{38}, \[ \mathop{L}_{x\doteq a} \frac{f(x)}{\phi(x)} \] exists and is equal to $K$. \end{proof} We have now derived conditions under which we can state a general rule for computing an indeterminate form. Provided $f(x)$ is not zero on every $V^*(a)$, any of the forms \hyperlink{case3}{(3)} to \hyperlink{case7}{(7)} can be reduced to \hypertarget{a}{\[ \tag{\textit{a}} \frac{F(x)}{\Phi(x)} \]} %-----File: 155.png---Folio 143------ where this is of type~\hyperlink{case1}{(1)} or \hyperlink{case2}{(2)}. Provided $F(x)$ and $\Phi(x)$ satisfy the conditions of Theorem~\hyperlink{thm91}{91}, the existence of the limit of \hyperlink{a}{(a)} depends on the existence of the limit of \hypertarget{b}{\[ \frac{F'(x)}{\Phi'(x)}.\tag{\textit{b}} \]} If \hyperlink{b}{(\textit{b})} is indeterminate, and $F'(x)$ and $\Phi'(x)$ satisfy the conditions of Theorem~\hyperlink{thm91}{91}, the limit of \hyperlink{b}{(\textit{b})} depends on the limit of \[ \frac{F''(x)}{\Phi''(x)},\tag{\textit{c}} \] and so on in general. If at each step the conditions of Theorem~\hyperlink{thm91}{91} are satisfied and the form is still indeterminate, the limit of \[ \frac{F^{(n)}(x)}{\Phi^{(n)}(x)}\tag{$n$} \] depends on the limit of \[ \frac{F^{(n+1)}(x)}{\Phi^{(n+1)}(x)}.\tag{$n+1$} \] If ($n$) is indeterminate for all values of $n$, this rule leads to no result. If for some value of $n$ \[ \mathop{L}_{x\doteq a}\frac{F^{(n)}(x)}{\Phi^{(n)}(x)}=K, \] then all the preceding limits exist and are equal to $K$, and so \[ \mathop{L}_{x\doteq a}\frac{F(x)}{\Phi(x)}=K. \] The original expression is equal to $K$ or $e^K$ according to the case under consideration. %-----File: 156.png---Folio 144------ \section{General Theorems on Derivatives.}\hypertarget{chVIIsec7}{}%[7] \begin{theorem}[92]\hypertarget{thm92}{} If $f(x)$ is continuous and $f'(x)$ exists for every $x$ on an interval $\interval{a}{b}$, then $f'(x)$ takes on every value between any two of its values. \end{theorem} \begin{proof} Consider any two values of $f'(x)$, $f'(x_1)$, and $f'(x_2)$ on the interval $\interval{a}{b}$. Consider, further, the function $\dfrac{f(x)-f(x_1)}{x-x_1}$ on the interval between $x_1$ and $x_2$. Since $\dfrac{f(x)-f(x_1)}{x-x_1}$ is a continuous function of $x$ on this interval, it takes on every value between $\dfrac{f(x_2)-f(x_1)}{x_2-x_1}$ and $f'(x_1)$, which is its limiting value as $x$ approaches $x_1$. Hence, by Theorem~\hyperlink{thm85}{85}, $f'(x)$ takes on all values between and including $f'(x_1)$, and $\dfrac{f(x_2)-f(x_1)}{x_2-x_1}$ for values of $x$ on the interval $\interval{x_1}{x_2}$. By considering in a similar manner the function $\dfrac{f(x_2)-f(x)}{x_2-x}$ on the interval $\interval{x_1}{x_2}$, we show that $f'(x)$ takes on all values between $\dfrac{f(x_2)-f(x_1)}{x_2-x_1}$ and $f'(x_2)$. Hence $f'(x)$ takes on all values between $f'(x_1)$ and $f'(x_2)$. \end{proof} \begin{theorem}[93]\hypertarget{thm93}{} If the derivative exists at every point on an interval, including its end-points, it does not follow that the derivative is continuous or that it takes on its upper and lower bounds. \end{theorem} \begin{proof} This is shown by the following example. The curve shall lie between the $x$-axis and the parabola $y = \frac12x^2$. The straight lines of slopes $1, 1\frac12, 1\frac34,\ldots, 1+\dfrac{2^n-1}{2^n}\ldots$ through the points $(\frac12,0), (\frac14,0),\ldots, \left(\dfrac{1}{2^{n+1}}, 0\right),\ldots$, respectively, meet the parabola in points $A_1, A_2, A_3,\ldots, A_n,\ldots$ The broken line $A_1\ (\frac12,0)$ $A_2\ (\frac14, 0)$ $A_3$ \ldots $A_n\ \left(\dfrac{1}{2^n}, 0\right)\ldots\infty$, has an %-----File: 157.png---Folio 145------ \begin{figure}[!htbp]\label{fig19}\hypertarget{fig19}{} \centering \includegraphics{images/fig19} \end{figure} infinitude of vertices. In each angle of the broken line consider an arc of circle tangent to and terminated by the sides of the angle, the points of tangency being one fourth of the distance to the nearest vertex. The function whose graph consists of these circular arcs and the portions of the broken line between them is continuous and differentiable on the interval $\interval{0}{1}$. Its derivative is discontinuous at $x=0$ and has the least upper bound 2, which is never reached. \end{proof} \begin{theorem}[94]\hypertarget{thm94}{} If $f'(x)$ exists and is equal to zero for every value of $x$ on the interval $\interval{a}{b}$, then $f(x)$ is a constant on that interval. \end{theorem} \begin{proof} By Theorem~\hyperlink{thm82}{82}, $f(x)$ is continuous. Suppose $f(x)$ not a constant, so that for two values of $x$, $x_1$, and $x_2$, $f(x_1) \neq f(x_2)$, then, by Theorem~\hyperlink{thm85}{85}, there is a value of $x$, $x = \xi$ between $x_1$ and $x_2$ such that \[ f'(\xi) = \frac{f(x_2)-f(x_1)}{x_2-x_1}, \] %-----File: 158.png---Folio 146------ which is different from zero, whence $f'(x)$ is not zero for every value of $x$ on the interval $\interval{a}{b}$. Hence $f(x)$ is a constant on $\interval{a}{b}$. \end{proof} \begin{corollary} If $f_1'(x)=f_2'(x)$ and is finite for every value of $x$ on an interval $\interval{a}{b}$, then $f_1(x)-f_2(x)$ is a constant on this interval. \end{corollary} \begin{theorem}[95]\hypertarget{thm95}{} If $f'(x)$ exists and is positive for every value of $x$ on the interval $\interval{a}{b}$, then $f(x)$ is monotonic increasing on this interval. If $f'(x)$ is negative for every value of $x$ on this interval, then $f(x)$ is monotonic decreasing. \end{theorem} \begin{proof} If $f'(x)$ is positive for every value of $x$, then it follows from Theorem~\hyperlink{thm85}{85}, provided that $f(x)$ is continuous, that the function is monotonic increasing, for if there were two values of $x$, $x_1$ and $x_2$, such that $f(x_1) \geqq f(x_2)$ while $x_1 < x_2$, then there would be a value of $x$, $x = \xi $, between $x_1$ and $x_2$ such that \[ f'(\xi)=\frac{f(x_2)-f(x_1)}{x_2-x_1}\leqq 0. \] In case $f(x)$ is not supposed continuous, the argument can be made as follows: If $f'(x_1)>0$, then, by Theorem~\hyperlink{thm23}{23}, there exists about the point $x_1$ a segment \correction{$\overline{(x_1-\delta)\ (x_1 + \delta)}$}{$(x_1-\delta)$, $(x_1 + \delta)$}, upon which \[ \frac{f(x)-f(x_1)}{x-x_1}>0, \] and hence, if $x>x_1$, $f(x) >f(x_1)$ and if $xf(x_1)$, while $xf(x)$, while $x>x_1$, which is contrary to the hypothesis that the function is monotonic increasing in the neighborhood of $x = x_1$. In the same manner we prove that if the function is monotonic decreasing, and if the derivative exists, then $f'(x)$ cannot be positive. \end{proof} The following theorem states necessary and sufficient conditions for the existence of the progressive and regressive derivatives. Conditions for the existence of a derivative proper are obtained by adding the condition that the progressive and regressive derivatives are equal. \begin{theorem}[97]\hypertarget{thm97}{} If $f(x)$, $x 1 + \frac32\pi$. %-----File: 163.png---Folio 151------ \chapter{DEFINITE INTEGRALS.}\hypertarget{chapVIII}{}%[VIII] \section{Definition of the Definite Integral.}\hypertarget{chVIIIsec1}{}%[1] The area of a rectangle the lengths of whose sides are exact multiples of the length of the side of a unit square, is the number of squares equal to the unit square contained within the rectangle, and is easily seen to be equal to the product of the lengths of its base and altitude.\footnote{% Of course the units are not necessarily squares; they may be triangles, parallelograms, etc.} In case the sides of the rectangle and the side of the unit square are commensurable, the sides of the rectangle not being exact multiples of the side of the square, the rectangle and the square are divided into a set of equal squares. The area of the rectangle is then defined as the ratio of the number of squares in the rectangle to be measured to the number of squares in the unit square. Again, the area is equal to the product of the base and altitude. Any figure so related to the unit square that both figures can be divided into a finite set of equal squares is said to be commensurable with the unit. The area of a rectangle incommensurable with the unit is defined as the least upper bound of the areas of all commensurable rectangles contained within it. It follows directly from the definition of the product of irrational numbers that this process gives the area as the product of the base and altitude.\footnote{% For the meaning of the length of a segment incommensurable with the unit segment, compare Chapter~\hyperlink{chapII}{II}, page~\pageref{chIIp33}.} %-----File: 164.png---Folio 152------ Turning to the figure bounded by the segment $\overline{a\ b}$ (which we take on the $x$ axis in a system of rectangular coordinates) the graph of a function $y=f(x)$ and the ordinates $x=a$ and $x=b$, \begin{figure}[!htpb]\label{fig20}\hypertarget{fig20}{} \centering \setlength{\unitlength}{0.05\textwidth} \begin{picture}(20,8.5)(-2,-1.5) \put(-2,0){\line(1,0){20}} \path(0,0)(0,4)(2,4)(2,0) \path(2,4)(2,6)(5,6)(5,0) \dashline{0.25}(1,0)(1,4) \dashline{0.25}(3,0)(3,6) \qbezier(0,2.5)(0.5,3.3)(1,4) \qbezier(1,4)(2,5.4)(3,6) \qbezier(3,6)(4.66,7)(7,7) \qbezier(7,7)(8,7)(9,6) \qbezier(9,6)(12,3)(14,3) \qbezier(14,3)(15,3)(16,3.5) \qbezier(16,3.5)(17,4)(17.5,6.5) \path(14,0)(14,3.5)(17.5,3.5)(17.5,6.5)(17.5,0) \dashline{0.25}(16,0)(16,3.5) \put(0,-0.25){\makebox(0,0)[tc]{$a$}} \put(1,-0.25){\makebox(0,0)[tc]{$\xi_1$}} \put(2,-0.25){\makebox(0,0)[tc]{$x_1$}} \put(3,-0.25){\makebox(0,0)[tc]{$\xi_2$}} \put(5,-0.25){\makebox(0,0)[tc]{$x_2$}} \put(14,-0.25){\makebox(0,0)[tc]{$x_{n-1}$}} \put(16,-0.25){\makebox(0,0)[tc]{$\xi_n$}} \put(17.5,-0.25){\makebox(0,0)[tc]{$b$}} \put(8,-1.5){\makebox(0,0)[bc]{\sc Fig.~20}} \end{picture} \end{figure} we obtain as follows an approximation to the common notion of the area of such figures. Let $x_0=a$, $x_1$, $x_2$, $\ldots$, $x_n=b$ be a set of points lying in order from $a$ to $b$. Such a set of points is called a partition of $\interval{a}{b}$, and is denoted by $\pi$. The intervals $\interval{x_0}{x_1}$, $\interval{x_1}{x_2}$, $\ldots$, $\interval{x_{n-1}}{x_n}$ are intervals of $\pi$. Let $x_1-x_0=\Delta_1x$, $x_2-x_1=\Delta_2x$, $\ldots$, $x_n-x_{n-1}=\Delta_nx$, and let \[ \xi_1,\ \xi_2, \ldots,\ \xi_n \] be a set of points such that $\xi_1$ is on the interval $\interval{x_0}{x_1}$, $\xi_2$ is on $\interval{x_1}{x_2} \ldots$, and $\xi_n$ is on $\interval{x_{n-1}}{x_n}$. Then \[ f(\xi_1),\ f(\xi_2),\ \ldots,\ f(\xi_n) \] are the altitudes of a set of rectangles whose combined area is a more or less close approximation of the area of our figure. Denote this approximate area by $S$. Then \[ S = f(\xi_1)\Delta_1x+f(\xi_2)\Delta_2x+\ldots+f(\xi_n)\Delta_nx = \sum_{k=1}^nf(\xi_k)\Delta_kx. \] As the greatest $\Delta_k x$ is taken smaller and smaller, the figure %-----File: 165.png---Folio 153------ composed of the rectangles comes nearer to the figure bounded by the curve. In consequence of these geometrical notions we define the area of the figure as the limit of $S$ as the $\Delta_kx$'s decrease indefinitely. The area $S$ is the definite integral of $f(x)$ from $a$ to $b$. It has been tacitly assumed that the graph of $y=f(x)$ is continuous, since we do not usually speak of an area being enclosed by a discontinuous curve. The definition of the definite integral when stated in its general form admits, however, of functions which are discontinuous in a great variety of ways. A more general definition of the definite integral is as follows:\index{Definite integral}\index{Integral!definite} \emph{Let $\interval{a}{b}$ (or $\interval{b}{a}$) be an interval upon which a function $f(x)$ is defined, single-valued and bounded. Let $\pi_\delta$ stand for any partition of $\interval{a}{b}$ or $\interval{b}{a}$ by the points $a=x_0, x_1, x_2,\ldots,x_n = b$ such that the numbers $\Delta_1x=x_1-a, \Delta_2x=x_2-x_1,\ldots,\Delta_nx=b-x_{n-1}$ are each numerically less than or equal to $\delta$. \correction{Let}{} \[ \xi_1,\xi_2,\ldots,\xi_n \] be a set of points on the intervals \correction{$\interval{x_0}{x_1}$}{$\interval{x_0-x_1}$}, $\interval{x_1}{x_2}$,\ldots, $\interval{x_{n-1}}{x_n}$ (or if $bC$. Let $\varepsilon=\dfrac{B-C}{4}$. By the definition of value approached, for every $\delta$ there must exist an $S$ (which we call $S_B$) such that \[ \tag{1} |S_B-B|<\varepsilon \] and such that the corresponding $\pi_B$ has its largest $\Delta_kx<\delta$. Similarly there must be an $S_C$ such that \[ \tag{2} |S_C-C|<\varepsilon, \] and such that the corresponding $\pi_C$ has its largest $\Delta_kx<\delta$. Let $\pi$ be a partition made up of the points both of $\pi_B$ and $\pi_C$, and let $S$ be one of the corresponding sums. $\pi$ is a repartition both of $\pi_B$ and $\pi_C$. %-----File: 170.png---Folio 158------ Therefore \[ \tag{3} |S-S_C|\leqq O_{\pi_C} \] and \hypertarget{eq4p158}{\[ \tag{4} |S-S_B| \leqq O_{\pi_B}. \]} But since $f(x)$ is continuous, by the theorem of uniform continuity, $\delta$ can be so chosen that if any two values of $x$ differ by less than $\delta$, the corresponding values of $f(x)$ differ by less than $\dfrac{\varepsilon}{|b-a|}$ and hence on the partitions $\pi_B$ and $\pi_C$, whose $\Delta_kx$'s are all less than $\delta$, the corresponding $\Delta_ky$'s are all less than $\dfrac{\varepsilon}{|b-a|}$. So we have (since $\displaystyle\sum_{k=1}^n \Delta_kx=b-a$) \[ O_{\pi_B} = \sum_{k=1}^n|\Delta_kx| \cdot \Delta_ky < \sum_{k=1}^n|\Delta_kx| \cdot \frac{\varepsilon}{|b-a|} = \varepsilon. \] Hence \[ O_{\pi_B}<\varepsilon \quad \text{and}\quad O_{\pi_C} < \varepsilon. \] So we have, since $\varepsilon=\dfrac{B-C}{4}$ and $\delta$ is so chosen that whenever $|x'-x''| < \delta$, $|f(x')-f(x'')| < \dfrac{\varepsilon}{|b-a|}$: \begin{align*} |S_B-B| &< \varepsilon, \\ |S_C-C| &< \varepsilon, \\ |S_B-S| &< \varepsilon, \\ |S_C-S| &< \varepsilon. \end{align*} From these inequalities it follows that $|B-C|<4\varepsilon$, which contradicts the statement that $\varepsilon=\dfrac{B-C}{4}$. Hence the hypothesis that $f(x)$ is not integrable is untenable. \end{proof} \begin{theorem}[99]\hypertarget{thm99}{} Every non-oscillating bounded function is integrable. \end{theorem} \begin{proof} The proof runs, as in the preceding theorem, to the %-----File: 171.png---Folio 159------ paragraph following \hyperlink{eq4p158}{(4)}. Let $D$ and $d$ be the upper and lower bounds of $f(x)$. $\delta$, being arbitrary, can be so chosen that $\delta = \dfrac{\varepsilon}{D-d}$. Then \[ O_{\pi_B} = \sum_{k=1}^n \Delta_ky\cdot|\Delta_kx| < \sum_{k=1}^n \Delta_ky\cdot\delta, \] and since $f(x)$ is non-oscillating, \[ \sum_{k=1}^n \Delta_ky = D-d. \] Therefore \[ O_{\pi_B}<(D-d)\delta=\varepsilon. \] Similarly $O_{\pi_C}<\varepsilon$. Hence again we have \begin{align*} |S_B-B| & < \varepsilon, \\ |S_C-C| & < \varepsilon, \\ |S_B-S| & < \varepsilon, \\ |S_C-S| & < \varepsilon, \end{align*} and therefore $|B-C|<4\varepsilon$, whereas $\varepsilon$ was assumed equal to $\dfrac{B-C}{4}$. Thus the hypothesis of a non-integrable non-oscillating function is untenable. \end{proof} \section{Computation of Definite Integrals.}\hypertarget{chVIIIsec3}{}%[3] In computing definite integrals it is important to observe that when the integral is known to exist the limit can be calculated on any properly chosen subset of the $S_\delta$'s. (See Theorem~\hyperlink{thm41}{41}.) So we have that if $S_{\delta_1}$, $S_{\delta_2}$, $\ldots$ is any sequence of sums such that $\displaystyle\mathop{L}_{n\doteq\infty}\delta_n=0$, then \[ \mathop{L}_{n\doteq\infty} S_{\delta_n} = \int_a^b f(x)dx. \] One case of this kind occurs when $\xi_k$ is taken as an end-point %-----File: 172.png---Folio 160------ of the interval $\interval{x_{k-1}}{x_k}$ and all the $\Delta_kx$'s are equal. Then we have \[ \int_a^b f(x)dx = \mathop{L}_{n\doteq\infty} \sum_{k=1}^n f(a+k\Delta x)\Delta x, \text{ where } \Delta x=\frac{b-a}{n}. \] A simple example of this principle is the proof of the following theorem. \begin{theorem}[100]\hypertarget{thm100}{} If $f(x)$ is a constant, $C$, then \[ \int_a^b Cdx=C(b-a). \] \end{theorem} \begin{proof} The function $f(x)=C$ is integrable either according to Theorem~\hyperlink{thm98}{98} or Theorem~\hyperlink{thm99}{99}. Hence \[ \int_a^b Cdx = \mathop{L}_{n\doteq\infty} \sum_{k=1}^n C\frac{b-a}{n} = \mathop{L}_{n\doteq\infty} n\cdot C\cdot \frac{b-a}{n} = C(b-a).\qedhere \] \end{proof} A few other examples follow. In each case the function is known to be integrable by the theorems of $\hyperlink{chVIIIsec2}{\S~2}$. \begin{theorem}[101]\hypertarget{thm101}{}\label{t101p160} \[ \int_a^b e^xdx=e^b-e^a. \] \end{theorem} \begin{proof} Let \begin{align*} S_{\Delta x} &= e^a\Delta x + e^{a+\Delta x} \cdot \Delta x + e^{a+2\Delta x}\cdot\Delta x + \ldots + e^{a+(n-1)\Delta x} \cdot \Delta x \\ &= e^a \cdot\Delta x[1+e^{\Delta x} + e^{2\Delta x} + \ldots + e^{(n-1)\Delta x}] \\ &= e^a\cdot\Delta x\cdot\frac{e^{n\Delta x}-1}{e^{\Delta x}-1} = \frac{e^{b-a}-1}{e^{\Delta x}-1}e^a\cdot\Delta x \\ &= (e^b-e^a) \cdot \frac{\Delta x}{e^{\Delta x}-1}. \end{align*} Whence the result follows since $\displaystyle\mathop{L}_{\Delta x\doteq 0} \dfrac{\Delta x}{e^{\Delta x}-1}=1$. (Differentiate numerator and denominator with respect to $\Delta x$ according to Theorem~\hyperlink{thm90}{90}.\correction{)}{} \end{proof} %-----File: 173.png---Folio 161------ Instead of arranging the partition-points in an arithmetical progression as in the cases above, we may put them in a geometrical progression, that is, we let \begin{gather*} \left(\frac ba \right)^{\frac1n} = q, \quad \frac ba = q^n, \\ \Delta_1 x = aq-a, \quad \Delta_2 x = aq^2-aq, \ldots, \Delta_n x = aq^n-aq^{n-1}, \\ \xi_1 = a, \quad \xi_2 = aq, \ldots, \xi_n = aq^{n-1}, \end{gather*} and obtain the formula \begin{align*} \int_a^b f(x) dx &= \mathop{L}_{q\doteq 1} a(q-1) [f(a) + qf(aq) + \ldots + q^{n-1} f(aq^{n-1})] \\ &= \mathop{L}_{q\doteq 1} a(q-1) \sum\limits_{k=0}^{n-1} q^k f(aq^k). \end{align*} We apply this scheme to the following. \begin{theorem}[102]\hypertarget{thm102}{} In all cases where $m$ is a whole number $\neq-1$, and if $a>0$, $b>0$ for every value of $m \neq-1$, \[ \int_a^b x^m dx = \frac{b^{m+1}-a^{m+1}}{m+1}. \] \end{theorem} \begin{proof} \hypertarget{eq1p161}{\begin{gather*} \int_a^b x^m dx = \mathop{L}_{q\doteq 1} a(q-1)\sum\limits_{k=0}^{n-1} q^k (aq^k)^m \\ = a^{m+1} \mathop{L}_{q\doteq 1} (q-1) [1 + (q^{m+1})+ (q^{m+1})^2 + \ldots + (q^{m+1})^{n-1}] \tag{1} \end{gather*}} \begin{align*} &= a^{m+1} \mathop{L}_{q\doteq 1} (q-1) \frac{(q^{m+1})^n-1}{q^{m+1}-1} \\ &= \mathop{L}_{q\doteq 1} a^{m+1} \{(q^n)^{m+1}-1\} \frac{q-1}{q^{m+1}-1} \\ &= (b^{m+1}-a^{m+1}) \mathop{L}_{q\doteq 1} \frac{q-1}{q^{m+1}-1}. \end{align*} %-----File: 174.png---Folio 162------ Hence \[ \int_a^b x^mdx=\frac{b^{m+1}-a^{m+1}}{m+1}, \] since \[ \mathop{L}_{q\doteq 1} \frac{q-1}{q^{m+1}-1} = \frac{1}{m+1}.\qedhere \] \end{proof} \begin{theorem}[103]\hypertarget{thm103}{} \[ \int_a^b\frac1xdx = \log b-\log a,\ (0 0$ there is an infinite set of partitions $\pi$, for which the largest $\Delta_k x$ is less than $\delta$, and for each of these there is a value of $O_\pi$. If $O_\delta$ stands for any such $O_\pi$, then $O_\delta$ is a many-valued function of $\delta$. \begin{theorem}[126]\hypertarget{thm126}{} A necessary and sufficient condition that a function $f(x)$, defined, single-valued, and bounded on an interval $\interval{a}{b}$, is integrable is that \[ \mathop{L}_{\delta\doteq 0} O_\delta = 0. \] \end{theorem} \begin{proof}\textit{The condition is necessary.} By Theorem~\hyperlink{thm125}{125} the integrability of $f(x)$ implies $\underline{B} O_\pi = 0$. Hence for every $\varepsilon$ there exists a partition $\pi$ such that \[ O_\pi < \varepsilon. \] By Lemma~\hyperlink{lem4p178}{4} there exists a $\delta_\varepsilon$ such that for every $\pi'$ whose greatest $\Delta x$ is less than $\delta_\varepsilon$ \[ O_{\pi'} < O_\pi + \varepsilon < 2\varepsilon. \] Hence \[ \mathop{L}_{\delta\doteq 0} \text{\correction{$O_\delta$}{$O^\delta$}} = 0. \] \textit{The condition is sufficient.} Since \[ \mathop{L}_{\delta\doteq 0} \text{\correction{$O_\delta$}{$O^\delta$}} = 0, \] and $O_\delta > 0$, \[ \underline{B} O_\pi = 0. \] Hence the function is integrable by Theorem~\hyperlink{thm125}{125}. \end{proof} \begin{theorem}[127]\hypertarget{thm127}{}\label{p182th127} A necessary and sufficient condition that a function, defined, single-valued, and bounded on an interval $\interval{a}{b}$, shall be integrable on that interval is that for every pair of positive %-----File: 195.png---Folio 183------ numbers $\sigma$ and $\lambda$ there exists a partition $\pi$ such that the sum of the lengths of those intervals on which the oscillation of the function is greater than $\sigma$ is less than $\lambda$. \end{theorem} \begin{proof}\textit{The condition is necessary.} If for a given pair of positive numbers $\sigma$ and $\lambda$ there exists no $\pi$ such as is required by the theorem, then $O_\pi > \sigma\cdot\lambda$ for every $\pi$, which is contrary to the conclusion of Theorem~\hyperlink{thm125}{125} that \[ \underline{B}O_\pi = 0. \] \textit{The condition is sufficient.} For a given positive $\varepsilon$ choose $\sigma$ and $\lambda$ so that \[ \sigma(b-a) < \frac\varepsilon2 \text{ and } \lambda \cdot R < \frac\varepsilon2, \] where $R$ is the oscillation of the function on $\interval{a}{b}$. Let $\pi$ be a partition such that the sum of the lengths of those intervals on which the oscillation of the function is greater than $\sigma$ is less than $\lambda$. Then the sum of the terms of $O_\pi$ which occur on these intervals is less than \[ \lambda \cdot R, \] and the sum of the terms of $O_\pi$ on the remaining intervals is less than \[ \sigma(b-a). \] Therefore \[ O_\pi < \lambda \cdot R + \sigma(b-a) < \varepsilon. \] Hence \[ \underline{B}O_\pi = 0, \] whence by Theorem~\hyperlink{thm125}{125} the integral exists. \end{proof} \begin{definition}\index{Content of a set of points} The \textit{content} of a set of points $[x]$ on an interval $\interval{a}{b}$ is a number $C[x]$ defined as follows: Let $\pi$ be any partition of $\interval{a}{b}$, none of the partition points of which are points of $[x]$, and $D_\pi$ the sum of the lengths of those intervals of $\pi$ %-----File: 196.png---Folio 184------ which contain points of [$x$] as interior points. Then \[ \underline{B}D_\pi = C[x]. \] An important special case is where \[ C[x]=0. \] It is evident that if a set [$x$] has content zero, for every $\varepsilon$ there exists a finite set of segments of lengths \[ \varepsilon_1,\; \varepsilon_2,\; \varepsilon_3, \ldots,\; \varepsilon_n \] which contain every point [$x$] and such that \[ \sum_{i=1}^n \varepsilon_i < \varepsilon. \] It is also evident that if the sets [$x_1$] and [$x_2$] are of content zero, then the set of all $x_1$ and $x_2$ is of content zero.\footnote{% For further discussion of the notion \emph{content} see \textsc{Pierpont}, \textit{Real Functions}, Vol.~I, p.~352, and \correction{\textsc{Lebesgue}}{\textsc{Lebesque}}, \textit{Le\c cons sur l'Int\'egration}.} \end{definition} \begin{theorem}[128]\hypertarget{thm128}{} A necessary and sufficient condition for the integrability of a function $f(x)$ on an interval $\interval{a}{b}$ is that for every $\sigma > 0$ the set of points $[x_\sigma]$ at which the oscillation of $f(x)$ is greater than or equal to $\sigma$ shall be of content zero.\footnote{% Compare the example on page~\pageref{egp155}.} \end{theorem} \begin{proof} If at every point of an interval $\interval{c}{d}$ the oscillation of $f(x)$ is less than $\sigma$, then about each point of $\interval{c}{d}$ there is a segment upon which the oscillation is less than $\sigma$, and hence by Theorem~\hyperlink{thm11}{11}, Chapter~\hyperlink{chapII}{II}, there is a partition of $\interval{c}{d}$ upon each interval of which the oscillation of $f(x)$ is less than $\sigma$. Now to prove the condition sufficient we observe that if the content of [$x_\sigma$] is zero, there exists for every $\lambda$ a partition $\pi_\lambda$, such that the sum of the lengths of the intervals containing points of [$x_\sigma$] is less than $\lambda$. Moreover we have just seen %-----File: 197.png---Folio 185------ that the intervals which do not contain points on $[x_\sigma]$ can be repartitioned into intervals on which the oscillation is less than $\sigma$. Hence, by Theorem~\hyperlink{thm127}{127}, the function is integrable. To prove the condition necessary we note that on every interval containing a point, $x_\sigma$, the oscillation of $f(x)$ is greater than \correction{or equal to}{or equal to or equal to} $\sigma$. Hence, if \[ C[x_\sigma] > 0, \] the sum of the intervals upon which the oscillation is greater than or equal to $\sigma$ is greater than $C[x_\sigma]$. \end{proof} \begin{definition}\index{Numerably infinite set}\index{Non-numerably infinite set} A set of points is said to be numerable if it is capable of being set into one-to-one correspondence with the positive integral numbers. If a set $[x]$ is numerable, it can always be indicated by the notation $x_1$, $x_2$, $x_3, \ldots$, $x_n, \ldots$, or $\{x_n\}$, but if it is not numerable, the notation $\{x_n\}$ cannot be applied with the understanding that $n$ is integral. \end{definition} \begin{theorem}[129]\hypertarget{thm129}{} A perfect set of points is not numerably infinite.\footnote{% For definition of perfect set see page~\pageref{dp41}.} \end{theorem} \begin{proof} Suppose the theorem not true. Then there exists a sequence of points $\{x_n\}$ containing every point of a perfect set $[x]$. Let $P_1$ be any point of $[x]$, and $\overline{a_1\ b_1}$ a segment containing $P_1$. Let $x_{n_1}$ be the first of $\{x_n\}$ within $\overline{a_1\ b_1}$. Since $x_n$ is a limit point of points of $[x]$, there are points of the set other than $P_1$ and $x_{n_1}$ on the segment $\overline{a_1\ b_1}$. Let $P_2$ be such a point, and let $\overline{a_2\ b_2}$ be a segment within $\overline{a_1\ b_1}$ and containing $P_2$ but neither $P_1$ nor $x_{n_1}$. Let $x_{n_2}$ be the first point of $\{x_n\}$ within $\overline{a_2\ b_2}$. Proceeding in this manner we obtain a sequence of segments $\{\overline{a_i\ b_i}\}$ such that every segment lies within the preceding and such that every segment $\overline{a_i\ b_i}$ contains no point $x_{n_{i-k}}$ of the sequence $\{x_n\}$. By the lemma on page~\pageref{lp42}, Chapter~\hyperlink{chapII}{II}, there is a point $P$ on every segment of this set. Since there are points of $[x]$ on every segment $\overline{a_i\ b_i}$, $P$ is a limit point of the set $[x]$. Since $[x]$ is a perfect set, $P$ is a point of $[x]$. But if $P$ %-----File: 198.png---Folio 186------ were in the sequence $\left\{x_n\right\}$, there would be only a finite number of points of $[x]$ preceding $P$, whereas by the construction there is an infinitude of such points. \end{proof} \begin{theorem}[130]\hypertarget{thm130}{} A numerably infinite set of sets of points each of content zero cannot contain every point of any interval. \end{theorem} \begin{proof} Let the set of sets be ordered into a sequence $\left\{[x]_n\right\}$. We show that on every segment $\overline{a\ b}$ there is at least one point not of $\left\{[x]_n\right\}$. Since $[x]_1$ is of content zero, there is a segment $\overline{a_1\ b_1}$ contained in $\overline{a\ b}$ which contains no point of $[x]_1$. Let $[x]_{{n}_1}$ be the first set of the sequence which contains a point of $\overline{a_1\ b_1}$. Since $[x]_{{n}_1}$ is of content zero, there is a segment $\overline{a_2\ b_2}$ contained in $\overline{a_1\ b_1}$ which contains no point of $[x]_{{n}_1}$. Continuing in this manner we obtain a sequence of segments $\overline{a\ b}$, $\overline{a_1\ b_1},\ldots$, $\overline{a_n\ b_n} \ldots$ such that every segment lies within the preceding, and such that $\overline{a_n\ b_n}$ contains no point of $[x]_1,\ldots$, $[x]_n$. By the lemma on page~\pageref{lp42} there is at least one point $P$ on all these segments. Hence $P$ is a point of $\overline{a\ b}$ and is not a point of any set of $\left\{[x]_n\right\}$. \end{proof} \begin{theorem}[131]\hypertarget{thm131}{} The points of discontinuity of an integrable function form at most a set consisting of a numerable set of sets, each of content zero. \end{theorem} \begin{proof} Let $\sigma_1$, $\sigma_2$, $\sigma_3,\ldots$ be any set of numbers such that \[ \sigma_n>\sigma_{n+1}, \] and \[ \mathop{L}_{n\doteq \infty}\sigma_n =0. \] By Theorem~\hyperlink{thm128}{128} the set of points $[x_{\sigma_n}]$ at which the oscillation of $f(x)$ is greater than or equal to $\sigma_{n+1}$ and less than $\sigma_n$ is of content zero. Since the set of sets $\left\{[x_{\sigma_n}]\right\}$ includes all the points of discontinuity of $f(x)$, this proves the theorem. \end{proof} \begin{theorem}[132]\hypertarget{thm132}{} If a function $f(x)$ is integrable on an interval $\interval{a}{b}$, then it is continuous at a set of points which is everywhere dense on $\interval{a}{b}$. \end{theorem} %-----File: 199.png---Folio 187------ \begin{proof} If the theorem fails to hold, then there exists an interval $\interval{a}{b}$ on which the function is discontinuous at every point. By Theorem~\hyperlink{thm131}{131} an integrable function is discontinuous at most on a numerably infinite set of sets each of content zero, and by Theorem~\hyperlink{thm130}{130} such sets of sets fail to contain every point of any interval. \end{proof} \begin{theorem}[133]\hypertarget{thm133}{} If \[ \int_a^X f(x)dx=0 \] for every $X$ of $\interval{a}{b}$, then $f(x) =0$ on a set of points everywhere dense on $\interval{a}{b}$, and for every $\sigma>0$ the points where $|f(x)|>\sigma$ form a set of content zero. \end{theorem} \begin{proof} At every point $X$ where $f(x)$ is continuous, according to the corollary of Theorem~\hyperlink{thm119}{119}, \[ \frac{d}{dX}\int_a^X f(x)dx = f(X) = 0, \] since $\displaystyle\int_a^X f(x)dx$ is a constant. The points of continuity of $f(x)$ are everywhere dense, according to Theorem~\hyperlink{thm132}{132}. Hence the zero points of $f(x)$ are everywhere dense. At a point of discontinuity the oscillation of $f(x)$ is greater than or equal to $|f(x)|$. Hence the points where $|f(x)|>\sigma$ form a set of content zero. \end{proof} \begin{theorem}[134]\hypertarget{thm134}{} If \[ \int_a^X f(x)dx = \int_a^X \phi(x)dx \] for every $X$ of $\interval{a}{b}$, then $f(x) = \phi(x)$ on a set of points everywhere dense on $\interval{a}{b}$, and for every $\sigma>0$ the points where $|f(x)-\phi(x)|>\sigma$ forms a set of content zero. \end{theorem} \begin{proof} Apply the theorem above to $f(x)-\phi(x)$. \end{proof} \begin{theorem}[135]\hypertarget{thm135}{} If $f(x)$ is integrable from $a$ to $b$, then $|f(x)|$ is integrable from $a$ to $b$.\footnote{% The converse theorem is not true; cf.~example given on page~\pageref{egp192}.} \end{theorem} %-----File: 200.png---Folio 188------ \begin{proof} Since \[ \text{\correction{$0$}{$O$}}\leqq O_{\pi}\left|f(x)\right|\leqq O_{\pi}f(x), \] it follows that $\underline{B}\ O_{\pi}f(x)=0$ implies $\underline{B}\ O_{\pi}|f(x)|=0$, and hence the integrability of $f(x)$ implies the integrability of $|f(x)|$. \end{proof} \begin{theorem}[136]\hypertarget{thm136}{} If $f(x)$ and $\phi(x)$ are both integrable on an interval $\interval{a}{b}$, then \hypertarget{fn1}{\[ f(x)\cdot \phi(x) \tag{1} \]} is integrable on $\interval{a}{b}$; and, provided there is a constant $m>0$ such that $|\phi(x)|-m>0$ for $x$ on $\interval{a}{b}$, then \hypertarget{fn2}{\[ f(x) \div \phi(x) \tag{2} \]} is integrable on $\interval{a}{b}$. \end{theorem} \begin{proof} Since $f(x)$ and $\phi(x)$ are both integrable on $\interval{a}{b}$, it follows that for every pair of positive numbers $\sigma$ and $\lambda$ there is a partition $\pi_1$ for $f(x)$ and a partition $\pi_2$ for $\phi(x)$ such that the sums of the lengths of the intervals on which the oscillations of $f(x)$ and $\phi(x)$ respectively are greater than $\sigma$ are less than $\lambda$. Let $\pi$ be the partition consisting of the points of both $\pi_1$ and $\pi_2$. Then the sum of the intervals of $\pi$ on which the oscillation of either $f(x)$ or $\phi(x)$ is greater than $\sigma$ is less than $2\lambda$. Let $M$ be the greater of $\overline{B}|f(x)|$ and $\overline{B}|\phi(x)|$ on $\interval{a}{b}$. Then on any interval of $\pi$ on which the oscillations of $f(x)$ and $\phi(x)$ are both less than $\sigma$ the oscillation of $f(x)\cdot \phi(x)$ is less than $\sigma M$. Hence the sum of the intervals on which the oscillation of $f(x)\cdot \phi(x)$ is greater than $\sigma M$ is less than $2\lambda$. Since $\sigma$ and $\lambda$ may be chosen so that $2\lambda$ and $\sigma M$ shall be any pair of preassigned numbers, it follows by Theorem~\hyperlink{thm127}{127} that $f(x)\cdot \phi(x)$ is integrable on $\interval{a}{b}$. In view of the argument above it is sufficient for the second %-----File: 201.png---Folio 189------ theorem to prove that $\dfrac{1}{\phi(x)}$ is integrable on $\interval{a}{b}$ if $\phi(x)$ is integrable and $|\phi(x)|>m$. Consider a partition $\pi$ such that the sum of the intervals on which the oscillation of $\phi(x)$ is greater than $\sigma$ is less than $\lambda$. Since \[ \left| \frac{1}{ \phi(x_1) } -\frac{1}{ \phi(x_2) } \right| = \frac{\left| \phi(x_1)-\phi(x_2) \right|} {\left| \phi(x_1) \right|\cdot \left| \phi(x_2) \right|}, \] it follows that $\pi$ is such that the sum of the intervals on which the oscillation of $\dfrac{1}{\phi(x)}$ is greater than $\dfrac{\sigma}{m^2}$ is less than $\lambda$, and $\dfrac{1}{\phi(x)}$ is integrable according to Theorem~\hyperlink{thm127}{127}. \end{proof} A second proof may be made by comparing the integral oscillations of $f(x)$ and $\phi(x)$ with those of the functions \hyperlink{fn1}{(1)} and \hyperlink{fn2}{(2)} and applying Theorem~\hyperlink{thm125}{125}.\footnote{% Cf.\ \textsc{Pierpont}, Vol.~I, pp.~346, 347, 348.} \begin{theorem}[137]\hypertarget{thm137}{} If $f(x)$ is an integrable function on an interval $\interval{a}{b}$, and if $\phi(y)$ is a continuous function on an interval $\interval{\underline{B}f}{\overline{B}f}$, where $\underline{B}f$ and $\overline{B}f$ are the lower and upper bounds respectively of $f(x)$ on $\interval{a}{b}$, then $\phi\{f(x)\}$ is an integrable function of $x$ on the interval $\interval{a}{b}$.\footnote{% This theorem is due to \textsc{Du Bois Reymond}. It cannot be modified so as to read ``an integrable function of an integrable function is integrable.'' Cf.\ \textsc{E.~H. Moore}, \textit{Annals of Mathematics}, new series, Vol.~2, 1901, p.~153.} \end{theorem} \begin{proof} By Theorem~\hyperlink{thm48}{48} there exists for every $\sigma>0$ a $\delta_{\sigma}$ such that for $|y_1-y_2|<\delta_{\sigma}$, \hypertarget{eq1p189}{\[ \left|\phi(y_1)-\phi(y_2)\right|<\sigma. \tag{1} \]} Since $f(x)$ is integrable on $\interval{a}{b}$ it follows by Theorem~\hyperlink{thm127}{127} that for every positive number $\lambda$ there is a partition $\pi$ such %-----File: 202.png---Folio 190------ that the sum of the intervals on which the oscillation of $f(x)$ is greater than $\delta_{\sigma}$ is less than $\lambda$. But by \hyperlink{eq1p189}{(1)} this means that the sum of the intervals on which the oscillation of $\phi\{f(x)\}$ is greater than $\sigma$ is less than $\lambda$. This, by Theorem~\hyperlink{thm127}{127}, proves that $\phi\left\{f(x)\right\}$ is integrable. \end{proof} %-----File: 203.png---Folio 191------ \chapter{IMPROPER DEFINITE INTEGRALS.}\hypertarget{chapIX}{}%[IX] \index{Improper definite integral} \section{The Improper Definite Integral on a Finite Interval.}\hypertarget{chIXsec1}{}%[1] \label{p191}If $f(x)$ is infinite at one or more points of the interval $\interval{a}{b}$, then, whatever may be the other properties of the function, the definite integral of $f(x)$ defined in Chapter~\hyperlink{chapVIII}{VIII} cannot exist on the interval $\interval{a}{b}$. \begin{definition}\label{dp192} If $\displaystyle \int_x^bf(x)dx$ exists for every $x$, $am$ for every $x$, then \[ \mathop{L}_{x\doteq a}\int_x^bf(x)dx \] cannot exist and be finite. \end{theorem} %-----File: 207.png---Folio 195------ \begin{proof}(1) In case \[ \int_x^b f(x)dx \] fails to exist for some value of $x$ between $a$ and $b$, \[ \mathop{L}_{x \doteq a} \int_x^b f(x) dx \] fails to exist because the limitand function does not exist. (2) If \[ \int_x^b f(x)dx \] exists for every value of $x$ between $a$ and $b$, we proceed as follows: Let $\delta<1$ be the length of a $V^*(a)$ on which $f(x)$ does not change sign, and on which $(x-a)^kf(x)>m$, and let $x_2$ be the extremity of this neighborhood, which is greater than $a$. Then $|f(x)|>\dfrac{m}{(x-a)^k}>\dfrac{m}{(x_2-a)^k}$ for every $x$ on this neighborhood. Take $x_1$ so that $(x_2-a)^k=2(x_2-x_1)$. Then \[ \left|\int_{x_1}^{x_2}f(x)dx\right| > \frac{m}{(x_2-a)^k} (x_2-x_1) = \tfrac{1}{2}m. \] Hence, by the necessary condition of Theorem~\hyperlink{thm138}{138}, \[ \mathop{L}_{x\doteq a} \int_x^b f(x) dx \] cannot exist and be finite. \end{proof} \begin{theorem}[142]\hypertarget{thm142}{} If \[ \mathop{L}_{x\doteq a}\int_x^b f(x)dx \] exists and is finite and if $f(x)$ approaches infinity monotonically as $x\doteq a$ on some $V^*(a)$, then \[ \mathop{L}_{x\doteq a} (x-a) \cdot f(x) = 0, \] %-----File: 208.png---Folio 196------ or in other words $f(x)$ has an infinity of order lower than $\dfrac{1}{x-a}$.\footnote{% $\displaystyle\mathop{L}_{x\doteq a} (x-a)\cdot f(x) =0$ is not a sufficient condition for the existence of \[ \mathop{L}_{x\doteq a} \int_x^b f(x)dx, \] as is shown by the following example. Consider a set of points $x_1$, $x_2$, $x_3,\ldots$, $x_n,\ldots$ such that $x_n-a = 2(x_{n+1}-a)$, $x_1-a$ being unity. Define $f(x_1)=1$, $f(x_2)=\frac43$, $f(x_3)=2,\ldots$, $f(x_n)=\dfrac{2^n}{n+1},\ldots$. Let the function be linear from $f(x_1)$ to $f(x_2)$, from $f(x_2)$ to $f(x_3)$, etc. Then \[ \left| \int_{x_1}^{x_2} f(x)dx\right| > \tfrac{1}{2}, \qquad \left| \int_{x_2}^{x_3} f(x)dx\right| > \tfrac{1}{3}, \text{ etc.} \] Since these integrals are all of the same sign, their sum for any given number of terms is greater than the sum of the corresponding number of terms in the harmonic series. Also $(x_n-a) \cdot f(x_n) = \dfrac{2}{n+1}$, whence $\displaystyle\mathop{L}_{x\doteq a} (x-a)\cdot f(x)=0$. } %end footnote \end{theorem} \begin{proof} By means of Theorem~\hyperlink{thm138}{138} it follows from the hypothesis that for every $\varepsilon$ there exists a ${V_\varepsilon}^*(a)$ within $V^*(a)$ such that for every $x_1$ and $x_2$ on $\interval{a}{b}$, and also on ${V_\varepsilon}^*(a)$, \[ \left| \int_{x_1}^{x_2} f(x)dx \right| < \varepsilon. \] Let $x_2$ be any point of such a neighborhood and let $x_1$ be so chosen that \[ x_1-a=x_2-x_1. \] Since $x_1$ and $x_2$ are on $V^*(a)$, \[ f(x_1) > f(x_2). \] It follows from Theorem~\hyperlink{thm116}{116} that \[ \left| \int_{x_1}^{x_2} f(x)dx \right| > |f(x_2)| \cdot (x_2-x_1). \] But \[ f(x_2) \cdot (x_2-x_1) = \tfrac{1}{2} f(x_2) \cdot (x_2-a). \] %-----File: 209.png---Folio 197------ Hence for $x=x_2$, \[ |f(x)| \cdot (x-a) < 2 \varepsilon. \] Since $\varepsilon$ is arbitrary, and since $x_2$ is any point in $V^*(a)$, it follows that \[ \mathop{L}_{x\doteq a} f(x)\cdot(x-a)=0.\qedhere \] \end{proof} \begin{corollary} If \[ \int_x^b f(x)dx \] exists for every $x$ between $a$ and $b$, and \[ \mathop{L}_{x\doteq a} \int_x^b f(x)dx \] exists and is finite, and if $f(x)$ is entirely positive or entirely negative, then zero is a value approached by $(x-a)\cdot f(x)$ as $x$ approaches $a$. \end{corollary} \begin{proof} Consider the case when the function is entirely positive. Suppose zero is not a value approached. Then there exists a pair of positive numbers $\varepsilon$ and $\delta$ such that for every $x$, $x-a<\delta$, \[ (x-a) \cdot f(x)>\varepsilon. \] On the interval, $\interval{a}{a+\delta}$, consider the function \[ \frac{\varepsilon}{x-a}. \] Since \[ \int_x^b \frac{\varepsilon}{x-a}dx \] is a non-oscillating function of $x$, it follows from Theorem~\hyperlink{thm25}{25} that \[ \mathop{L}_{x\doteq a} \int_x^b \frac{\varepsilon}{x-a}dx \] exists, and by Theorem~\hyperlink{thm142}{142} this limit must be infinite. %-----File: 210.png---Folio 198------ Since \[ |f(x)|> \frac{\varepsilon}{x-a} \] on the neighborhood under consideration, it follows from Theorem~\hyperlink{thm107}{107} and Corollary~\hyperlink{cor2p82}{2}; Theorem~\hyperlink{thm40}{40}, that \[ \mathop{L}_{x\doteq a} \int_x^b f(x)dx \] exists and is infinite, which is contrary to the hypothesis. \end{proof} \begin{theorem}[143\footnotemark]\hypertarget{thm143}{}\footnotetext{% This is what Professor \textsc{Moore} in his lectures calls the relative convergence theorem. Theorems~143, 144, 151, 152 in this form are due to him.} If \begin{enumerate} \item[\textnormal{(1)}] $f_1(x)$ and $f_2(x)$ are of the same rank of infinity at $x = a$, or if $f_1(x)$ is of lower order than $f_2(x)$, \item[\textnormal{(2)}] $\displaystyle\int_x^b f_1(x)dx$ and $\displaystyle\int_x^b f_2(x)dx$ both exist for every $x$ on the segment $\overline{a\ b}$, \item[\textnormal{(3)}] There is a neighborhood of $x = a$ on which $f_2(x)$ does not change sign, \item[\textnormal{(4)}] $\displaystyle{\mathop{L}_{x\doteq a} \int_a^b} f_2(x)dx$ is finite,\footnote{% We notice that since under the hypothesis $f_2(x)$ does not change sign, \[ L \int_x^b f_2(x)dx \] cannot fail to exist either finite or infinite, for it follows from this hypothesis that $\displaystyle\int_x^b f_2(x)dx$ is a non-oscillating function of $x$ and therefore, by Theorem~\hyperlink{thm25}{25} that the limit exists.} \end{enumerate} then it follows that $\displaystyle{\mathop{L}_{x\doteq a} \int_x^b} f_1(x)dx$ exists and is finite. \end{theorem} %-----File: 211.png---Folio 199------ \begin{proof} Since from the hypothesis \[ \mathop{L}_{x\doteq a} \int_x^b f_2(x)dx \] exists and is finite, we have by Theorem~\hyperlink{thm138}{138} that for every $\varepsilon$ there exists a $V_\varepsilon^*(a)$ such that for every $x_1$ and $x_2$ on segment $\overline{a\ b}$ and on $V_\varepsilon^*(a)$ \[ \left|\int_{x_1}^{x_2} f_2(x)dx \right|< \varepsilon. \] Consider $x_1$ and $x_2$ on a neighborhood of $x = a$ for which $\left|\dfrac{f_1(x)}{f_2(x)} \right|< M$ and for which $f_2(x)$ does not change sign. Then, by Theorem~\hyperlink{thm113}{113}, \[ \left|\int_{x_1}^{x_2} f_1(x)dx \right| < M \cdot \left|\int_{x_1}^{x_2} f_2(x)dx \right| < M \cdot \varepsilon. \] Since $M \cdot \varepsilon$ can be made small at will by making $\varepsilon$ small, it follows by Theorem~\hyperlink{thm138}{138} that \[ \mathop{L}_{x\doteq a} \int_x^b f_1(x)dx \] exists and is finite. \end{proof} An important special case of this theorem is when $f_1(x)$ is of the same or lower order of infinity than $f_2(x)$, i.e., $\displaystyle{\mathop{L}_{x\doteq a}} \dfrac{f_1(x)}{f_2(x)} = K$, a constant not zero. The reader should verify for himself that Theorem~\hyperlink{thm140}{140} is a corollary of Theorem~\hyperlink{thm143}{143}. The other previous tests for the existence of the improper definite integral can all be reduced to special cases of Theorem~\hyperlink{thm143}{143}. Cf., for example, the logarithmic test on page~410 of \textsc{Pierpont}. \begin{theorem}[144]\hypertarget{thm144}{} If \begin{enumerate} \item[\textnormal{(1)}] $f_1(x)$ and $f_2(x)$ are of the same rank of infinity at $x = a$, or if $f_1(x)$ is of higher order than $f_2(x)$, \item[\textnormal{(2)}] $\displaystyle\int_x^b f_1(x)dx$ and $\displaystyle\int_x^b f_2(x)dx$ both exist for every $x$ on the segment $\interval{a}{b}$, %-----File: 212.png---Folio 200------ \item[\textnormal{(3)}] There is a neighborhood of $x=a$ on which $f_1(x)$ does not change sign, \item[\textnormal{(4)}] $\displaystyle \mathop{L}_{x\doteq a}\int_x^bf_2(x)dx$ is infinite (see note under Theorem~\hyperlink{thm143}{143}), \end{enumerate} then $\displaystyle \mathop{L}_{x\doteq a}\int_x^bf_1(x)dx$ exists and is infinite or fails to exist.\footnote{% This is what Professor \textsc{Moore} calls the relative divergence theorem.} \end{theorem} \begin{proof} This is a direct consequence of Theorem~\hyperlink{thm143}{143}, since if \[ \mathop{L}_{x\doteq a}\int_x^bf_1(x)dx, \] which exists by the foot-note of Theorem~\hyperlink{thm143}{143}, were finite, then \[ \mathop{L}_{x\doteq a}\int_x^bf_2(x)dx \] would exist and be finite. \end{proof} \begin{theorem}[145]\hypertarget{thm145}{} If for a function $f_1(x)$ which does not change sign in the neighborhood of $x=a$ there exists a monotonic function $f_2(x)$ infinite of the same rank as $f_1(x)$ as $x$ approaches $a$, $\displaystyle \int_x^bf_1(x)dx$ and $\displaystyle \int_x^bf_2(x)dx$ both existing for every $x$ on the segment $\overline{a\ b}$, then a necessary condition that $\displaystyle \mathop{L}_{x\doteq a}\int_x^bf_1(x)dx$ shall exist and be finite is that \[ \mathop{L}_{x\doteq a}(x-a)\cdot f_1(x)=0. \] \end{theorem} \begin{proof} By hypothesis \[ \mathop{L}_{x\doteq a}\int_x^bf_1(x)dx \] %-----File: 213.png---Folio 201------ exists and is finite. Hence, by Theorem~\hyperlink{thm143}{143}, \[ \mathop{L}_{x\doteq a}\int_x^bf_2(x)dx \] exists and is finite. Therefore, by Theorem~\hyperlink{thm142}{142}, \[ \mathop{L}_{x\doteq a}(x-a)\cdot f_2(x)=0. \] Since $ \left|\dfrac{f_1(x)}{f_2(x)}\right|$ is bounded as $x$ approaches $a$, i.e., $|f_1(x)|1$, then \[ \mathop{L}_{x\doteq \infty}\int_a^xf(x)dx \] exists and is finite. \end{theorem} %-----File: 215.png---Folio 203------ \begin{proof} If in the proof of Theorem~\hyperlink{thm140}{140} we write $D_{\varepsilon}^{1-k}= \dfrac{\varepsilon (1-k)}{M}$ instead of $\delta_{\varepsilon}^{1-k}=\dfrac{\varepsilon (1-k)}{M}$, and use Theorem~\hyperlink{thm146}{146} instead of 138, the proof of Theorem~\hyperlink{thm140}{140} will apply to Theorem~\hyperlink{thm148}{148}. \end{proof} \begin{theorem}[149]\hypertarget{thm149}{} If $f(x)$ does not change sign for $x$ greater than some fixed number $D$, and if for some positive number $m$ and some number $ k\leqq 1$\correction{,}{} $\left|(x-a)^k\cdot f(x)\right|>m$ for every $x$ greater than $D$, then \[ \mathop{L}_{x\doteq \infty}\int_a^xf(x)dx \] cannot exist and be finite. \end{theorem} \begin{proof} By making suitable changes in the proof of Theorem~\hyperlink{thm141}{141} so as to make $x_1$ and $x_2$ approach infinity instead of $a$, that proof applies to this theorem. \end{proof} \begin{theorem}[150]\hypertarget{thm150}{} If \[ \mathop{L}_{x\doteq \infty}\int_a^xf(x)dx \] exists and is finite, and if $f(x)$ is monotonic for all values of $x$ greater than some fixed number, then \[ \mathop{L}_{x\doteq \infty}(x-a)\cdot f(x)=0. \] \end{theorem} \begin{proof} By making slight modifications of the proof of Theorem~\hyperlink{thm142}{142}, that proof applies to this theorem. \end{proof} \begin{corollary} If \[ \int_a^xf(x)dx \] exists for every $x$ greater than $a$, and \[ \mathop{L}_{x\doteq \infty}\int_a^xf(x)dx \] exists and is finite, and if $f(x)$ does not change sign for $x$ greater %-----File: 216.png---Folio 204------ than some fixed number, then zero is a value approached by $(x-a)f(x)$ as $x$ approaches $\infty$. \end{corollary} The proof is similar to that of the corollary of Theorem~\hyperlink{thm142}{142}. \begin{theorem}[151]\hypertarget{thm151}{} If \begin{enumerate} \item[\textnormal{(1)}] $f_1(x)$ and $f_2(x)$ are infinitesimals of the same rank as $x$ approaches $\infty$, or if $f_1(x)$ is of higher order than $f_2(x)$, \item[\textnormal{(2)}] $\displaystyle \int_a^xf_1(x)dx$ and $\displaystyle \int_a^xf_2(x)dx$ both exist for every $x$, $aa$, then a necessary condition that \[ \mathop{L}_{x\doteq \infty}\int_a^xf_1(x)dx \] shall exist and be finite is that \[ \mathop{L}_{x\doteq \infty}(x-a)\cdot f_1(x)=0. \] \end{theorem} The proof is like that of Theorem~\hyperlink{thm145}{145}. \section{Properties of the Simple Improper Definite Integral.}\hypertarget{chIXsec3}{}%[3] \index{Improper definite integral!simple}\index{Simple improper definite integral} The following definition of the simple improper definite integral is equivalent in substance to that given on page~\pageref{dp192}, and in form is partly the definition of the general improper definite integral given on page~\pageref{s3p210}. The \label{dp205}definite integral of a function is said to \index{Proper existence of the definite integral at a point}\index{Integral!existing properly at a point}\textit{exist properly at a point} $x_1$ or in the neighborhood of this point, on the interval $\interval{a}{b}$ if there exists an interval on $\interval{a_1}{b_1}$ containing $x_1$ as an interior point (or as an end point in case $x_1=a$ or $x_1=b$) such that the proper definite integral of $f(x)$ exists on this interval. The integral is said to \index{Improper existence of the definite integral}exist improperly at a point $x_1$ on the interval $\interval{a}{b}$ if $f(x)$ has an infinite singularity at $x_1$ and there exists an interval $\interval{a_1}{b_1}$ on $\interval{a}{b}$ containing $x_1$ as an interior point (or end point in case $x_1=a$ or $x_1=b$) such that the improper definite integral exists on each of the intervals $\interval{a_1}{x_1}$ and $\interval{x_1}{b_1}$. If on an interval $\interval{a}{b}$ the definite integral exists properly at every point except a finite number of points, and exists improperly at each of these points, then the improper definite integral is said to exist simply on the interval $\interval{a}{b}$, or the simple improper definite integral is said to exist on %-----File: 218.png---Folio 206------ the interval $\interval{a}{b}$. Let $x_1$, $x_2, \ldots$, $x_n$ be the points of $\interval{a}{b}$ at which the integral exists improperly. The \emph{simple improper definite integral} on $\interval{a}{b}$ is the sum of the improper definite integrals on the intervals $\interval{a}{x_1}$, $\interval{x_1}{x_2}, \ldots$, $\interval{x_{n-1}}{x_n}$, $\interval{x_n}{b}$. We denote the simple improper definite integral of $f(x)$ on the interval $\interval{a}{b}$ by \[ \sideset{_S}{_a^b}\int f(x)dx. \] This symbol is used generically to include the proper as well as the improper definite integral. \begin{theorem}[154]\hypertarget{thm154}{} If $a 2 |f_1(x_1)| > 2M, \] and such that \[ (x_2-a)\leqq \textstyle\frac12(x_1-a). \] Let $x_1$, $x_2$, $x_3, \ldots$, $x_n,\ldots$ be a sequence of points dense only at $a$ such that \[ |f_1(x_n)| > 2 |f_1 (x_{n-1})| > 2^{n-1} \cdot M, \] and such that \[ |x_n-a| \leqq \textstyle\frac12|x_{n-1}-a|. \] We define $f_2(x)$ as follows: \[ f_2(x) =\frac1n\ \text{\textit{on the points $x_1$, $x_2, \ldots$, $x_n,\ldots$}} \] %-----File: 231.png---Folio 219------ \textit{and $f_2(x)$ is linear between the points of the sequence $x_1$, $x_2$, \ldots, $x_n$, \ldots.} Then there are values of $x$ on \correction{$\interval{x_n}{x_{n-1}}$}{$\interval{x_n}{x}_{n-1}$} such that \[ |f_1(x)|\cdot f_2(x) > {\frac2n}^{n-1} \cdot M, \] whence $f_1(x)\cdot f_2(x)$ is unbounded in the neighborhood of $a$.\footnote{% In case $\displaystyle{\mathop{L}_{x=0}} f_1(x) = \infty$, $f_2(x) = \frac{1}{\sqrt{f_1(x)}}$ or $f_2(x) = \frac{1}{\log f_1(x)}$ would satisfy the requirements of the lemma except that they need not make $\frac{f_2(x)}{x-a}$ monotonic.} Obviously $\frac{f_2(x)}{x-a}$ is monotonic increasing as $x$ approaches $a$. \end{proof} \begin{theorem}[170]\hypertarget{thm170}{} For every function $f_1(x)$ which is unbounded in every neighborhood of $x=a$ there exists a non-oscillating function $f_2(x)$ such that \[ \mathop{L}_{x\doteq a} f_1(x) \int_x^b f_2(x)dx \] exists and is finite, while \[ (x-a) \cdot f_1(x) \cdot f_2(x) \] is unbounded in the neighborhood of $x=a$. \end{theorem} \begin{proof} According to the lemma there exists a function $f_3(x)$ such that \[ \mathop{L}_{x\doteq a} f_3(x) = 0, \] while $f_3(x)\cdot f_1(x)$ is unbounded and the function \[ f_4(x) = \frac{f_3(x)}{x-a} \] is monotonic increasing as $x$ approaches $a$. Since \[ (x-a) f_4(x) \cdot f_1(x) = f_3(x) \cdot f_1(x), \] %-----File: 232.png---Folio 220------ $(x-a)\cdot f_4(x)\cdot f_1(x)$ is unbounded in the neighborhood of $x=a$. Let $x_1, \ldots, x_n, \ldots$ be a sequence of points on $\interval{a}{b}$ whose only limit point is $a$, such that $f_3(x)\cdot f_1(x)$ is unbounded on this set. In the sequence \hypertarget{seq1}{\[ (x_1-a)f_4(x_1),\quad (x_2-a)f_4(x_2),\quad \ldots,\quad (x_n-a)f_4\text{\correction{$(x_n)$,}{$(x)_n$.}} \tag{1} \]} $\displaystyle\mathop{L}_{n\doteq\infty} (x_n-a)f_4(x_n) = 0$, since $\ \displaystyle\mathop{L}_{x\doteq a} (x-a)f_4(x ) = 0$. Hence there is a value of $n$, $n_1$, such that \[ |(x_1-a) f_4(x_1 )|\geqq 2|(x_{n_1}-a) f_4(x_{n_1})|, \] and another value of $n$, $n_2$ such that \[ |(x_{n_1}-a) f_4(x_{n_1})|\geqq 2|(x_{n_2}-a) f_4(x_{n_2})|, \text{ etc.,} \] $n_{m+1}$ being so chosen that \[ |(x_{n_m}-a) f_4(x_{n_m} )|\geqq 2|(x_{n_{m+1}}-a) f_4(x_{n_{m+1}})|. \] In this manner we select from the sequence~\hyperlink{seq1}{(1)} a set of terms forming the convergent series \hypertarget{ser2}{\[ (x_1-a)f_4(x_1 ) + (x_{n_1}-a)f_4(x_{n_1}) + \ldots + (x_{n_m}-a)f_4(x_{n_m}) + \ldots. \tag{2} \]} We then obtain a function $f_2(x)$ as follows: For the set of values of $x$ \[ x_{n_{m+1}} < x \leqq x_{n_m}, \quad f_2(x) = f_4(x_{n_m}). \] Then \begin{enumerate} \item[(1)] $f_2(x)$ is non-oscillating since \[ f_4(x_{n_m}) < f_4(x_{n_{m+1}}). \] \item[(2)] $(x-a) f_2(x)\cdot f_1(x)$ is unbounded on the set $x_1$, $x_{n_1}$, $x_{n_2}, \ldots, x_{n_m}, \ldots$, since on this set \[ f_2(x) = f_4(x). \] %-----File: 233.png---Folio 221------ \item[(3)] $\qquad\displaystyle\mathop{L}_{x\doteq a} \int_x^b f_2(x)dx = \sum_{m=1}^\infty (x_{n_m}-x_{n_{m+1}}) f_4(x_{n_m})$. \end{enumerate} But the terms of this series are numerically smaller than the corresponding terms of the convergent series~\hyperlink{ser2}{(2)}. Hence \[ \mathop{L}_{x\doteq a} \int_x^b f_2(x)dx \] exists and is finite. \end{proof} Theorem~\hyperlink{thm170}{170} may be regarded as showing that \[ \mathop{L}_{x\doteq a} (x-a) f_2(x) = 0 \] is a strong necessary condition that, under the hypothesis of Theorem~\hyperlink{thm142}{142}, \[ \mathop{L}_{x\doteq a} \int_x^b f_2(x)dx \] shall exist and be finite. For, according to Theorem~\hyperlink{thm170}{170}, it is impossible to modify the function $(x-a)$ by any factor $f_1(x)$ which shall approach infinity so slowly that for every function $f_2(x)$ where \[ \mathop{L}_{x\doteq a} \int_x^b f_2(x)dx \] exists and is finite \[ \mathop{L}_{x\doteq a} (x-a) f_1(x)\cdot f_2(x) = 0.\footnotemark \] \footnotetext{% See \textsc{Pringsheim}, Mathematische Annalen, Vol.~37, pp.~591--604 (1890).} \begin{theorem}[171]\hypertarget{thm171}{} For every function $f_1(x)$ defined on the interval $\interval{a}{b}$ there exists a function $f_2(x)$ such that \begin{enumerate} \item[\textnormal{(1)}] $f_2(x)$ is continuous and does not change sign on a certain neighborhood of $x\doteq a$. %-----File: 234.png---Folio 222------ \item[\textnormal{(2)}] $\displaystyle\mathop{L}_{x\doteq a} \int_x^b f_2(x)dx$ exists and is finite. \item[\textnormal{(3)}]\hypertarget{set3}{} For $x$ on a certain set $[x']$ \[ \mathop{L}_{x\doteq a} \frac{f_1(x')}{f_2(x')} = 0. \] \end{enumerate} \end{theorem} \begin{proof} Let $x_1'$, $x_2', \ldots, x_n', \ldots$ be a set of points of the interval $\interval{a}{b}$ dense only at $a$. Let $B_1$, $B_2$, $B_3, \ldots, B_n, \ldots$ be a set of numbers such that \[ B_n\cdot n|f_1({x'}_n)|\geqq 2\cdot B_{n+1} (n+1) |f_1(x'_{n+1})|.\qquad \text{($n = 1, 2, 3,\ldots$)} \] On the $x$ axis lay off a set of segments $[\sigma_n]$ such that $\sigma_n$ is of length $B_n$ and $x_n$ is its middle point. On the segments $\sigma_n$ as bases construct isosceles triangles on the positive side of the $x$ axis whose altitudes are $n\cdot|f_1(x)|$. The measures of areas of these triangles form a convergent series. Let $f_3(x)$ be any continuous, monotonic, unbounded function such that \[ \mathop{L}_{x\doteq a} \int_x^b f_3(x)dx \] exists and is finite. We then define $f_2(x)$ as the function represented by the following curve: \begin{enumerate} \item[(1)] Those parts of the boundaries of the isosceles triangles just described which lie above the curve defined by $f_3(x)$. \item[(2)] Those parts of the curve defined by $f_3(x)$ which lie outside the triangles or on their boundary. \end{enumerate} Obviously the function so defined has the properties specified in the theorem, the points $x_1'$, $x_2', \ldots, x_n', \ldots$ being the set $[x']$ specified by \hyperlink{set3}{(3)} of the theorem. \end{proof} Theorem~\hyperlink{thm171}{171} means that from the hypothesis that the improper definite integral of $f(x)$ exists on $\interval{a}{b}$ it is impossible to obtain any conclusion whatever as to the order of infinity or the rank of infinity of $f(x)$ at $x=a$. This is what one would %-----File: 235.png---Folio 223------ expect \textit{a priori}, since the definite integral is a function of two parameters, while the necessary condition in terms of boundedness would be in terms of only one of these. \section[Existence of Improper Definite Integrals on the Infinite Interval]{Special Theorems on the Criteria of the Existence of the Improper Definite Integral on the Infinite Interval.}\hypertarget{chIXsec6}{}%[6] \begin{theorem}[172]\hypertarget{thm172}{} For every function $f_1(x)$ which is unbounded as $x$ approaches $\infty$ there exists a non-oscillating function $f_2(x)$ such that \[ \mathop{L}_{x\doteq\infty} \int_a^x f_2(x)dx \] exists and is finite, while $(x-a)f_1(x)\cdot f_2(x)$ is unbounded as $x$ approaches $\infty$. \end{theorem} \begin{proof} Obviously the lemma of Theorem~\hyperlink{thm170}{170} can be stated so as to apply to the case where $x$ approaches $\infty$ instead of $a$. If then in the proof of Theorem~\hyperlink{thm161}{161} the set of points $x_1\ldots x_n\ldots$ is so taken that \[ \mathop{L}_{n\doteq\infty} x_n=\infty \] instead of $a$, the proof of Theorem~\hyperlink{thm161}{161} applies with the exception that $f_2(x)$ is non-oscillating \textit{decreasing} instead of non-oscillating \textit{increasing}. \end{proof} \begin{theorem}[173]\hypertarget{thm173}{} For every function $f_1(x)$ defined on the interval $\interval{a}{\infty}$ there exists a function $f_2(x)$ such that \begin{enumerate} \item[\textnormal{(1)}] $f_2(x)$ is continuous and does not change sign for $x$ greater than a certain fixed number. \item[\textnormal{(2)}] \[ \mathop{L}_{x\doteq\infty} \int_x^a f_2(x)dx \] exists and is finite. %-----File: 236.png---Folio 224------ \item[\textnormal{(3)}] For $x$ on a certain set $[x']$ \[ \mathop{L}_{x\doteq\infty} \frac{f_1(x')}{f_2(x')} = 0. \] \end{enumerate} \end{theorem} \begin{proof} Such a function $f_2(x)$ may be defined in a manner analogous to that of the proof of Theorem~\hyperlink{thm171}{171}. The remarks as to the meaning of Theorems \hyperlink{thm170}{170} and \hyperlink{thm171}{171} apply with obvious modifications to Theorems \hyperlink{thm172}{172} and \hyperlink{thm173}{173}. \end{proof} \backmatter %-----File: 237.png---Folio 225------ %\chapter*{INDEX} {\setlength{\columnsep}{1cm} \printindex} %-----File: 238.png---Folio 226------ %-----File: 239.png---Folio 227------ %-----File: 240.png---Folio 228------ %[Blank Page] %-----File: 241.png---Index 1-------- \pagestyle{plain} \newpage\setcounter{page}{1} {\centering {\sffamily\Huge SHORT-TITLE CATALOGUE\\[0.5ex]} \small OF THE\\[0.5ex] \LARGE PUBLICATIONS\\[0.5ex] \small OF\\ \Huge JOHN WILEY \& SONS,\\[0.5ex] \sffamily\sc\large New York.\\[2ex] \Large\sc London: CHAPMAN \& HALL, Limited.\\[0.5ex] \rule[0.5ex]{2cm}{.2pt}\\ \normalfont\normalsize ARRANGED UNDER SUBJECTS.\\ \rule[0.5ex]{2cm}{.2pt}\\} \footnotesize Descriptive circulars sent on application. Books marked with an asterisk (*) are sold at \textit{net} prices only. All books are bound in cloth unless otherwise stated. \bigskip \begin{center} \rule[0.5ex]{2cm}{.2pt} \end{center} \bigskip \footnotesize \begin{longtable}{@{}l@{ }r@{}} \multicolumn{2}{c}{\large AGRICULTURE.}\\[1em] \nopagebreak Armsby's Manual of Cattle-feeding.\dotfill\ldots 12mo, &\$1\ 75\\ \indent Principles of Animal Nutrition.\dotfill\ldots 8vo, &4\ 00\\ Budd and Hansen's American Horticultural Manual:\\ \indent Part I\@. Propagation, Culture, and Improvement.\dotfill\ldots 12mo, &1\ 50\\ \indent Part II\@. Systematic Pomology.\dotfill 12mo, &1\ 50\\ Downing's Fruits and Fruit-trees of America.\dotfill\ldots 8vo, &5\ 00\\ Elliott's Engineering for Land Drainage.\dotfill\ldots 12mo, &1\ 50\\ \indent Practical Farm Drainage.\dotfill\ldots 12mo, &1\ 00\\ Graves's Forest Mensuration.\dotfill\ldots 8vo, &4\ 00\\ Green's Principles of American Forestry.\dotfill\ldots 12mo, &1\ 50\\ Grotenfelt's Principles of Modern Dairy Practice. (Woll.)\dotfill 12mo, &2\ 00\\ Kemp's Landscape Gardening.\dotfill\ldots 12mo, &2\ 50\\ Maynard's Landscape Gardening as Applied to Home Decoration.\dotfill\ldots 12mo, &1\ 50\\ * McKay and Larsen's Principles and Practice of Butter-making.\dotfill\ldots 8vo, &1\ 50\\ Sanderson's Insects Injurious to Staple Crops.\dotfill\ldots 12mo, &1\ 50\\ \indent Insects Injurious to Garden Crops. (In preparation.)\\ \indent Insects Injuring Fruits. (In preparation.)\\ Stockbridge's Rocks and Soils.\dotfill\ldots 8vo, &2\ 50\\ Winton's Microscopy of Vegetable Foods.\dotfill\ldots 8vo, &7\ 50\\ Woll's Handbook for Farmers and Dairymen.\dotfill\ldots 16mo, &1\ 50\\[3em] \multicolumn{2}{c}{\large ARCHITECTURE.}\\[1em] \nopagebreak Baldwin's Steam Heating for Buildings.\dotfill\ldots 12mo, &2\ 50\\ Bashore's Sanitation of a Country House.\dotfill\ldots 12mo. &1\ 00\\ Berg's Buildings and Structures of American Railroads.\dotfill\ldots 4to, &5\ 00\\ Birkmire's Planning and Construction of American Theatres.\dotfill\ldots 8vo, &3\ 00\\ \indent Architectural Iron and Steel.\dotfill\ldots 8vo, &3\ 50\\ \indent Compound Riveted Girders as Applied in Buildings.\dotfill\ldots 8vo, &2\ 00\\ \indent Planning and Construction of High Office Buildings.\dotfill\ldots 8vo, &3\ 50\\ \indent Skeleton Construction in Buildings.\dotfill\ldots 8vo, &3\ 00\\ Brigg's Modern American School Buildings.\dotfill\ldots 8vo, &4\ 00\\ %-----File: 242.png---Index 2-------- Carpenter's Heating and Ventilating of Buildings.\dotfill\ldots 8vo, &4\ 00\\ Freitag's Architectural Engineering.\dotfill\ldots 8vo, &3\ 50\\ \nopagebreak \indent Fireproofing of Steel Buildings.\dotfill\ldots 8vo, &2\ 50\\ French and Ives's Stereotomy.\dotfill\ldots 8vo, &2\ 50\\ Gerhard's Guide to Sanitary House-inspection.\dotfill\ldots 16mo, &1\ 00\\ \nopagebreak \indent Theatre Fires and Panics.\dotfill\ldots 12mo, &1\ 50\\ * Greene's Structural Mechanics.\dotfill\ldots 8vo, &2\ 50\\ Holly's Carpenters' and Joiners' Handbook.\dotfill\ldots 18mo, &75\\ Johnson's Statics by Algebraic and Graphic Methods.\dotfill\ldots 8vo, &2\ 00\\ Kidder's Architects' and Builders' Pocket-book.\\ \nopagebreak \indent\indent Rewritten Edition.\dotfill\ldots 16mo, mor., &5\ 00\\ Merrill's Stones for Building and Decoration.\dotfill\ldots 8vo, &5\ 00\\ \nopagebreak \indent Non-metallic Minerals: Their Occurrence and Uses.\dotfill\ldots 8vo, &4\ 00\\ Monckton's Stair-building.\dotfill\ldots 4to, &4\ 00\\ Patton's Practical Treatise on Foundations.\dotfill\ldots 8vo, &5\ 00\\ Peabody's Naval Architecture.\dotfill\ldots 8vo, &7\ 50\\ Rice's Concrete-block Manufacture.\dotfill\ldots 8vo, &2\ 00\\ Richey's Handbook for Superintendents of Construction.\dotfill\ldots 16mo, mor., &4\ 00\\ \makebox[0pt]{\hspace{.5ex} *}\indent Building Mechanics' Ready Reference Book. Carpenters'\\ \nopagebreak \indent\indent and Woodworkers' Edition.\dotfill\ldots 16mo, morocco, &1\ 50\\ Sabin's Industrial and Artistic Technology of Paints and Varnish.\dotfill\ldots 8vo, &3\ 00\\ Siebert and Biggin's Modern Stone-cutting and Masonry.\dotfill\ldots 8vo, &1\ 50\\ Snow's Principal Species of Wood.\dotfill\ldots 8vo, &3\ 50\\ Sondericker's Graphic Statics with Applications to Trusses, Beams,\\ \nopagebreak \indent\indent and Arches.\dotfill\ldots 8vo, &2\ 00\\ Towne's Locks and Builders' Hardware.\dotfill\ldots 18mo, morocco, &3\ 00\\ Wait's Engineering and Architectural Jurisprudence.\dotfill\ldots 8vo, &6\ 00\\ \nopagebreak \hfill Sheep, &6\ 50\\ \indent Law of Operations Preliminary to Construction in Engineering\\ \indent\indent and Architecture.\dotfill\ldots 8vo, &5\ 00\\ \nopagebreak \hfill Sheep, &5\ 50\\ \indent Law of Contracts.\dotfill\ldots 8vo, &3\ 00\\ Wood's Rustless Coatings: Corrosion and Electrolysis of Iron\\ \nopagebreak \indent\indent and Steel.\dotfill\ldots 8vo, &4\ 00\\ Worcester and Atkinson's Small Hospitals, Establishment and\\ \nopagebreak \indent\indent Maintenance, Suggestions for Hospital Architecture,\\ \nopagebreak \indent\indent with Plans for a Small Hospital.\dotfill\ldots 12mo, &1\ 25\\ The World's Columbian Exposition of 1893.\dotfill\ldots Large 4to, &1\ 00\\[3em] \multicolumn{2}{c}{\large ARMY AND NAVY.}\\[1em] \nopagebreak Bernadou's Smokeless Powder, Nitro-cellulose, and the Theory of the \\ \nopagebreak \indent\indent Cellulose Molecule.\dotfill\ldots 12mo, &2\ 50\\ * Bruff's Text-book Ordnance and Gunnery.\dotfill\ldots 8vo, &6\ 00\\ Chase's Screw Propellers and Marine Propulsion.\dotfill\ldots 8vo, &3\ 00\\ Cloke's Gunner's Examiner.\dotfill\ldots 8vo, &1\ 50\\ Craig's Azimuth.\dotfill\ldots 4to, &3\ 50\\ Crehore and Squier's Polarizing Photo-chronograph.\dotfill\ldots 8vo, &3\ 00\\ * Davis's Elements of Law.\dotfill\ldots 8vo, &2\ 50\\ \nopagebreak \makebox[0pt]{\hspace{.5ex} *}\indent Treatise on the Military Law of United States.\dotfill\ldots 8vo, &7\ 00\\ \nopagebreak \hfill Sheep, &7\ 50\\ De Brack's Cavalry Outposts Duties. (Carr.)\dotfill\ldots 24mo, morocco, &2\ 00\\ Dietz's Soldier's First Aid Handbook.\dotfill\ldots 16mo, morocco, &1\ 25\\ * Dudley's Military Law and the Procedure\\ \nopagebreak \indent\indent of Courts-martial.\dotfill\ldots Large 12mo, & 2\ 50\\ Durand's Resistance and Propulsion of Ships.\dotfill\ldots 8vo, &5\ 00\\ * Dyer's Handbook of Light Artillery.\dotfill\ldots 12mo, &3\ 00\\ Eissler's Modern High Explosives.\dotfill\ldots 8vo, &4\ 00\\ * Fiebeger's Text-book on Field Fortification.\dotfill\ldots Small 8vo, &2\ 00\\ Hamilton's The Gunner's Catechism.\dotfill\ldots 18mo, &1\ 00\\ * Hoff's Elementary Naval Tactics.\dotfill\ldots 8vo, &1\ 50\\ %-----File: 243.png---Index 3-------- Ingalls's Handbook of Problems in Direct Fire.\dotfill\ldots 8vo, &4\ 00\\ \nopagebreak \makebox[0pt]{\hspace{.5ex} *}\indent Ballistic Tables.\dotfill\ldots 8vo, &1\ 50\\ * Lyons's Treatise on Electromagnetic Phenomena. Vols.~I.\\ \nopagebreak \indent\indent and II.\dotfill\ldots 8vo, each, &6\ 00\\ * Mahan's Permanent Fortifications. (Mercur.)\dotfill 8vo, half morocco, &7\ 50\\ Manual for Courts-martial.\dotfill\ldots 16mo, morocco, &1\ 50\\ * Mercur's Attack of Fortified Places.\dotfill\ldots 12mo, &2\ 00\\ \nopagebreak \makebox[0pt]{\hspace{.5ex} *}\indent Elements of the Art of War.\dotfill\ldots 8vo, &4\ 00\\ Metcalf's Cost of Manufactures---And the Administration\\ \nopagebreak \indent\indent of Workshops.\dotfill\ldots 8vo, &5\ 00\\ \nopagebreak \makebox[0pt]{\hspace{.5ex} *}\indent Ordnance and Gunnery. 2 vols.\dotfill\ldots 12mo, &5 00\\ Murray's Infantry Drill Regulations.\dotfill\ldots 18mo, paper, &10\\ Nixon's Adjutants' Manual.\dotfill\ldots 24mo, &1\ 00\\ Peabody's Naval Architecture.\dotfill\ldots 8vo, &7\ 50\\ * Phelps's Practical Marine Surveying.\dotfill\ldots 8vo, &2\ 50\\ Powell's Army Officer's Examiner.\dotfill\ldots 12mo, &4\ 00\\ Sharpe's Art of Subsisting Armies in War.\dotfill\ldots 18mo, morocco, &1\ 50\\ * Tupes and Poole's Manual of Bayonet Exercises and Musketry Fencing.\\ \nopagebreak \hfill 24mo, leather, &50\\ * Walke's Lectures on Explosives.\dotfill\ldots 8vo, &4\ 00\\ Weaver's Military Explosives.\dotfill\ldots 8vo, &3\ 00\\ * Wheeler's Siege Operations and Military Mining.\dotfill\ldots 8vo, &2\ 00\\ Winthrop's Abridgment of Military Law.\dotfill\ldots 12mo, &2\ 50\\ Woodhull's Notes on Military Hygiene.\dotfill\ldots 16mo, &1\ 50\\ Young's Simple Elements of Navigation.\dotfill\ldots 16mo, morocco, &2\ 00\\[3em] \multicolumn{2}{c}{\large ASSAYING.}\\[1em] \nopagebreak Fletcher's Practical Instructions in Quantitative Assaying with\\ \nopagebreak \indent\indent the Blowpipe.\dotfill\ldots 12mo, morocco, &1\ 50\\ Furman's Manual of Practical Assaying.\dotfill\ldots 8vo, &3\ 00\\ Lodge's Notes on Assaying and Metallurgical Laboratory\\ \nopagebreak \indent\indent Experiments.\dotfill\ldots 8vo,&3\ 00\\ Low's Technical Methods of Ore Analysis.\dotfill\ldots 8vo, &3\ 00\\ Miller's Manual of Assaying.\dotfill\ldots 12mo, &1\ 00\\ \nopagebreak \indent Cyanide Process.\dotfill\ldots 12mo, &1\ 00\\ Minet's Production of Aluminum and its Industrial Use. (Waldo.)\dotfill\ldots 12mo, &2\ 50\\ O'Driscoll's Notes on the Treatment of Gold Ores.\dotfill\ldots 8vo, &2\ 00\\ Ricketts and Miller's Notes on Assaying.\dotfill\ldots 8vo, &3\ 00\\ Robine and Lenglen's Cyanide Industry. (Le Clerc.)\dotfill\ldots 8vo, &4\ 00\\ Ulke's Modern Electrolytic Copper Refining.\dotfill\ldots 8vo, &3\ 00\\ Wilson's Cyanide Processes.\dotfill\ldots 12mo, &1\ 50\\ \nopagebreak \indent Chlorination Process.\dotfill\ldots 12mo, &1\ 50\\[3em] \multicolumn{2}{c}{\large ASTRONOMY.}\\[1em] \nopagebreak Comstock's Field Astronomy for Engineers.\dotfill\ldots 8vo, &2\ 50\\ Craig's Azimuth.\dotfill\ldots 4to, &3\ 50\\ Crandall's Text-book on Geodesy and Least Squares.\dotfill\ldots 8vo, & 3\ 00\\ Doolittle's Treatise on Practical Astronomy.\dotfill\ldots 8vo, &4\ 00\\ Gore's Elements of Geodesy.\dotfill\ldots 8vo, &2\ 50\\ Hayford's Text-book of Geodetic Astronomy.\dotfill\ldots 8vo, &3\ 00\\ Merriman's Elements of Precise Surveying and Geodesy.\dotfill\ldots 8vo, &2\ 50\\ * Michie and Harlow's Practical Astronomy.\dotfill\ldots 8vo, &3\ 00\\ * White's Elements of Theoretical and Descriptive Astronomy.\dotfill\ldots 12mo, &\correction{2}{}\ 00\\[3em] \multicolumn{2}{c}{\large BOTANY.}\\[1em] \nopagebreak Davenport's Statistical Methods, with Special Reference\\ \nopagebreak \indent\indent to Biological Variation.\dotfill\ldots 16mo, morocco, &1\ 25\\ Thom\'e and Bennett's Structural and Physiological Botany.\dotfill\ldots 16mo, &2\ 25\\ Westermaier's Compendium of General Botany. (Schneider.)\dotfill 8vo, &2\ 00\\[3em] %-----File: 244.png---Index 4-------- \multicolumn{2}{c}{\large CHEMISTRY.}\\[1em] \nopagebreak * Abegg's Theory of Electrolytic Dissociation. (Von Ende.)\dotfill\ldots 12mo, & 1\ 25 \\ Adriance's Laboratory Calculations and Specific Gravity Tables.\dotfill\ldots 12mo, &1\ 25\\ Alexeyeff's General Principles of Organic Synthesis. (Matthews.)\dotfill\ldots 8vo, &3\ 00\\ Allen's Tables for Iron Analysis.\dotfill\ldots 8vo, &3\ 00\\ Arnold's Compendium of Chemistry. (Mandel.)\dotfill\ldots Small 8vo, &3\ 50\\ Austen's Notes for Chemical Students.\dotfill\ldots 12mo, &1\ 50\\ Bernadou's Smokeless Powder.---Nitro-cellulose, and Theory of\\ \nopagebreak \indent\indent the Cellulose Molecule.\dotfill\ldots 12mo, &2\ 50\\ * Browning's Introduction to the Rarer Elements.\dotfill\ldots 8vo, &1\ 50\\ Brush and Penfield's Manual of Determinative Mineralogy.\dotfill\ldots 8vo, &4\ 00\\ * Claassen's Beet-sugar Manufacture. (Hall and Rolfe.)\dotfill\ldots 8vo, &3\ 00\\ Classen's Quantitative Chemical Analysis by\\ \nopagebreak \indent\indent Electrolysis. (Boltwood.)\dotfill\ldots 8vo, &3\ 00\\ Cohn's Indicators and Test-papers.\dotfill\ldots 12mo, &2\ 00\\ \nopagebreak \indent Tests and Reagents.\dotfill\ldots 8vo, &3\ 00\\ Crafts's Short Course in Qualitative Chemical\\ \nopagebreak \indent\indent Analysis. (Schaeffer.)\dotfill\ldots 12mo, &1\ 50\\ * Danneel's Electrochemistry. (Merriam.)\dotfill\ldots 12mo, & 1\ 25\\ Dolezalek's Theory of the Lead Accumulator (Storage Battery).\\ \nopagebreak \indent (Von Ende.)\dotfill\ldots 12mo, &2\ 50\\ Drechsel's Chemical Reactions. (Merrill.)\dotfill\ldots 12mo, &1\ 25\\ Duhem's Thermodynamics and Chemistry. (Burgess.)\dotfill\ldots 8vo, &4\ 00\\ Eissler's Modern High Explosives.\dotfill\ldots 8vo, &4\ 00\\ Effront's Enzymes and their Applications. (Prescott.)\dotfill\ldots 8vo, &3\ 00\\ Erdmann's Introduction to Chemical Preparations. (Dunlap.)\dotfill\ldots 12mo, &1\ 25\\ Fletcher's Practical Instructions in Quantitative Assaying with\\ \nopagebreak \indent\indent the Blowpipe.\dotfill\ldots 12mo, morocco, &1\ 50\\ Fowler's Sewage Works Analyses.\dotfill\ldots 12mo, &2\ 00\\ Fresenius's Manual of Qualitative Chemical Analysis. (Wells.)\dotfill\ldots 8vo, &5\ 00\\ \indent Manual of Qualitative Chemical Analysis. Part I.\\ \nopagebreak \indent\indent Descriptive. (Wells.)\dotfill\ldots 8vo, &3\ 00\\ \indent System of Instruction in Quantitative Chemical Analysis. (Cohn.)\\ \nopagebreak \indent\indent 2 vols.\dotfill\ldots 8vo, &12\ 50\\ Fuertes's Water and Public Health.\dotfill\ldots 12mo, &1\ 50\\ Furman's Manual of Practical Assaying.\dotfill\ldots 8vo, &3\ 00\\ * Getman's Exercises in Physical Chemistry.\dotfill\ldots 12mo, &2\ 00\\ Gill's Gas and Fuel Analysis for Engineers.\dotfill\ldots 12mo, &1\ 25\\ * Gooch and Browning's Outlines of Qualitative\\ \nopagebreak \indent\indent Chemical Analysis.\dotfill\ldots Small 8vo, & 1\ 25\\ Grotenfelt's Principles of Modern Dairy Practice. (Woll.)\dotfill\ldots 12mo, &2\ 00\\ Groth's Introduction to Chemical Crystallography (Marshall)\dotfill\ldots 12mo, &1\ 25\\ Hammarsten's Text-book of Physiological Chemistry. (Mandel.)\dotfill\ldots 8vo, &4\ 00\\ Helm's Principles of Mathematical Chemistry. (Morgan.)\dotfill\ldots 12mo, &1\ 50\\ Hering's Ready Reference Tables (Conversion Factors)\dotfill\ldots 16mo, morocco, &2\ 50\\ Hind's Inorganic Chemistry.\dotfill\ldots 8vo, &3\ 00\\ \makebox[0pt]{\hspace{.5ex} *}\indent Laboratory Manual for Students.\dotfill\ldots 12mo, &1\ 00\\ Holleman's Text-book of Inorganic Chemistry. (Cooper.)\dotfill\ldots 8vo, &2\ 50\\ \nopagebreak \indent Text-book of Organic Chemistry. (Walker and Mott.)\dotfill\ldots 8vo, &2\ 50\\ \makebox[0pt]{\hspace{.5ex} *}\indent Laboratory Manual of Organic Chemistry. (Walker.)\dotfill\ldots 12mo, &1\ 00\\ Hopkins's Oil-chemists' Handbook.\dotfill\ldots 8vo, &3\ 00\\ Iddings's Rock Minerals.\dotfill\ldots 8vo, & 5\ 00\\ Jackson's Directions for Laboratory Work in Physiological\\ \nopagebreak \indent\indent Chemistry.\dotfill\ldots 8vo, &1\ 25\\ Keep's Cast Iron.\dotfill\ldots 8vo, &2\ 50\\ Ladd's Manual of Quantitative Chemical Analysis.\dotfill\ldots 12mo, &1\ 00\\ Landauer's Spectrum Analysis. (Tingle.)\dotfill\ldots 8vo, &3\ 00\\ * Langworthy and Austen. The Occurrence of Aluminium in Vegetable\\ \nopagebreak \indent\indent Products, Animal Products, and Natural Waters.\dotfill\ldots 8vo, &2\ 00\\ Lassar-Cohn's Application of Some General Reactions to Investigations\\ \nopagebreak \indent\indent in Organic Chemistry. (Tingle.)\dotfill\ldots 12mo, &1\ 00\\ Leach's The Inspection and Analysis of Food with Special Reference \\ \nopagebreak \indent\indent to State Control.\dotfill\ldots 8vo, &7\ 50\\ L\"{o}b's Electrochemistry of Organic Compounds. (Lorenz.)\dotfill\ldots 8vo, &3\ 00\\ %-----File: 245.png---Index 5-------- Lodge's Notes on Assaying and Metallurgical Laboratory\\ \nopagebreak \indent\indent Experiments.\dotfill\ldots 8vo, &3\ 00\\ Low's Technical Method of Ore Analysis.\dotfill\ldots 8vo, &3\ 00\\ Lunge's Techno-chemical Analysis. (Cohn.)\dotfill\ldots 12mo, &1\ 00\\ * McKay and Larsen's Principles and Practice of Butter-making.\dotfill\ldots 8vo, &1\ 50\\ Mandel's Handbook for Bio-chemical Laboratory.\dotfill\ldots 12mo, &1\ 50\\ * Martin's Laboratory Guide to Qualitative Analysis with\\ \nopagebreak \indent\indent the Blowpipe.\dotfill\ldots 12mo, &60\\ Mason's Water-supply. (Considered Principally from a Sanitary\\ \nopagebreak \indent\indent Standpoint.) 3d Edition, Rewritten.\dotfill\ldots 8vo, &4\ 00\\ \nopagebreak \indent Examination of Water. (Chemical and Bacteriological.)\dotfill\ldots 12mo, &1\ 25\\ Matthew's The Textile Fibres.\dotfill\ldots 8vo, &3\ 50\\ Meyer's Determination of Radicles in Carbon Compounds.\\ \nopagebreak \indent\indent (Tingle.)\dotfill\ldots 12mo, &1\ 00\\ Miller's Manual of Assaying.\dotfill\ldots 12mo, &1\ 00\\ \nopagebreak \indent Cyanide Process.\dotfill\ldots 12mo, &1\ 00\\ Minet's Production of Aluminum and its Industrial Use. (Waldo.)\dotfill\ldots 12mo, &2\ 50\\ Mixter's Elementary Text-book of Chemistry.\dotfill\ldots 12mo, &1\ 50\\ Morgan's An Outline of the Theory of Solutions and its Results.\dotfill\ldots 12mo, &1\ 00\\ \nopagebreak \indent Elements of Physical Chemistry.\dotfill\ldots 12mo, &3\ 00\\ \nopagebreak \indent * Physical Chemistry for Electrical Engineers.\dotfill\ldots 12mo, &1\ 50\\ Morse's Calculations used in Cane-sugar Factories.\dotfill\ldots 16mo, morocco, &1\ 50\\ * Muir's History of Chemical Theories and Laws.\dotfill\ldots 8vo, & 4\ 00\\ Mulliken's General Method for the Identification of Pure\\ \nopagebreak \indent\indent Organic Compounds. Vol.~I.\dotfill\ldots Large 8vo, &5\ 00\\ O'Brine's Laboratory Guide in Chemical Analysis.\dotfill\ldots 8vo, &2\ 00\\ O'Driscoll's Notes on the Treatment of Gold Ores.\dotfill\ldots 8vo, &2\ 00\\ Ostwald's Conversations on Chemistry. Part One. (Ramsey.)\dotfill\ldots 12mo, &1\ 50\\ \nopagebreak \phantom{Ostw}\makebox[0pt]{``}\phantom{ald's Conve}\makebox[0pt]{``}\phantom{rsations }\makebox[0pt]{\; ``}\phantom{on Chem}\makebox[0pt]{``}\phantom{istry.} Part Two. (Turnbull.)..\dotfill 12mo, &2\ 00\\ * Pauli's Physical Chemistry in the Service of Medicine. (Fischer.)\dotfill\ldots 12mo, & 1\ 25\\ * Penfield's Notes on Determinative Mineralogy and Record\\ \nopagebreak \indent\indent of Mineral Tests.\dotfill\ldots 8vo, paper, &50\\ Pictet's The Alkaloids and their Chemical Constitution. (Biddle.)\dotfill\ldots 8vo, &5\ 00\\ Pinner's Introduction to Organic Chemistry. (Austen.)\dotfill\ldots 12mo &1\ 50\\ Poole's Calorific Power of Fuels.\dotfill\ldots 8vo, &3\ 00\\ Prescott and Winslow's Elements of Water Bacteriology, with Special \\ \nopagebreak \indent\indent Reference to Sanitary Water Analysis.\dotfill\ldots 12mo, &1\ 25\\ * Reisig's Guide to Piece-dyeing.\dotfill\ldots 8vo, &25\ 00\\ Richards and Woodman's Air, Water, and Food from\\ \nopagebreak \indent\indent a Sanitary Standpoint.\dotfill\ldots 8vo, &2\ 00\\ Ricketts and Russell's Skeleton Notes upon Inorganic Chemistry.\\ \nopagebreak \indent\indent (Part I\@. Non-metallic Elements.)\dotfill\ldots 8vo, morocco, &75\\ Ricketts and Miller's Notes on Assaying.\dotfill\ldots 8vo, &3\ 00\\ Rideal's Sewage and the Bacterial Purification of Sewage.\dotfill\ldots 8vo, &3\ 50\\ \nopagebreak \indent Disinfection and the Preservation of Food.\dotfill\ldots 8vo, &4\ 00\\ Riggs's Elementary Manual for the Chemical Laboratory.\dotfill\ldots 8vo, &1\ 25\\ Robine and Lenglen's Cyanide Industry. (Le Clerc.)\dotfill\ldots 8vo, &4\ 00\\ Ruddiman's Incompatibilities in Prescriptions.\dotfill\ldots 8vo, &2\ 00\\ \makebox[0pt]{\hspace{.5ex} *}\indent Whys in Pharmacy.\dotfill\ldots 12mo,&1\ 00\\ Sabin's Industrial and Artistic Technology of Paints and Varnish.\dotfill\ldots 8vo, &3\ 00\\ Salkowski's Physiological and Pathological Chemistry. (Orndorff.)\dotfill\ldots 8vo, &2\ 50\\ Schimpf's Text-book of Volumetric Analysis.\dotfill\ldots 12mo,&2\ 50\\ \nopagebreak \indent Essentials of Volumetric Analysis.\dotfill\ldots 12mo,&1\ 25\\ \makebox[0pt]{\hspace{.5ex} *}\indent Qualitative Chemical Analysis.\dotfill\ldots 8vo, &1\ 25\\ Smith's Lecture Notes on Chemistry for Dental Students.\dotfill\ldots 8vo, &2\ 50\\ Spencer's Handbook for Chemists of Beet-sugar Houses.\dotfill\ldots 16mo, morocco, &3\ 00\\ \nopagebreak \indent Handbook for Cane Sugar Manufacturers.\dotfill\ldots 16mo, morocco, &3\ 00\\ Stockbridge's Rocks and Soils.\dotfill\ldots 8vo, &2\ 50\\ * Tillman's Elementary Lessons in Heat.\dotfill\ldots 8vo, &1\ 50\\ \makebox[0pt]{\hspace{.5ex} *}\indent Descriptive General Chemistry.\dotfill\ldots 8vo, &3\ 00\\ Treadwell's Qualitative Analysis. (Hall.)\dotfill\ldots 8vo, &3\ 00\\ \nopagebreak \indent Quantitative Analysis. (Hall.)\dotfill\ldots 8vo, &4\ 00\\ Turneaure and Russell's Public Water-supplies.\dotfill\ldots 8vo, &5\ 00\\ %-----File: 246.png---Index 6-------- Van Deventer's Physical Chemistry for Beginners. (Boltwood.)\dotfill\ldots 12mo, &1\ 50\\ * Walke's Lectures on Explosives.\dotfill\ldots 8vo, &4\ 00\\ Ware's Beet-sugar Manufacture and Refining.\dotfill\ldots Small 8vo, cloth, &4\ 00\\ Washington's Manual of the Chemical Analysis of Rocks.\dotfill\ldots 8vo, &2\ 00\\ Weaver's Military Explosives.\dotfill\ldots 8vo, &3\ 00 \\ Wehrenfennig's Analysis and Softening of Boiler Feed-Water.\dotfill\ldots 8vo, &4\ 00 \\ Wells's Laboratory Guide in Qualitative Chemical Analysis.\dotfill\ldots 8vo, &1\ 50 \\ \nopagebreak \indent Short Course in Inorganic Qualitative Chemical Analysis for \\ \nopagebreak \indent\indent Engineering Students.\dotfill\ldots 12mo, &1\ 50 \\ \nopagebreak \indent Text-book of Chemical Arithmetic.\dotfill\ldots 12mo, &1\ 25 \\ Whipple's Microscopy of Drinking-water.\dotfill\ldots 8vo, &3\ 50 \\ Wilson's Cyanide Processes.\dotfill\ldots 12mo, &1\ 50 \\ \nopagebreak \indent Chlorination Process.\dotfill\ldots 12mo, &1\ 50 \\ Winton's Microscopy of Vegetable Foods.\dotfill\ldots 8vo, &7\ 50 \\ Wulling's Elementary Course in Inorganic, Pharmaceutical, and Medical \\ \nopagebreak \indent\indent Chemistry.\dotfill\ldots 12mo, &2\ 00 \\[3em] \multicolumn{2}{c}{\large CIVIL ENGINEERING.}\\[1em] \nopagebreak \multicolumn{2}{c}{BRIDGES AND ROOFS\@. HYDRAULICS\@. MATERIALS OF ENGINEERING.}\\ \nopagebreak \multicolumn{2}{c}{RAILWAY ENGINEERING.}\\[1em] \nopagebreak Baker's Engineers' Surveying Instruments.\dotfill\ldots 12mo, &3\ 00\\ Bixby's Graphical Computing Table.\dotfill\ldots Paper $19\frac12 \times 24\frac14$ inches &25 \\ Breed and Hosmer's Principles and Practice of Surveying.\dotfill\ldots 8vo, & 3\ 00\\ * Burr's Ancient and Modern Engineering and\\ \nopagebreak \indent\indent the Isthmian Canal.\dotfill\ldots 8vo, &3 50 \\ Comstock's Field Astronomy for Engineers.\dotfill\ldots 8vo, &2 50 \\ Crandall's Text-book on Geodesy and Least Squares.\dotfill\ldots 8vo, & 3\ 00\\ Davis's Elevation and Stadia Tables.\dotfill\ldots 8vo, &1 00 \\ Elliott's Engineering for Land Drainage.\dotfill\ldots 12mo, &1 50 \\ \nopagebreak \indent Practical Farm Drainage.\dotfill\ldots 12mo, &1 00 \\ * Fiebeger's Treatise on Civil Engineering.\dotfill\ldots 8vo, &5 00 \\ Flemer's Phototopographic Methods and Instruments.\dotfill\ldots 8vo, &5 00 \\ Folwell's Sewerage. (Designing and Maintenance.)\dotfill\ldots 8vo, &3 00 \\ Freitag's Architectural Engineering. 2d Edition, Rewritten.\dotfill\ldots 8vo, &3 50 \\ French and Ives's Stereotomy.\dotfill\ldots 8vo, &2 50 \\ Goodhue's Municipal Improvements.\dotfill\ldots 12mo, &1 75 \\ Gore's Elements of Geodesy.\dotfill\ldots 8vo, &2 50 \\ Hayford's Text-book of Geodetic Astronomy.\dotfill\ldots 8vo, &3 00 \\ Hering's Ready Reference Tables (Conversion Factors)\dotfill\ldots 16mo, morocco, &2 50 \\ Howe's Retaining Walls for Earth.\dotfill\ldots 12mo, &1 25 \\ * Ives's Adjustments of the Engineer's Transit and Level.\dotfill\ldots 16mo, Bds, &25 \\ Ives and Hilts's Problems in Surveying.\dotfill\ldots 16mo, morocco, &1 50 \\ Johnson's (J.~B.) Theory and Practice of Surveying.\dotfill\ldots Small 8vo, &4 00 \\ Johnson's (L.~J.) Statics by Algebraic and Graphic Methods.\dotfill\ldots 8vo, &2 00\\ Laplace's Philosophical Essay on Probabilities (Truscott\\ \nopagebreak \indent\indent and Emory.)\dotfill\ldots 12mo, &2 00 \\ Mahan's Treatise on Civil Engineering. (1873.) (Wood.)\dotfill\ldots 8vo, &5 00 \\ \makebox[0pt]{\hspace{.5ex} *}\indent Descriptive Geometry.\dotfill\ldots 8vo, &1 50 \\ Merriman's Elements of Precise Surveying and Geodesy.\dotfill\ldots 8vo, &2 50 \\ Merriman and Brooks's Handbook for Surveyors.\dotfill\ldots 16mo, morocco, &2 00 \\ Nugent's Plane Surveying.\dotfill\ldots 8vo, &3 50 \\ Ogden's Sewer Design.\dotfill\ldots 12mo, &2 00 \\ Parsons's Disposal of Municipal Refuse.\dotfill\ldots 8vo, &2 00 \\ Patton's Treatise on Civil Engineering.\dotfill\ldots 8vo half leather, &7 50 \\ Reed's Topographical Drawing and Sketching.\dotfill\ldots 4to, &5 00 \\ Rideal's Sewage and the Bacterial Purification of Sewage.\dotfill\ldots 8vo, &3 50 \\ Siebert and Biggin's Modern Stone-cutting and Masonry.\dotfill\ldots 8vo, &1 50 \\ %-----File: 247.png---Index 7-------- Smith's Manual of Topographical Drawing. (McMillan.)\dotfill\ldots 8vo, &2 50 \\ Sondericker's Graphic Statics, with Applications to Trusses, Beams, \\ \nopagebreak \indent\indent and Arches.\dotfill\ldots 8vo, &2 00 \\ Taylor and Thompson's Treatise on Concrete, Plain and Reinforced.\dotfill\ldots 8vo, & 5\ 00\\ * Trautwine's Civil Engineer's Pocket-book.\dotfill\ldots 16mo, morocco, &5\ 00\\ Venable's Garbage Crematories in America.\dotfill\ldots 8vo, &2\ 00\\ Wait's Engineering and Architectural Jurisprudence.\dotfill\ldots 8vo, &6\ 00\\ \nopagebreak \hfill Sheep, &6\ 50\\ \indent Law of Operations Preliminary to Construction in Engineering\\ \nopagebreak \indent\indent and Architecture.\dotfill\ldots 8vo, &5\ 00\\ \nopagebreak \hfill Sheep, &5\ 50\\ \indent Law of Contracts.\dotfill\ldots 8vo, &3\ 00\\ Warren's Stereotomy---Problems in Stone-cutting.\dotfill\ldots 8vo, &2\ 50\\ Webb's Problems in the Use and Adjustment\\ \nopagebreak \indent\indent of Engineering Instruments.\dotfill\ldots 16mo, morocco, &1\ 25\\ Wilson's Topographic Surveying.\dotfill\ldots 8vo, &3\ 50\\[2em] \multicolumn{2}{c}{BRIDGES AND ROOFS.}\\[1em] \nopagebreak Boller's Practical Treatise on the Construction of Iron\\ \nopagebreak \indent\indent Highway Bridges.\dotfill\ldots 8vo, &2\ 00\\ \makebox[0pt]{\hspace{.5ex} *}\indent Thames River Bridge.\dotfill\ldots 4to, paper, &5\ 00\\ Burr's Course on the Stresses in Bridges and Roof Trusses, Arched Ribs,\\ \nopagebreak \indent\indent and Suspension Bridges.\dotfill\ldots 8vo, &3\ 50\\ Burr and Falk's Influence Lines for Bridge and Roof Computations.\dotfill\ldots 8vo, &3\ 00\\ \nopagebreak \indent Design and Construction of Metallic Bridges.\dotfill\ldots 8vo, &5\ 00\\ Du Bois's Mechanics of Engineering. Vol.~II.\dotfill\ldots Small 4to, &10\ 00\\ Foster's Treatise on Wooden Trestle Bridges.\dotfill\ldots 4to, &5\ 00\\ Fowler's Ordinary Foundations.\dotfill\ldots 8vo, &3\ 50\\ Greene's Roof Trusses.\dotfill\ldots 8vo, &1 25\\ \nopagebreak \indent Bridge Trusses.\dotfill\ldots 8vo, &2\ 50\\ \nopagebreak \indent Arches in Wood, Iron, and Stone.\dotfill\ldots 8vo, &2\ 50\\ Howe's Treatise on Arches.\dotfill\ldots 8vo, &4\ 00\\ \nopagebreak \indent Design of Simple Roof-trusses in Wood and Steel.\dotfill\ldots 8vo, &2\ 00\\ \nopagebreak \indent Symmetrical Masonry Arches.\dotfill\ldots 8vo, &2\ 50\\ Johnson, Bryan, and Turneaure's Theory and Practice in the Designing\\ \nopagebreak \indent\indent of Modern Framed Structures.\dotfill\ldots Small 4to, &10\ 00\\ Merriman and Jacoby's Text-book on Roofs and Bridges:\\ \indent Part I\@. Stresses in Simple Trusses.\dotfill\ldots 8vo, &2\ 50\\ \indent Part II\@. Graphic Statics.\dotfill\ldots 8vo, &2\ 50\\ \indent Part III\@. Bridge Design.\dotfill\ldots 8vo, &2\ 50\\ \indent Part IV\@. Higher Structures.\dotfill\ldots 8vo, &2\ 50\\ Morison's Memphis Bridge.\dotfill\ldots 4to, &10\ 00\\ Waddell's De Pontibus, a Pocket-book for Bridge\\ \nopagebreak \indent\indent Engineers.\dotfill\ldots 16mo, morocco, &2\ 00\\ \nopagebreak \indent * Specifications for Steel Bridges.\dotfill\ldots 12mo, &\ 50\\ Wright's Designing of Draw-spans. Two parts in one volume.\dotfill\ldots 8vo, &3\ 50\\[2em] \multicolumn{2}{c}{HYDRAULICS.}\\[1em] \nopagebreak Barnes's Ice Formation.\dotfill\ldots 8vo, &3\ 00\\ Bazin's Experiments upon the Contraction of the Liquid Vein Issuing\\ \nopagebreak \indent\indent from an Orifice. (Trautwine.)\dotfill\ldots 8vo, &2\ 00\\ Bovey's Treatise on Hydraulics.\dotfill\ldots 8vo, &5\ 00\\ Church's Mechanics of Engineering.\dotfill\ldots 8vo, &6\ 00\\ \nopagebreak \indent Diagrams of Mean Velocity of Water in Open Channels.\dotfill\ldots paper, &1\ 50\\ \nopagebreak \indent Hydraulic Motors.\dotfill\ldots 8vo, &2\ 00\\ Coffin's Graphical Solution of Hydraulic Problems.\dotfill\ldots 16mo, morocco, &2\ 50\\ Flather's Dynamometers, and the Measurement of Power.\dotfill\ldots 12mo, &3\ 00\\ %-----File: 248.png---Index 8-------- Folwell's Water-supply Engineering.\dotfill\ldots 8vo, &4\ 00\\ Frizell's Water-power.\dotfill\ldots 8vo, &5\ 00\\ Fuertes's Water and Public Health.\dotfill\ldots 12mo, &1\ 50\\ \nopagebreak \indent Water-filtration Works.\dotfill\ldots 12mo, &2\ 50\\ Ganguillet and Kutter's General Formula for the Uniform Flow of Water\\ \nopagebreak \indent\indent in Rivers and Other Channels. (Hering and Trautwine.)\dotfill\ldots 8vo, &4\ 00\\ Hazen's Filtration of Public Water-supply.\dotfill\ldots 8vo, &3\ 00\\ Hazlehurst's Towers and Tanks for Water-works.\dotfill\ldots 8vo, &2\ 50\\ Herschel's 115 Experiments on the Carrying Capacity of Large, \\ \nopagebreak \indent\indent Riveted, Metal Conduits.\dotfill\ldots 8vo, &2\ 00\\ Mason's Water-supply. (Considered Principally from\\ \nopagebreak \indent\indent a Sanitary Standpoint.)\dotfill\ldots 8vo, &4\ 00\\ Merriman's Treatise on Hydraulics.\dotfill\ldots 8vo, &5\ 00\\ * Michie's Elements of Analytical Mechanics.\dotfill\ldots 8vo, &4\ 00\\ Schuyler's Reservoirs for Irrigation, Water-power, and Domestic\\ \nopagebreak \indent\indent Water-supply.\dotfill\ldots Large 8vo, &5\ 00\\ * Thomas and Watt's Improvement of Rivers.\dotfill\ldots 4to, &6\ 00\\ Turneaure and Russell's Public Water-supplies.\dotfill\ldots 8vo, &5\ 00\\ Wegmann's Design and Construction of Dams.\dotfill\ldots 4to, &5\ 00\\ \nopagebreak \indent Water-supply of the City of New York from 1658 to 1895.\dotfill\ldots 4to, &10\ 00\\ Whipple's Value of Pure Water.\dotfill\ldots Large 12mo, & 1\ 00\\ Williams and Hazen's Hydraulic Tables.\dotfill\ldots 8vo, &1\ 50\\ Wilson's Irrigation Engineering.\dotfill\ldots Small 8vo, &4\ 00\\ Wolff's Windmill as a Prime Mover.\dotfill\ldots 8vo, &3\ 00\\ Wood's Turbines.\dotfill\ldots 8vo, &2\ 50\\ \nopagebreak \indent Elements of Analytical Mechanics.\dotfill\ldots 8vo, &3\ 00\\[2em] \multicolumn{2}{c}{MATERIALS OF ENGINEERING.}\\[1em] \nopagebreak Baker's Treatise on Masonry Construction.\dotfill\ldots 8vo, &5\ 00\\ \nopagebreak \indent Roads and Pavements.\dotfill\ldots 8vo, &5\ 00\\ Black's United States Public Works.\dotfill\ldots Oblong 4to, &5\ 00\\ * Bovey's Strength of Materials and Theory of Structures.\dotfill\ldots 8vo, &7\ 50\\ Burr's Elasticity and Resistance of the Materials of Engineering.\dotfill\ldots 8vo, &7\ 50\\ Byrne's Highway Construction.\dotfill\ldots 8vo, &5\ 00\\ \nopagebreak \indent Inspection of the Materials and Workmanship Employed\\ \nopagebreak \indent\indent in Construction.\dotfill\ldots 16mo, &3\ 00\\ Church's Mechanics of Engineering.\dotfill\ldots 8vo, &6\ 00\\ Du Bois's Mechanics of Engineering. Vol.~I.\dotfill\ldots Small 4to, &7\ 50\\ * Eckel's Cements, Limes, and Plasters.\dotfill\ldots 8vo, &6\ 00\\ Johnson's Materials of Construction.\dotfill\ldots Large 8vo, &6\ 00\\ Fowler's Ordinary Foundations.\dotfill\ldots 8vo, &3\ 50\\ Graves's Forest Mensuration.\dotfill\ldots 8vo, &4\ 00\\ * Greene's Structural Mechanics.\dotfill\ldots 8vo, &2\ 50\\ Keep's Cast Iron.\dotfill\ldots 8vo, &2\ 50\\ Lanza's Applied Mechanics.\dotfill\ldots 8vo, &7\ 50\\ Marten's Handbook on Testing Materials. (Henning.) 2 vols.\dotfill\ldots 8vo, &7\ 50\\ Maurer's Technical Mechanics.\dotfill\ldots 8vo, &4\ 00\\ Merrill's Stones for Building and Decoration.\dotfill\ldots 8vo, &5\ 00\\ Merriman's Mechanics of Materials.\dotfill\ldots 8vo, &5\ 00\\ \nopagebreak \makebox[0pt]{\hspace{.5ex} *}\indent Strength of Materials.\dotfill\ldots 12mo, &1\ 00\\ Metcalf's Steel. A Manual for Steel-users.\dotfill\ldots 12mo, &2\ 00\\ Patton's Practical Treatise on Foundations.\dotfill\ldots 8vo, &5\ 00\\ Richardson's Modern Asphalt Pavements.\dotfill\ldots 8vo, &3\ 00\\ Richey's Handbook for Superintendents of Construction.\dotfill\ldots 16mo, mor., &4\ 00\\ * Ries's Clays: Their Occurrence, Properties, and Uses.\dotfill\ldots 8vo, &5\ 00\\ Rockwell's Roads and Pavements in France.\dotfill\ldots 12mo, &11\ 25\\ %-----File: 249.png---Index 9-------- Sabin's Industrial and Artistic Technology of Paints and Varnish.\dotfill\ldots 8vo, &3\ 00\\ Smith's Materials of Machines.\dotfill\ldots 12mo, &1\ 00\\ Snow's Principal Species of Wood.\dotfill\ldots 8vo, &3\ 50\\ Spalding's Hydraulic Cement.\dotfill\ldots 12mo, &2\ 00\\ \nopagebreak \indent Text-book on Roads and Pavements.\dotfill\ldots 12mo, &2\ 00\\ Taylor and Thompson's Treatise on Concrete, Plain and Reinforced.\dotfill\ldots 8vo, &5\ 00\\ Thurston's Materials of Engineering. 3 Parts.\dotfill\ldots 8vo, &8\ 00\\ \nopagebreak \indent Part I.\quad Non-metallic Materials of Engineering and Metallurgy.\dotfill\ldots 8vo, &2\ 00\\ \nopagebreak \indent Part II.\quad Iron and Steel.\dotfill\ldots 8vo, &3\ 50\\ \nopagebreak \indent Part III.\quad A Treatise on Brasses, Bronzes, and Other Alloys and\\ \nopagebreak \indent\indent their Constituents.\dotfill\ldots 8vo, &2\ 50\\ Tillson's Street Pavements and Paving Materials.\dotfill\ldots 8vo, &4\ 00\\ Waddell's De Pontibus (A Pocket-book for\\ \nopagebreak \indent\indent Bridge Engineers.)\dotfill\ldots 16mo, mor., &2\ 00\\ \nopagebreak \indent Specifications for Steel Bridges.\dotfill\ldots 12mo, &1\ 25\\ Wood's (De V.) Treatise on the Resistance of Materials, and an Appendix\\ \nopagebreak \indent\indent on the Preservation of Timber.\dotfill\ldots 8vo, &2\ 00\\ Wood's (De V.) Elements of Analytical Mechanics.\dotfill\ldots 8vo, &3\ 00\\ Wood's (M.~P.) Rustless Coatings: Corrosion and Electrolysis of Iron\\ \nopagebreak \indent\indent and Steel.\dotfill\ldots 8vo, &4\ 00\\[2em] \multicolumn{2}{c}{RAILWAY ENGINEERING.}\\[1em] \nopagebreak Andrew's Handbook for Street Railway Engineers.\dotfill\ldots $3\times 5$ inches, morocco, &1\ 25\\ Berg's Buildings and Structures of American Railroads.\dotfill\ldots 4to, &5\ 00\\ Brook's Handbook of Street Railroad Location.\dotfill\ldots 16mo, morocco, &1\ 50\\ Butt's Civil Engineer's Field-book.\dotfill\ldots 16mo, morocco, &2\ 50\\ Crandall's Transition Curve.\dotfill\ldots 16mo, morocco, &1\ 50\\ \nopagebreak \indent Railway and Other Earthwork Tables.\dotfill\ldots 8vo, &1\ 50\\ Dawson's ``Engineering'' and Electric Traction\\ \nopagebreak \indent\indent Pocket-book.\dotfill\ldots 16mo, morocco, &5\ 00\\ Dredge's History ol the Pennsylvania Railroad: (1879)\dotfill\ldots Paper, &5\ 00\\ Fisher's Table of Cubic Yards.\dotfill\ldots Cardboard, &\ 25\\ Godwin's Railroad Engineers' Field-book\\ \nopagebreak \indent\indent and Explorers' Guide.\dotfill\ldots 16mo, mor., &2\ 50\\ Hudson's Tables for Calculating the Cubic Contents of Excavations\\ \nopagebreak \indent\indent and Embankments.\dotfill\ldots 8vo, &1\ 00\\ Molitor and Beard's Manual for Resident Engineers.\dotfill\ldots 16mo, &1\ 00\\ Nagle's Field Manual for Railroad Engineers.\dotfill\ldots 16mo, morocco, &3\ 00\\ Philbrick's Field Manual for Engineers.\dotfill\ldots 16mo, morocco, &3\ 00\\ Searles's Field Engineering.\dotfill\ldots 16mo, morocco, &3\ 00\\ \nopagebreak \indent Railroad Spiral.\dotfill\ldots 16mo, morocco, &1\ 50\\ Taylor's Prismoidal Formul\ae{} and Earthwork.\dotfill\ldots 8vo, &1\ 50\\ * Trautwine's Method of Calculating the Cube Contents of Excavations\\ \nopagebreak \indent\indent and Embankments by the Aid of Diagrams.\dotfill\ldots 8vo, &2\ 00\\ \indent The Field Practice of Laying Out Circular Curves\\ \nopagebreak \indent\indent for Railroads.\dotfill\ldots 12mo, morocco, &2\ 50\\ \indent Cross-section Sheet.\dotfill\ldots Paper, &\ 25\\ Webb's Railroad Construction.\dotfill\ldots 16mo, morocco, &5\ 00\\ \indent Economics of Railroad Construction.\dotfill\ldots Large 12mo, &2\ 50\\ Wellington's Economic Theory of the Location of Railways.\dotfill\ldots Small 8vo, &5\ 00\\[3em] \multicolumn{2}{c}{\large DRAWING.}\\[1em] \nopagebreak Barr's Kinematics of Machinery.\dotfill\ldots 8vo, &2\ 50\\ * Bartlett's Mechanical Drawing.\dotfill\ldots 8vo, &3\ 00\\ * \phantom{Bart}\makebox[0pt]{``}\phantom{lett's Mech}\makebox[0pt]{``}\phantom{anical Dra}\makebox[0pt]{``}\phantom{wing } Abridged Ed.\dotfill\ldots 8vo, &1\ 50\\ Coolidge's Manual of Drawing.\dotfill\ldots 8vo, paper, &1\ 00\\ %-----File: 250.png---Index 10------- Coolidge and Freeman's Elements of General Drafting\\ \nopagebreak \indent\indent for Mechanical Engineers.\dotfill\ldots Oblong 4to, &2\ 50\\ Durley's Kinematics of Machines.\dotfill\ldots 8vo, &4\ 00\\ Emch's Introduction to Projective Geometry and its Applications.\dotfill\ldots 8vo, &2\ 50\\ Hill's Text-book on Shades and Shadows, and Perspective.\dotfill\ldots 8vo, &2\ 00\\ Jamison's Elements of Mechanical Drawing.\dotfill\ldots 8vo, &2\ 50\\ \nopagebreak \indent Advanced Mechanical Drawing.\dotfill\ldots 8vo, &2\ 00\\ Jones's Machine Design:\\ \nopagebreak \indent Part I.\quad Kinematics of Machinery.\dotfill\ldots 8vo, &1\ 50\\ \nopagebreak \indent Part II.\quad Form, Strength, and Proportions of Parts.\dotfill\ldots 8vo, &3\ 00\\ MacCord's Elements of Descriptive Geometry.\dotfill\ldots 8vo, &3\ 00\\ \nopagebreak \indent Kinematics; or, Practical Mechanism.\dotfill\ldots 8vo, &5\ 00\\ \nopagebreak \indent Mechanical Drawing.\dotfill\ldots 4to, &4\ 00\\ \nopagebreak \indent Velocity Diagrams.\dotfill\ldots 8vo, &1\ 50\\ MacLeod's Descriptive Geometry.\dotfill\ldots Small 8vo, &1\ 50\\ * Mahan's Descriptive Geometry and Stone-cutting.\dotfill\ldots 8vo, &1\ 50\\ \nopagebreak \indent Industrial Drawing. (Thompson.)\dotfill 8vo, &3\ 50\\ Moyer's Descriptive Geometry.\dotfill\ldots 8vo, &2\ 00\\ Reed's Topographical Drawing and Sketching.\dotfill\ldots 4to, &5\ 00\\ Reid's Course in Mechanical Drawing.\dotfill\ldots 8vo, &2\ 00\\ \nopagebreak \indent Text-book of Mechanical Drawing and Elementary\\ \nopagebreak \indent\indent Machine Design.\dotfill\ldots 8vo, &3\ 00\\ Robinson's Principles of Mechanism.\dotfill\ldots 8vo, &3\ 00\\ Schwamb and Merrill's Elements of Mechanism.\dotfill\ldots 8vo, &3\ 00\\ Smith's (R.~S.) Manual of Topographical Drawing. (McMillan.)\dotfill 8vo, &2\ 50\\ Smith (A.~W.) and Marx's Machine Design.\dotfill\ldots 8vo, &3\ 00\\ * Titsworth's Elements of Mechanical Drawing.\dotfill\ldots Oblong 8vo, &1\ 25\\ Warren's Elements of Plane and Solid Free-hand Geometrical\\ \nopagebreak \indent\indent Drawing.\dotfill\ldots 12mo, &1\ 00\\ \indent Drafting Instruments and Operations.\dotfill\ldots 12mo, &1\ 25\\ \indent Manual of Elementary Projection Drawing.\dotfill\ldots 12mo, &1\ 50\\ \indent Manual of Elementary Problems in the Linear Perspective of Form\\ \nopagebreak \indent\indent and Shadow.\dotfill\ldots 12mo, &1\ 00\\ \indent Plane Problems in Elementary Geometry.\dotfill\ldots 12mo, &1\ 25\\ \indent Primary Geometry.\dotfill\ldots 12mo, &\ 75\\ \indent Elements of Descriptive Geometry, Shadows, and Perspective.\dotfill\ldots 8vo, &3\ 50\\ \indent General Problems of Shades and Shadows.\dotfill\ldots 8vo, &3\ 00\\ \indent Elements of Machine Construction and Drawing.\dotfill\ldots 8vo, &7\ 50\\ \indent Problems, Theorems, and Examples in Descriptive Geometry.\dotfill\ldots 8vo, &2\ 50\\ Weisbach's Kinematics and Power of Transmission. (Hermann\\ \nopagebreak \indent\indent and Klein.)\dotfill\ldots 8vo, &5\ 00\\ Whelpley's Practical Instruction in the Art of Letter Engraving.\dotfill\ldots 12mo, &2\ 00\\ Wilson's (H.~M.) Topographic Surveying.\dotfill\ldots 8vo, &3\ 50\\ Wilson's (V.~T.) Free-hand Perspective.\dotfill\ldots 8vo, &2\ 50\\ Wilson's (V.~T.) Free-hand Lettering.\dotfill\ldots 8vo, &1\ 00\\ Woolf's Elementary Course in Descriptive Geometry.\dotfill\ldots Large 8vo, &3\ 00\\[3em] \multicolumn{2}{c}{\large ELECTRICITY AND PHYSICS.}\\[1em] \nopagebreak * Abegg's Theory of Electrolytic Dissociation. (Von Ende.)\dotfill\ldots 12mo, & 1\ 25 \\ Anthony and Brackett's Text-book of Physics. (Magie.)\dotfill Small 8vo, &3\ 00\\ Anthony's Lecture-notes on the Theory of Electrical\\ \nopagebreak \indent\indent Measurements.\dotfill\ldots 12mo, &1\ 00\\ Benjamin's History of Electricity.\dotfill\ldots 8vo, &3\ 00\\ \nopagebreak \indent Voltaic Cell.\dotfill\ldots 8vo, &3\ 00\\ Classen's Quantitative Chemical Analysis\\ \nopagebreak \indent\indent by Electrolysis. (Boltwood.)\dotfill 8vo, &3\ 00\\ * Collins's Manual of Wireless Telegraphy.\dotfill\ldots 12mo, &1\ 50\\ \nopagebreak \hfill Morocco, &2\ 00\\ Crehore and Squier's Polarizing Photo-chronograph.\dotfill\ldots 8vo, &3\ 00\\ * Danneel's Electrochemistry. (Merriam.)\dotfill\ldots 12mo, & 1\ 25 \\ Dawson's ``Engineering'' and Electric Traction\\ \nopagebreak \indent\indent Pocket-book.\dotfill\ldots 16mo, morocco, &5\ 00\\ %-----File: 251.png---Index 11------- Dolezalek's Theory of the Lead Accumulator (Storage Battery).\\ \nopagebreak \indent\indent (Von Ende.)\dotfill\ldots 12mo, &2 50\\ Duhem's Thermodynamics and Chemistry. (Burgess.)\dotfill\ldots 8vo, &4\ 00\\ Flather's Dynamometers, and the Measurement of Power.\dotfill\ldots 12mo, &3\ 00\\ Gilbert's De Magnete. (Mottelay.)\dotfill\ldots 8vo, &2\ 50\\ Hanchett's Alternating Currents Explained.\dotfill\ldots 12mo, &1\ 00\\ Hering's Ready Reference Tables (Conversion Factors)\dotfill\ldots 16mo, morocco, &2\ 50\\ Holman's Precision of Measurements.\dotfill\ldots 8vo, &2\ 00\\ \nopagebreak \indent Telescopic Mirror-scale Method, Adjustments, and Tests.\dotfill\ldots Large 8vo, & 75\\ Kinzbrunner's Testing of Continuous-current Machines.\dotfill\ldots 8vo, &2\ 00\\ Landauer's Spectrum Analysis. (Tingle.)\dotfill\ldots 8vo, &3\ 00\\ Le Chatelier's High-temperature Measurements.\\ \nopagebreak \indent\indent (Boudouard---Burgess.)\dotfill\ldots 12mo, &3\ 00\\ L\"ob's Electrochemistry of Organic Compounds. (Lorenz.)\dotfill\ldots 8vo, &3 00\\ * Lyons's Treatise on Electromagnetic Phenomena. Vols.~I.\\ \nopagebreak \indent\indent and II.\dotfill\ldots 8vo, each, &6\ 00\\ * Michie's Elements of Wave Motion Relating to Sound and Light.\dotfill\ldots 8vo, &4\ 00\\ Niaudet's Elementary Treatise on Electric Batteries. (Fishback.)\dotfill\ldots 12mo, &\ 50\\ * Parshall and Hobart's Electric Machine Design.\dotfill\ldots 4to, half morocco, &12\ 50\\ Reagan's Locomotives: Simple, Compound, and Electric.\\ \nopagebreak \indent\indent New Edition.\dotfill\ldots Large 12mo, &2 50\\ * Rosenberg's Electrical Engineering. (Haldane Gee---Kinzbrunner.)\dotfill\ldots 8vo, &1\ 50\\ Ryan, Norris, and Hoxie's Electrical Machinery. Vol.~I.\dotfill\ldots 8vo, &2\ 50\\ Thurston's Stationary Steam-engines.\dotfill\ldots 8vo, &2\ 50\\ * Tillman's Elementary Lessons in Heat.\dotfill\ldots 8vo, &1\ 50\\ Tory and Pitcher's Manual of Laboratory Physics.\dotfill\ldots Small 8vo, &2\ 00\\ Ulke's Modern Electrolytic Copper Refining.\dotfill\ldots 8vo, &3\ 00\\[3em] \multicolumn{2}{c}{\large LAW.}\\[1em] \nopagebreak * Davis's Elements of Law.\dotfill\ldots 8vo, &2\ 50\\ \makebox[0pt]{\hspace{.5ex} *}\indent Treatise on the Military Law of United States.\dotfill\ldots 8vo, &7 00\\ \nopagebreak *\hfill Sheep, &7\ 50\\ * Dudley's Military Law and the Procedure\\ \nopagebreak \indent\indent of Courts-martial.\dotfill\ldots Large 12mo, & 2\ 50\\ Manual for Courts-martial.\dotfill\ldots 16mo, morocco, &1\ 50\\ Wait's Engineering and Architectural Jurisprudence.\dotfill\ldots 8vo, &6 00\\ \nopagebreak \hfill Sheep, &6\ 50\\ \indent Law of Operations Preliminary to Construction in Engineering\\ \nopagebreak \indent\indent and Architecture.\dotfill\ldots 8vo, &5\ 00\\ \nopagebreak \hfill Sheep, &5\ 50\\ \indent Law of Contracts.\dotfill\ldots 8vo, &3\ 00\\ Winthrop's Abridgment of Military Law.\dotfill\ldots 12mo, &2\ 50\\[3em] \multicolumn{2}{c}{\large MANUFACTURES.}\\[1em] \nopagebreak Bernadou's Smokeless Powder---Nitro-cellulose and Theory of\\ \nopagebreak \indent\indent the Cellulose Molecule.\dotfill\ldots 12mo, &2\ 50\\ Bolland's Iron Founder.\dotfill\ldots 12mo, &2\ 50\\ \nopagebreak \indent \correction{``}{}The Iron Founder,'' Supplement.\dotfill\ldots 12mo, &2\ 50\\ \indent Encyclopedia of Founding and Dictionary of Foundry Terms Used\\ \nopagebreak \indent\indent in the Practice of Moulding.\dotfill\ldots 12mo, &3\ 00\\ * Claassen's Beet-sugar Manufacture. (Hall and Rolfe.)\dotfill\ldots 8vo, &3\ 00\\ * Eckel's Cements, Limes, and Plasters.\dotfill\ldots 8vo, &6\ 00\\ Eissler's Modern High Explosives.\dotfill\ldots 8vo, &4\ 00\\ Effront's Enzymes and their Applications. (Prescott.)\dotfill\ldots 8vo, &3\ 00\\ Fitzgerald's Boston Machinist.\dotfill\ldots 12mo, &1\ 00\\ Ford's Boiler Making for Boiler Makers.\dotfill\ldots 18mo, &1\ 00\\ Hopkin's Oil-chemists' Handbook.\dotfill\ldots 8vo, &3\ 00\\ Keep's Cast Iron.\dotfill\ldots 8vo, &2\ 50\\ %-----File: 252.png---Index 12------- Leach's The Inspection and Analysis of Food with Special Reference\\ \nopagebreak \indent\indent to State Control.\dotfill\ldots Large 8vo, &7\ 50\\ * McKay and Larsen's Principles and Practice of Butter-making.\dotfill\ldots 8vo, &1\ 50\\ Matthews's The Textile Fibres.\dotfill\ldots 8vo, &3\ 50\\ Metcalf's Steel. A Manual for Steel-users.\dotfill\ldots 12mo, &2\ 00\\ Metcalfe's Cost of Manufactures---And the Administration\\ \nopagebreak \indent\indent of Workshops.\dotfill\ldots 8vo, &5\ 00\\ Meyer's Modern Locomotive Construction.\dotfill\ldots 4to, &10\ 00\\ Morse's Calculations used in Cane-sugar Factories.\dotfill\ldots 16mo, morocco, &1\ 50\\ * Reisig's Guide to Piece-dyeing.\dotfill\ldots 8vo, &25\ 00\\ Rice's Concrete-block Manufacture.\dotfill\ldots 8vo, &2\ 00\\ Sabin's Industrial and Artistic Technology of Paints and Varnish.\dotfill\ldots 8vo, &3\ 00\\ Smith's Press-working of Metals.\dotfill\ldots 8vo, &3\ 00\\ Spalding's Hydraulic Cement.\dotfill\ldots 12mo, &2\ 00\\ Spencer's Handbook for Chemists of Beet-sugar Houses.\dotfill\ldots 16mo, morocco, &3\ 00\\ \nopagebreak \indent Handbook for Cane Sugar Manufacturers.\dotfill\ldots 16mo, morocco, &3\ 00\\ Taylor and Thompson's Treatise on Concrete, Plain and Reinforced.\dotfill\ldots 8vo, &5\ 00\\ Thurston's Manual of Steam-boilers, their Designs, Construction\\ \nopagebreak \indent\indent and Operation.\dotfill\ldots 8vo, &5\ 00\\ * Walke's Lectures on Explosives.\dotfill\ldots 8vo, &4\ 00\\ Ware's Beet-sugar Manufacture and Refining.\dotfill\ldots Small 8vo, &4\ 00\\ Weaver's Military Explosives.\dotfill\ldots 8vo, &3\ 00\\ West's American Foundry Practice.\dotfill\ldots 12mo, &2\ 50\\ \nopagebreak \indent Moulder's Text-book.\dotfill\ldots 12mo, &2\ 50\\ Wolff's Windmill as a Prime Mover.\dotfill\ldots 8vo, &3\ 00\\ Wood's Rustless Coatings: Corrosion and Electrolysis of Iron\\ \nopagebreak \indent\indent and Steel.\dotfill\ldots 8vo, &4\ 00\\[3em] \multicolumn{2}{c}{\large MATHEMATICS.}\\[1em] \nopagebreak Baker's Elliptic Functions.\dotfill\ldots 8vo, &1\ 50\\ * Bass's Elements of Differential Calculus.\dotfill\ldots 12mo, &4\ 00\\ Briggs's Elements of Plane Analytic Geometry.\dotfill\ldots 12mo, &1\ 00\\ Compton's Manual of Logarithmic Computations.\dotfill\ldots 12mo, &1\ 50\\ Davis's Introduction to the Logic of Algebra.\dotfill\ldots 8vo, &1\ 50\\ * Dickson's College Algebra.\dotfill\ldots Large 12mo, &1\ 50\\ \makebox[0pt]{\hspace{.5ex} *}\indent Introduction to the Theory of Algebraic Equations.\dotfill\ldots Large 12mo, &1\ 25\\ Emch's Introduction to Projective Geometry and its Applications.\dotfill\ldots 8vo, &2\ 50\\ Halsted's Elements of Geometry.\dotfill\ldots 8vo, &1\ 75\\ \nopagebreak \indent Elementary Synthetic Geometry.\dotfill\ldots 8vo, &1\ 50\\ \nopagebreak \indent Rational Geometry.\dotfill\ldots 12mo, &1 75\\ * Johnson's (J.~B.) Three-place Logarithmic Tables:\\ \nopagebreak \indent\indent Vest-pocket size.\dotfill\ldots paper, &\ 15\\ \nopagebreak \hfill 100 copies for &5\ 00\\ \nopagebreak *\hfill Mounted on heavy cardboard, $8\times10$ inches, &\ 25\\ \nopagebreak \hfill 10 copies for &2\ 00\\ Johnson's (W.~W.) Elementary Treatise on Differential\\ \nopagebreak \indent\indent Calculus.\dotfill\ldots Small 8vo, &3\ 00\\ \nopagebreak \indent Elementary Treatise on the Integral Calculus.\dotfill\ldots Small 8vo, &1\ 50\\ Johnson's (W.~W.) Curve Tracing in Cartesian Co-ordinates.\dotfill\ldots 12mo, &1\ 00\\ Johnson's (W.~W.) Treatise on Ordinary and Partial Differential\\ \nopagebreak \indent\indent Equations.\dotfill\ldots Small 8vo, &3\ 50\\ Johnson's (W.~W.) Theory of Errors and the Method of\\ \nopagebreak \indent\indent Least Squares.\dotfill\ldots 12mo, &1\ 50\\ * Johnson's (W.~W.) Theoretical Mechanics,.\dotfill\ldots 12mo, &3\ 00\\ Laplace's Philosophical Essay on Probabilities. (Truscott\\ \nopagebreak \indent\indent and Emory.)\dotfill\ldots 12mo, &2\ 00\\ * Ludlow and Bass. Elements of Trigonometry and Logarithmic\\ \nopagebreak \indent\indent and Other Tables.\dotfill\ldots 8vo, &3\ 00\\ \nopagebreak \indent Trigonometry and Tables published separately.\dotfill\ldots Each, &2\ 00\\ * Ludlow's Logarithmic and Trigonometric Tables.\dotfill\ldots 8vo, &1\ 00\\ Manning's Irrational Numbers and their Representation by Sequences\\ \nopagebreak \indent\indent and Series.\dotfill\ldots 12mo &1\ 25\\ %-----File: 253.png---Index 13------- Mathematical Monographs. Edited by Mansfield Merriman and Robert \\ \indent\indent S.~Woodward.\dotfill\ldots Octavo, each &1\ 00\\ \indent \begin{minipage}{.8\textwidth} No.~1. History of Modern Mathematics, by David Eugene Smith.\quad No.~2. Synthetic Projective Geometry, by George Bruce Halsted.\quad No.~3. Determinants, by Laenas Gifford Weld.\quad No.~4. Hyperbolic Functions, by James McMahon.\quad No.~5. Harmonic Functions, by William E. Byerly.\quad No.~6. Grassmann's Space Analysis, by Edward W. Hyde.\quad No.~7. Probability and Theory of Errors, by Robert S. Woodward.\quad No.~8. Vector Analysis and Quaternions, by Alexander Macfarlane.\quad No.~9. Differential Equations, by William Woolsey Johnson.\quad No.~10. The Solution of Equations, by Mansfield Merriman.\quad No.~11. Functions of a Complex Variable, by Thomas S. Fiske. \end{minipage}\\ Maurer's Technical Mechanics.\dotfill\ldots 8vo, &4\ 00\\ Merriman's Method of Least Squares.\dotfill\ldots 8vo, &2\ 00\\ Rice and Johnson's Elementary Treatise on the Differential\\ \nopagebreak \indent\indent Calculus.\dotfill\ldots Sm. 8vo, &3\ 00\\ \indent Differential and Integral Calculus. 2 vols.\ in one.\dotfill\ldots Small 8vo, &2\ 50\\ * Veblen and Lennes's Introduction to the Real Infinitesimal Analysis\\ \indent\indent of One Variable.\dotfill\ldots 8vo, & 2\ 00\\ Wood's Elements of Co-ordinate Geometry.\dotfill\ldots 8vo, &2\ 00\\ \indent Trigonometry: Analytical, Plane, and Spherical.\dotfill\ldots 12mo, &1\ 00\\[3em] \multicolumn{2}{c}{\large MECHANICAL ENGINEERING.}\\[1em] \nopagebreak \multicolumn{2}{c}{MATERIALS OF ENGINEERING, STEAM-ENGINES AND BOILERS.}\\[1em] \nopagebreak Bacon's Forge Practice.\dotfill\ldots 12mo, &1\ 50\\ Baldwin's Steam Heating for Buildings.\dotfill\ldots 12mo, &2\ 50\\ Barr's Kinematics of Machinery.\dotfill\ldots 8vo, &2\ 50\\ * Bartlett's Mechanical Drawing.\dotfill\ldots 8vo, &3\ 00\\ * \phantom{Bart}\makebox[0pt]{``}\phantom{lett's Mech}\makebox[0pt]{``}\phantom{anical Dra}\makebox[0pt]{``}\phantom{wing } Abridged Ed.\dotfill\ldots 8vo, &1\ 50\\ Benjamin's Wrinkles and Recipes.\dotfill\ldots 12mo, &2\ 00\\ Carpenter's Experimental Engineering.\dotfill\ldots 8vo, &6\ 00\\ \indent Heating and Ventilating Buildings.\dotfill\ldots 8vo, &4\ 00\\ Clerk's Gas and Oil Engine.\dotfill\ldots Small 8vo, &4\ 00\\ Coolidge's Manual of Drawing.\dotfill\ldots 8vo, paper, &1\ 00\\ Coolidge and Freeman's Elements of General Drafting\\ \indent\indent for Mechanical Engineers.\dotfill\ldots Oblong 4to, &2\ 50\\ Cromwell's Treatise on Toothed Gearing.\dotfill\ldots 12mo, &1\ 50\\ \indent Treatise on Belts and Pulleys.\dotfill\ldots 12mo, &1\ 50\\ Durley's Kinematics of Machines.\dotfill\ldots 8vo, &4\ 00\\ Flather's Dynamometers and the Measurement of Power.\dotfill\ldots 12mo, &3\ 00\\ \indent Rope Driving.\dotfill\ldots 12mo, &2\ 00\\ Gill's Gas and Fuel Analysis for Engineers.\dotfill\ldots 12mo, &1\ 25\\ Hall's Car Lubrication.\dotfill\ldots 12mo, &1\ 00\\ Hering's Ready Reference Tables (Conversion Factors)\dotfill\ldots 16mo, morocco, &2\ 50\\ Hutton's The Gas Engine.\dotfill\ldots 8vo, &5\ 00\\ Jamison's Mechanical Drawing.\dotfill\ldots 8vo, &2 50\\ Jones's Machine Design:\\ \indent Part I.\quad Kinematics of Machinery.\dotfill\ldots 8vo, &1\ 50\\ \indent Part II.\quad Form, Strength, and Proportions of Parts.\dotfill\ldots 8vo, &3\ 00\\ Kent's Mechanical Engineers' Pocket-book.\dotfill\ldots 16mo, morocco, &5\ 00\\ Kerr's Power and Power Transmission.\dotfill\ldots 8vo, &2\ 00\\ Leonard's Machine Shop, Tools, and Methods.\dotfill\ldots 8vo, &4\ 00\\ * Lorenz's Modern Refrigerating Machinery. (Pope, Haven,\\ \indent\indent and Dean.)\dotfill\ldots 8vo, &4\ 00\\ MacCord's Kinematics; or Practical Mechanism.\dotfill\ldots 8vo, &5\ 00\\ \indent Mechanical Drawing.\dotfill\ldots 4to, &4\ 00\\ \indent Velocity Diagrams.\dotfill\ldots 8vo, &1\ 50\\ %-----File: 254.png---Index 14------- MacFarland's Standard Reduction Factors for Gases.\dotfill\ldots 8vo, &1\ 50\\ Mahan's Industrial Drawing. (Thompson.)\dotfill\ldots 8vo, &3\ 50\\ Poole's Calorific Power of Fuels.\dotfill\ldots 8vo, &3\ 00\\ Reid's Course in Mechanical Drawing.\dotfill\ldots 8vo, &2\ 00\\ \indent Text-book of Mechanical Drawing and Elementary\\ \nopagebreak \indent\indent Machine Design.\dotfill\ldots 8vo, &3\ 00\\ Richard's Compressed Air.\dotfill\ldots 12mo, &1 50\\ Robinson's Principles of Mechanism.\dotfill\ldots 8vo, &3\ 00\\ Schwamb and Merrill's Elements of Mechanism.\dotfill\ldots 8vo, &3\ 00\\ Smith's (O.) Press-working of Metals.\dotfill\ldots 8vo, &3\ 00\\ Smith (A.~W.) and Marx's Machine Design.\dotfill\ldots 8vo, &3\ 00\\ Thurston's Treatise on Friction and Lost Work in Machinery\\ \nopagebreak \indent\indent and Mill Work.\dotfill\ldots 8vo, &3\ 00\\ \indent Animal as a Machine and Prime Motor, and the Laws\\ \nopagebreak \indent\indent of Energetics.\dotfill\ldots 12mo, &1\ 00\\ Tillson's Complete Automobile Instructor.\dotfill\ldots 16mo, & 1\ 50\\ \nopagebreak \hfill Morocco, & 2\ 00\\ Warren's Elements of Machine Construction and Drawing.\dotfill\ldots 8vo, &7\ 50\\ Weisbach's Kinematics and the Power of Transmission. \\ \nopagebreak \indent\indent (Herrmann---Klein.)\dotfill\ldots 8vo, &5\ 00\\ \indent Machinery of Transmission and Governors. (Herrmann---Klein.)\dotfill\ldots 8vo, &5\ 00\\ Wolff's Windmill as a Prime Mover.\dotfill\ldots 8vo, &3\ 00\\ Wood's Turbines.\dotfill\ldots 8vo, &2\ 50\\[2em] \multicolumn{2}{c}{MATERIALS OF ENGINEERING.}\\[1em] \nopagebreak * Bovey's Strength of Materials and Theory of Structures.\dotfill\ldots 8vo, &7\ 50\\ Burr's Elasticity and Resistance of the Materials of Engineering.\\ \nopagebreak \indent\indent 6th Edition. Reset.\dotfill\ldots 8vo, &7\ 50\\ Church's Mechanics of Engineering.\dotfill\ldots 8vo, &6\ 00\\ * Greene's Structural Mechanics.\dotfill\ldots 8vo, &2\ 50\\ Johnson's Materials of Construction.\dotfill\ldots 8vo, &6\ 00\\ Keep's Cast Iron.\dotfill\ldots 8vo, &2\ 50\\ Lanza's Applied Mechanics.\dotfill\ldots 8vo, &7\ 50\\ Martens's Handbook on Testing Materials. (Henning.)\dotfill\ldots 8vo, &7\ 50\\ Maurer's Technical Mechanics.\dotfill\ldots 8vo, &4\ 00\\ Merriman's Mechanics of Materials.\dotfill\ldots 8vo, &5\ 00\\ \makebox[0pt]{\hspace{.5ex} *}\indent Strength of Materials.\dotfill\ldots 12mo, &1\ 00\\ Metcalf's Steel. A Manual for Steel-users.\dotfill\ldots 12mo, &2\ 00\\ Sabin's Industrial and Artistic Technology of Paints and Varnish.\dotfill\ldots 8vo, &3\ 00\\ Smith's Materials of Machines.\dotfill\ldots 12mo, &1\ 00\\ Thurston's Materials of Engineering.\dotfill\ldots 3 vols., 8vo, &8\ 00\\ \indent Part II.\quad Iron and Steel.\dotfill\ldots 8vo, &3\ 50\\ \indent Part III.\quad A Treatise on Brasses, Bronzes, and Other Alloys and\\ \nopagebreak \indent\indent their Constituents.\dotfill\ldots 8vo, &2\ 50\\ Wood's (De V.) Treatise on the Resistance of Materials and an Appendix\\ \nopagebreak \indent\indent on the Preservation of Timber.\dotfill\ldots 8vo, &2\ 00\\ \indent Elements of Analytical Mechanics.\dotfill\ldots 8vo, &3\ 00\\ Wood's (M.~P.) Rustless Coatings: Corrosion and Electrolysis of Iron\\ \indent\indent and Steel.\dotfill\ldots 8vo, &4\ 00\\[2em] \multicolumn{2}{c}{STEAM-ENGINES AND BOILERS.}\\[1em] \nopagebreak Berry's Temperature-entropy Diagram.\dotfill\ldots 12mo, &1\ 25\\ Carnot's Reflections on the Motive Power of Heat (Thurston.)\dotfill\ldots 12mo, &1\ 50\\ Dawson's ``Engineering'' and Electric Traction Pocket-book.\dotfill\ldots 16mo mor., &5\ 00\\ Ford's Boiler Making for Boiler Makers.\dotfill\ldots 18mo, &1\ 00\\ Goss's Locomotive Sparks.\dotfill\ldots 8vo, &2\ 00\\ \indent Locomotive Performance.\dotfill\ldots 8vo, & 5\ 00\\ Hemenway's Indicator Practice and Steam-engine Economy.\dotfill\ldots 12mo, &2\ 00\\ %-----File: 255.png---Index 15------- Hutton's Mechanical Engineering of Power Plants.\dotfill\ldots 8vo, &5\ 00\\ \indent Heat and Heat-engines.\dotfill\ldots 8vo, &5\ 00\\ Kent's Steam boiler Economy.\dotfill\ldots 8vo, &4\ 00\\ Kneass's Practice and Theory of the Injector.\dotfill\ldots 8vo, &1\ 50\\ MacCord's Slide-valves.\dotfill\ldots 8vo, &2\ 00\\ Meyer's Modern Locomotive Construction.\dotfill\ldots 4to, &10\ 00\\ Peabody's Manual of the Steam-engine Indicator.\dotfill\ldots 12mo, &1\ 50\\ \indent Tables of the Properties of Saturated Steam and Other Vapors.\dotfill\ldots 8vo, &1\ 00\\ \indent Thermodynamics of the Steam-engine and Other Heat-engines.\dotfill\ldots 8vo, &5\ 00\\ \indent Valve-gears for Steam-engines.\dotfill\ldots 8vo, &2\ 50\\ Peabody and Miller's Steam-boilers.\dotfill\ldots 8vo, &4\ 00\\ Pray's Twenty Years with the Indicator.\dotfill\ldots Large 8vo, &2\ 50\\ Pupin's Thermodynamics of Reversible Cycles in Gases and\\ \nopagebreak \indent\indent Saturated Vapors. (Osterberg.)\dotfill\ldots 12mo, &1\ 25\\ Reagan's Locomotives: Simple, Compound,\\ \nopagebreak \indent\indent and Electric.\dotfill\ldots Large 12mo, &2\ 50\\ Rontgen's Principles of Thermodynamics. (Du Bois.)\dotfill\ldots 8vo, &5\ 00\\ Sinclair's Locomotive Engine Running and Management.\dotfill\ldots 12mo, &2\ 00\\ Smart's Handbook of Engineering Laboratory Practice.\dotfill\ldots 12mo, &2\ 50\\ Snow's Steam-boiler Practice.\dotfill\ldots 8vo, &3\ 00\\ Spangler's Valve-gears.\dotfill\ldots 8vo, &2\ 50\\ \indent Notes on Thermodynamics.\dotfill\ldots 12mo, &1\ 00\\ Spangler, Greene, and Marshall's Elements of Steam-engineering.\dotfill\ldots 8vo, &3\ 00\\ Thomas's Steam-turbines.\dotfill\ldots 8vo, &3\ 50\\ Thurston's Handy Tables.\dotfill\ldots 8vo, &1\ 50\\ \indent Manual of the Steam-engine.\dotfill\ldots 2 vols., 8vo, &10\ 00\\ \indent Part I.\quad History, Structure, and Theory.\dotfill\ldots 8vo, &6\ 00\\ \indent Part II.\quad Design, Construction, and Operation.\dotfill\ldots 8vo, &6\ 00\\ \indent Handbook of Engine and Boiler Trials, and the Use of the Indicator\\ \nopagebreak \indent\indent and the Prony Brake.\dotfill\ldots 8vo, &5\ 00\\ \indent Stationary Steam-engines.\dotfill\ldots 8vo, &2\ 50\\ \indent Steam-boiler Explosions in Theory and in Practice.\dotfill\ldots 12mo, &1\ 50\\ \indent Manual of Steam-boilers, their Designs, Construction,\\ \nopagebreak \indent\indent and Operation.\dotfill\ldots 8vo, &5\ 00\\ \correction{Wehrenfennigs's}{Wehrenfenning's} Analysis and Softening of Boiler\\ \nopagebreak \indent\indent Feed-water (Patterson)\dotfill\ldots 8vo, &4\ 00\\ Weisbach's Heat, Steam, and Steam-engines. (Du Bois.)\dotfill\ldots 8vo, &5\ 00\\ Whitham's Steam-engine Design.\dotfill\ldots 8vo, &5\ 00\\ Wood's Thermodynamics, Heat Motors,\\ \nopagebreak \indent\indent and Refrigerating Machines.\dotfill\ldots 8vo, &4\ 00\\[3em] \multicolumn{2}{c}{\large MECHANICS AND MACHINERY.}\\[1em] \nopagebreak Barr's Kinematics of Machinery.\dotfill\ldots 8vo, &2\ 50\\ * Bovey's Strength of Materials and Theory of Structures.\dotfill\ldots 8vo, &7\ 50\\ Chase's The Art of Pattern-making.\dotfill\ldots 12mo, &2\ 50\\ Church's Mechanics of Engineering.\dotfill\ldots 8vo, &6\ 00\\ \indent Notes and Examples in Mechanics.\dotfill\ldots 8vo, &2\ 00\\ Compton's First Lessons in Metal-working.\dotfill\ldots 12mo, &1\ 50\\ Compton and De Groodt's The Speed Lathe.\dotfill\ldots 12mo, &1\ 50\\ Cromwell's Treatise on Toothed Gearing.\dotfill\ldots 12mo, &1\ 50\\ \indent Treatise on Belts and Pulleys.\dotfill\ldots 12mo, &1\ 50\\ Dana's Text-book of Elementary Mechanics for Colleges\\ \nopagebreak \indent\indent and Schools.\dotfill\ldots 12mo, & 1\ 50\\ Dingey's Machinery Pattern Making.\dotfill\ldots 12mo, &2\ 00\\ Dredge's Record of the Transportation Exhibits Building of\\ \nopagebreak \indent\indent the World's Columbian Exposition of 1893.\dotfill\ldots 4to half morocco, &5\ 00\\ Du Bois's Elementary Principles of Mechanics:\\ \indent Vol.~\phantom{I}I.\quad Kinematics.\dotfill\ldots 8vo. &3\ 50\\ \indent Vol.~II.\quad Statics.\dotfill\ldots 8vo, &4\ 00\\ \indent Mechanics of Engineering. Vol.~I.\dotfill\ldots Small 4to, &7\ 50\\ \indent \phantom{Mechanics of Engineering. }Vol.~II.\dotfill\ldots Small 4to, &10\ 00\\ Durley's Kinematics of Machines.\dotfill\ldots 8vo. &4\ 00\\ %-----File: 256.png---Index 16------- Fitzgerald's Boston Machinist.\dotfill\ldots 16mo, & 1\ 00\\ Flather's Dynamometers, and the Measurement of Power.\dotfill\ldots 12mo, & 3\ 00\\ \indent Rope Driving.\dotfill\ldots 12mo, & 2\ 00\\ Goss's Locomotive Sparks.\dotfill\ldots 8vo, & 2\ 00\\ Locomotive Performance.\dotfill\ldots 8vo, & 5\ 00\\ \correction{}{\indent}* Greene's Structural Mechanics.\dotfill\ldots 8vo, & 2\ 50\\ Hall's Car Lubrication.\dotfill\ldots 12mo, & 1\ 00 \\ Holly's Art of Saw Filing.\dotfill\ldots 18mo, & \ 75\\ James's Kinematics of a Point and the Rational Mechanics\\ \nopagebreak \indent\indent of a Particle.\dotfill\ldots Small 8vo, & 2\ 00\\ * Johnson's (W.~W.) Theoretical Mechanics.\dotfill\ldots 12mo, & 3\ 00\\ Johnson's (L.~J.) Statics by Graphic and Algebraic Methods.\dotfill\ldots 8vo, & 2\ 00\\ Jones's Machine Design:\\ \indent Part~\phantom{I}I.\quad Kinematics of Machinery.\dotfill\ldots 8vo, & 1\ 50\\ \indent Part~II.\quad Form, Strength, and Proportions of Parts.\dotfill\ldots 8vo, & 3\ 00\\ Kerr's Power and Power Transmission.\dotfill\ldots 8vo, & 2\ 00\\ Lanza's Applied Mechanics.\dotfill\ldots 8vo, & 7\ 50\\ Leonard's Machine Shop, Tools, and Methods.\dotfill\ldots 8vo, & 4\ 00\\ * Lorenz's Modern Refrigerating Machinery. (Pope, Haven,\\ \nopagebreak \indent\indent and Dean.)\dotfill\ldots 8vo, & 4\ 00\\ MacCord's Kinematics; or, Practical Mechanism.\dotfill\ldots 8vo, & 5\ 00\\ \indent Velocity Diagrams.\dotfill\ldots 8vo, & 1\ 50\\ * Martin's Text Book on Mechanics, Vol.~I, Statics.\dotfill\ldots 12mo, & 1\ 25\\ Maurer's Technical Mechanics.\dotfill\ldots 8vo, & 4\ 00\\ Merriman's Mechanics of Materials.\dotfill\ldots 8vo, & 5\ 00\\ \makebox[0pt]{\hspace{.5ex} *}\indent Elements of Mechanics.\dotfill\ldots 12mo, & 1\ 00\\ * Michie's Elements of Analytical Mechanics.\dotfill\ldots 8vo, & 4\ 00\\ * Parshall and Hobart's Electric Machine Design.\dotfill\ldots 4to, half morocco, & 12\ 50\\ Reagan's Locomotives: Simple, Compound,\\ \nopagebreak \indent\indent and Electric. New Edition.\dotfill\ldots Large 12mo, & 3\ 00\\ Reid's Course in Mechanical Drawing.\dotfill\ldots 8vo, & 2\ 00\\ \indent Text-book of Mechanical Drawing and Elementary\\ \nopagebreak \indent\indent Machine Design.\dotfill\ldots 8vo, & 3\ 00\\ Richards's Compressed Air.\dotfill\ldots 12mo, & 1\ 50\\ Robinson's Principles of Mechanism.\dotfill\ldots 8vo, & 3\ 00\\ Ryan, Norris, and Hoxie's Electrical Machinery. Vol.~I.\dotfill\ldots 8vo, & 2\ 50\\ Sanborn's Mechanics: Problems.\dotfill\ldots Large 12mo, & 1\ 50\\ Schwamb and Merrill's Elements of Mechanism.\dotfill\ldots 8vo, & 3\ 00\\ Sinclair's Locomotive-engine Running and Management.\dotfill\ldots 12mo, & 2\ 00\\ Smith's (O.) Press-working of Metals.\dotfill\ldots 8vo, & 3\ 00\\ Smith's (A.~W.) Materials of Machines.\dotfill\ldots 12mo, & 1\ 00\\ Smith (A.~W.) and Marx's Machine Design.\dotfill\ldots 8vo, & 3\ 00\\ Spangler, Greene, and Marshall's Elements of Steam-engineering.\dotfill\ldots 8vo, & 3\ 00\\ Thurston's Treatise on Friction and Lost Work in Machinery\\ \nopagebreak \indent\indent and Mill Work.\dotfill\ldots 8vo, & 3\ 00\\ \indent Animal as a Machine and Prime Motor, and the Laws\\ \nopagebreak \indent\indent of Energetics.\dotfill\ldots 12mo, & 1\ 00\\ Tillson's Complete Automobile Instructor.\dotfill\ldots 16mo, & 1\ 50\\ \hfill Morocco, & 2\ 00\\ Warren's Elements of Machine Construction and Drawing.\dotfill\ldots 8vo, & 7\ 50\\ Weisbach's Kinematics and Power of Transmission.\\ \nopagebreak \indent\indent (Herrmann---Klein.)\dotfill\ldots 8vo, & 5\ 00\\ \indent Machinery of Transmission and Governors.\\ \nopagebreak \indent\indent (Herrmann---Klein.)\dotfill\ldots 8vo, & 5\ 00\\ Wood's Elements of Analytical Mechanics.\dotfill\ldots 8vo, & 3\ 00\\ \indent Principles of Elementary Mechanics.\dotfill\ldots 12mo, & 1\ 25\\ \indent Turbines.\dotfill\ldots 8vo, & 2\ 50\\ The World's Columbian Exposition of 1893.\dotfill\ldots 4to, & 1\ 00\\[3em] \multicolumn{2}{c}{\large MEDICAL.}\\[1em] \nopagebreak De Fursac's Manual of Psychiatry. (Rosanoff and Collins.)\dotfill\ldots Large 12mo, & 2\ 50\\ Ehrlich's Collected Studies on Immunity. (Bolduan.)\dotfill\ldots 8vo, & 6\ 00\\ Hammarsten's Text-book on Physiological Chemistry. (Mandel.)\dotfill\ldots 8vo, & 4\ 00\\ %-----File: 257.png---Index 17------- Lassar-Cohn's Practical Urinary Analysis. (Lorenz.)\dotfill\ldots 12mo, & 1\ 00\\ * Pauli's Physical Chemistry in the Service\\ \nopagebreak \indent\indent of Medicine. (Fischer.)\dotfill\ldots 12mo, & 1\ 25\\ * Pozzi-Escot's The Toxins and Venoms and\\ \nopagebreak \indent\indent their Antibodies. (Cohn.)\dotfill\ldots 12mo, & 1\ 00\\ Rostoski's Serum Diagnosis. (Bolduan.)\dotfill\ldots 12mo, & 1\ 00\\ Salkowski's Physiological and Pathological Chemistry. (Orndorff.)\dotfill\ldots 8vo, & 2\ 50\\ * Satterlee's Outlines of Human Embryology.\dotfill\ldots 12mo, & 1\ 25\\ Steel's Treatise on the Diseases of the Dog.\dotfill\ldots 8vo, & 3\ 50\\ Von Behring's Suppression of Tuberculosis. (Bolduan.)\dotfill\ldots 12mo, & 1\ 00\\ Wassermann's Immune Sera: H\ae{}molysis, Cytotoxins,\\ \nopagebreak \indent\indent and Precipitins.\\ \indent\indent (Bolduan.)\dotfill\ldots 12mo, cloth, & 1\ 00\\ Woodhull's Notes on Military Hygiene.\dotfill\ldots 16mo, & 1\ 50\\ \makebox[0pt]{\hspace{.5ex} *}\indent Personal Hygiene.\dotfill\ldots 12mo, & 1\ 00\\ Wulling's An Elementary Course in Inorganic Pharmaceutical\\ \indent\indent and Medical Chemistry.\dotfill\ldots 12mo, & 2\ 00\\[3em] \multicolumn{2}{c}{\large METALLURGY.}\\[1em] \nopagebreak Egleston's Metallurgy of Silver, Gold, and Mercury:\\ \indent Vol.~\phantom{I}I.\quad Silver.\dotfill\ldots 8vo, & 7\ 50\\ \indent Vol.~II.\quad Gold and Mercury.\dotfill\ldots 8vo, & 7\ 50\\ Goesel's Minerals and Metals: A Reference Book.\dotfill\ldots 16mo, mor. & 3\ 00\\ * Iles's Lead-smelting.\dotfill\ldots 12mo, & 2\ 50\\ Keep's Cast Iron.\dotfill\ldots 8vo, & 2\ 50\\ Kunhardt's Practice of Ore Dressing in Europe.\dotfill\ldots 8vo, & 1\ 50\\ Le Chatelier's High-temperature Measurements.\\ \nopagebreak \indent\indent (Boudouard---Burgess.)\dotfill\ldots 12mo, & 3\ 00\\ Metcalf's Steel. A Manual for Steel-users.\dotfill\ldots 12mo, & 2\ 00\\ Miller's Cyanide Process.\dotfill\ldots 12mo, & 1\ 00\\ Minet's Production of Aluminum and its Industrial Use. (Waldo.)\dotfill\ldots 12mo, & 2\ 50\\ Robine and Lenglen's Cyanide Industry. (Le Clerc.)\dotfill\ldots 8vo, & 4\ 00\\ Smith's Materials of Machines.\dotfill\ldots 12mo, & 1\ 00\\ Thurston's Materials of Engineering. In Three Parts.\dotfill\ldots 8vo, & 8\ 00\\ \indent Part~II.\quad Iron and Steel.\dotfill\ldots 8vo, & 3\ 50\\ \indent Part~III.\quad A Treatise on Brasses, Bronzes, and Other Alloys\\ \indent\indent and their Constituents.\dotfill\ldots 8vo, & 2\ 50\\ Ulke's Modern Electrolytic Copper Refining.\dotfill\ldots 8vo, & 3\ 00\\[3em] \multicolumn{2}{c}{\large MINERALOGY.}\\[1em] \nopagebreak Barringer's Description of Minerals\\ \nopagebreak \indent\indent of Commercial Value.\dotfill\ldots Oblong, morocco, & 2\ 50\\ Boyd's Resources of Southwest Virginia.\dotfill\ldots 8vo, & 3\ 00\\ \indent Map of Southwest \correction{Virginia}{Virignia}.\dotfill\ldots Pocket-book form, & 2\ 00\\ Brush's Manual of Determinative Mineralogy. (Penfield.)\dotfill\ldots 8vo, & 4\ 00\\ Chester's Catalogue of Minerals.\dotfill\ldots 8vo, paper, & 1\ 00\\ \hfill Cloth, & 1\ 25\\ \indent Dictionary of the Names of Minerals.\dotfill\ldots 8vo, & 3\ 50\\ Dana's System of Mineralogy.\dotfill\ldots Large 8vo, half leather, & 12\ 50\\ \indent First Appendix to Dana's New ``System of Mineralogy.''\dotfill\ldots Large 8vo, & 1\ 00\\ \indent Text-book of Mineralogy.\dotfill\ldots 8vo, & 4\ 00\\ \indent Minerals and How to Study Them.\dotfill\ldots 12mo, & 1\ 50\\ \indent Catalogue of American Localities of Minerals.\dotfill\ldots Large 8vo, & 1\ 00\\ \indent Manual of Mineralogy and Petrography.\dotfill\ldots 12mo, & 2\ 00\\ Douglas's Untechnical Addresses on Technical Subjects.\dotfill\ldots 12mo, & 1\ 00\\ Eakle's Mineral Tables.\dotfill\ldots 8vo, & 1\ 25\\ Egleston's Catalogue of Minerals and Synonyms.\dotfill\ldots 8vo, & 2\ 50\\ Goesel's Minerals and Metals: A Reference Book.\dotfill\ldots 16mo, mor. & 3\ 00\\ Groth's Introduction to Chemical Crystallography (Marshall)\dotfill\ldots 12mo, & 1\ 25\\ %-----File: 258.png---Index 18------- Iddings's Rock Minerals.\dotfill\ldots 8vo, & 5\ 00\\ Merrill's Non-metallic Minerals: Their Occurrence and Uses.\dotfill\ldots 8vo, & 4\ 00\\ * Penfield's Notes on Determinative Mineralogy and Record\\ \nopagebreak \indent\indent of Mineral Tests.\dotfill\ldots 8vo, paper, & \ 50\\ * Richards's Synopsis of Mineral Characters.\dotfill\ldots 12mo, morocco, & 1\ 25\\ * Ries's Clays: Their Occurrence, Properties, and Uses.\dotfill\ldots 8vo, & 5 00 \\ Rosenbusch's Microscopical Physiography of\\ \nopagebreak \indent\indent the Rock-making Minerals. (Iddings.)\dotfill\ldots 8vo, & 5\ 00\\ * Tillman's Text-book of Important Minerals and Rocks.\dotfill\ldots 8vo, & 2\ 00\\[3em] \multicolumn{2}{c}{\large MINING.}\\[1em] \nopagebreak Boyd's Resources of Southwest Virginia.\dotfill\ldots 8vo, & 3\ 00\\ \indent Map of Southwest Virginia.\dotfill\ldots Pocket-book form, & 2\ 00\\ Douglas's Untechnical Addresses on Technical Subjects.\dotfill\ldots 12mo, & 1\ 00\\ Eissler's Modern High Explosives.\dotfill\ldots 8vo, & 4\ 00\\ Goesel's Minerals and Metals: A Reference Book.\dotfill\ldots 16mo, mor. & 3\ 00\\ Goodyear's Coal-mines of the Western Coast of the United States.\dotfill\ldots 12mo, & 2\ 50\\ Ihlseng's Manual of Mining.\dotfill\ldots 8vo, & 5\ 00\\ * Iles's Lead-smelting.\dotfill\ldots 12mo, & 2\ 50\\ Kunhardt's Practice of Ore Dressing In Europe.\dotfill\ldots 8vo, & 1\ 50\\ Miller's Cyanide Process.\dotfill\ldots 12mo, & 1\ 00\\ O'Driscoll's Notes on the Treatment of Gold Ores.\dotfill\ldots 8vo, & 2\ 00\\ Robine and Lenglen's Cyanide Industry. (Le Clerc.)\dotfill\ldots 8vo, & 4\ 00\\ * Walke's Lectures on Explosives.\dotfill\ldots 8vo, & 4\ 00\\ Weaver's Military Explosives.\dotfill\ldots 8vo, & 3\ 00\\ Wilson's Cyanide Processes.\dotfill\ldots 12mo, & 1\ 50\\ \indent Chlorination Process.\dotfill\ldots 12mo, & 1\ 50\\ \indent Hydraulic and Placer Mining.\dotfill\ldots 12mo, & 2\ 00\\ \indent Treatise on Practical and Theoretical Mine Ventilation.\dotfill\ldots 12mo, & 1\ 25\\[3em] \multicolumn{2}{c}{\large SANITARY SCIENCE.}\\[1em] \nopagebreak Bashore's Sanitation of a Country House.\dotfill\ldots 12mo, & 1\ 00\\ \makebox[0pt]{\hspace{.5ex} *}\indent Outlines of Practical Sanitation.\dotfill\ldots 12mo, & 1\ 25\\ Folwell's Sewerage. (Designing, Construction, and Maintenance.)\dotfill\ldots 8vo, & 3\ 00\\ \indent Water-supply Engineering.\dotfill\ldots 8vo, & 4\ 00\\ Fowler's Sewage Works Analyses.\dotfill\ldots 12mo, & 2\ 00\\ Fuertes's Water and Public Health.\dotfill\ldots 12mo, & 1\ 50\\ \indent Water-filtration Works.\dotfill\ldots 12mo, & 2\ 50\\ Gerhard's Guide to Sanitary House-inspection.\dotfill\ldots 16mo, & 1\ 00\\ Hazen's Filtration of Public Water-supplies.\dotfill\ldots 8vo, & 3\ 00\\ Leach's The Inspection and Analysis of Food with Special Reference\\ \indent\indent to State Control.\dotfill\ldots 8vo, & 7\ 50\\ Mason's Water-supply (Considered principally from\\ \nopagebreak \indent\indent a Sanitary Standpoint)\dotfill\ldots 8vo, & 4\ 00\\ \indent Examination of Water. (Chemical and Bacteriological.)\dotfill\ldots 12mo, & 1\ 25\\ * Merriman's Elements of Sanitary Engineering.\dotfill\ldots 8vo, & 2\ 00\\ Ogden's Sewer Design.\dotfill\ldots 12mo, & 2\ 00\\ Prescott and Winslow's Elements of Water Bacteriology, with Special\\ \indent\indent Reference to Sanitary Water Analysis.\dotfill\ldots 12mo, & 1\ 25\\ * Price's Handbook on Sanitation.\dotfill\ldots 12mo, & 1\ 50\\ Richards's Cost of Food. A Study in Dietaries.\dotfill\ldots 12mo, & 1\ 00\\ \indent Cost of Living as Modified by Sanitary Science.\dotfill\ldots 12mo, & 1\ 00\\ \indent Cost of Shelter.\dotfill\ldots 12mo, & 1\ 00\\ %-----File: 259.png---Index 19------- Richards and Woodman's Air, Water, and Food from a Sanitary\\ \nopagebreak \indent\indent Standpoint.\dotfill\ldots 8vo, &2\ 00\\ * Richards and Williams's The Dietary Computer.\dotfill\ldots 8vo, &1\ 50\\ Rideal's Sewage and Bacterial Purification of Sewage.\dotfill\ldots 8vo, &4\ 00\\ Turneaure and Russell's Public Water-supplies.\dotfill\ldots 8vo, &5\ 00\\ Von Behring's Suppression of Tuberculosis. (Bolduan.)\dotfill 12mo, &1\ 00\\ Whipple's Microscopy of Drinking-water.\dotfill\ldots 8vo, &3\ 50\\ Winton's Microscopy of Vegetable Foods.\dotfill\ldots 8vo, &7\ 50\\ Woodhull's Notes on Military Hygiene.\dotfill\ldots 16mo, &1\ 50\\ \makebox[0pt]{\hspace{.5ex} *}\indent Personal Hygiene.\dotfill\ldots 12mo, &1\ 00\\[3em] \multicolumn{2}{c}{\large MISCELLANEOUS.}\\[1em] \nopagebreak Emmons's Geological Guide-book of the Rocky Mountain Excursion\\ \nopagebreak \indent\indent of the International Congress of Geologists.\dotfill\ldots Large 8vo, &1\ 50\\ Ferrel's Popular Treatise on the Winds.\dotfill\ldots 8vo, &4\ 00\\ Gannett's Statistical Abstract of the World.\dotfill\ldots 24mo, & \ 75\\ Haines's American Railway Management.\dotfill\ldots 12mo, &2\ 50\\ Ricketts's History of Rensselaer Polytechnic Institute,\\ \nopagebreak \indent\indent 1824--1894.\dotfill\ldots Small 8vo, &3\ 00\\ Rotherham's Emphasized New Testament.\dotfill\ldots Large 8vo, &3\ 00\\ The World's Columbian Exposition of 1893.\dotfill\ldots 4to, &1 00\\ Winslow's Elements of Applied Microscopy.\dotfill\ldots 12mo, &1 50\\[3em] \multicolumn{2}{c}{\large HEBREW AND CHALDEE TEXT-BOOKS.}\\[1em] \nopagebreak Green's Elementary Hebrew Grammar.\dotfill\ldots 12mo, &1 25\\ \indent Hebrew Chrestomathy..\dotfill 8vo, &2 00\\ Gesenius's Hebrew and Chaldee Lexicon to the Old Testament\\ \nopagebreak \indent\indent Scriptures. (Tregelles.)\dotfill\ldots Small 4to, half morocco, &5 00\\ Letteris's Hebrew Bible.\dotfill\ldots 8vo, &2 25\\ \end{longtable} \newpage \small \pagenumbering{gobble} \begin{verbatim} End of Project Gutenberg's Introduction to Infinitesimal Analysis by Oswald Veblen and N. J. Lennes *** END OF THIS PROJECT GUTENBERG EBOOK INFINITESIMAL ANALYSIS *** *** This file should be named 18741-t.tex or 18741-t.zip *** *** or 18741-pdf.pdf or 18741-pdf.pdf *** This and all associated files of various formats will be found in: http://www.gutenberg.org/1/8/7/4/18741/ Produced by K.F. Greiner, Joshua Hutchinson, Laura Wisewell, Owen Whitby and the Online Distributed Proofreading Team at http://www.pgdp.net (This file was produced from images generously made available by Cornell University Digital Collections.) Updated editions will replace the previous one--the old editions will be renamed. Creating the works from public domain print editions means that no one owns a United States copyright in these works, so the Foundation (and you!) can copy and distribute it in the United States without permission and without paying copyright royalties. Special rules, set forth in the General Terms of Use part of this license, apply to copying and distributing Project Gutenberg-tm electronic works to protect the PROJECT GUTENBERG-tm concept and trademark. Project Gutenberg is a registered trademark, and may not be used if you charge for the eBooks, unless you receive specific permission. If you do not charge anything for copies of this eBook, complying with the rules is very easy. You may use this eBook for nearly any purpose such as creation of derivative works, reports, performances and research. They may be modified and printed and given away--you may do practically ANYTHING with public domain eBooks. Redistribution is subject to the trademark license, especially commercial redistribution. *** START: FULL LICENSE *** THE FULL PROJECT GUTENBERG LICENSE PLEASE READ THIS BEFORE YOU DISTRIBUTE OR USE THIS WORK To protect the Project Gutenberg-tm mission of promoting the free distribution of electronic works, by using or distributing this work (or any other work associated in any way with the phrase "Project Gutenberg"), you agree to comply with all the terms of the Full Project Gutenberg-tm License (available with this file or online at http://gutenberg.net/license). Section 1. General Terms of Use and Redistributing Project Gutenberg-tm electronic works 1.A. By reading or using any part of this Project Gutenberg-tm electronic work, you indicate that you have read, understand, agree to and accept all the terms of this license and intellectual property (trademark/copyright) agreement. If you do not agree to abide by all the terms of this agreement, you must cease using and return or destroy all copies of Project Gutenberg-tm electronic works in your possession. If you paid a fee for obtaining a copy of or access to a Project Gutenberg-tm electronic work and you do not agree to be bound by the terms of this agreement, you may obtain a refund from the person or entity to whom you paid the fee as set forth in paragraph 1.E.8. 1.B. "Project Gutenberg" is a registered trademark. It may only be used on or associated in any way with an electronic work by people who agree to be bound by the terms of this agreement. There are a few things that you can do with most Project Gutenberg-tm electronic works even without complying with the full terms of this agreement. See paragraph 1.C below. There are a lot of things you can do with Project Gutenberg-tm electronic works if you follow the terms of this agreement and help preserve free future access to Project Gutenberg-tm electronic works. See paragraph 1.E below. 1.C. The Project Gutenberg Literary Archive Foundation ("the Foundation" or PGLAF), owns a compilation copyright in the collection of Project Gutenberg-tm electronic works. Nearly all the individual works in the collection are in the public domain in the United States. If an individual work is in the public domain in the United States and you are located in the United States, we do not claim a right to prevent you from copying, distributing, performing, displaying or creating derivative works based on the work as long as all references to Project Gutenberg are removed. Of course, we hope that you will support the Project Gutenberg-tm mission of promoting free access to electronic works by freely sharing Project Gutenberg-tm works in compliance with the terms of this agreement for keeping the Project Gutenberg-tm name associated with the work. You can easily comply with the terms of this agreement by keeping this work in the same format with its attached full Project Gutenberg-tm License when you share it without charge with others. 1.D. The copyright laws of the place where you are located also govern what you can do with this work. Copyright laws in most countries are in a constant state of change. If you are outside the United States, check the laws of your country in addition to the terms of this agreement before downloading, copying, displaying, performing, distributing or creating derivative works based on this work or any other Project Gutenberg-tm work. The Foundation makes no representations concerning the copyright status of any work in any country outside the United States. 1.E. Unless you have removed all references to Project Gutenberg: 1.E.1. The following sentence, with active links to, or other immediate access to, the full Project Gutenberg-tm License must appear prominently whenever any copy of a Project Gutenberg-tm work (any work on which the phrase "Project Gutenberg" appears, or with which the phrase "Project Gutenberg" is associated) is accessed, displayed, performed, viewed, copied or distributed: This eBook is for the use of anyone anywhere at no cost and with almost no restrictions whatsoever. You may copy it, give it away or re-use it under the terms of the Project Gutenberg License included with this eBook or online at www.gutenberg.net 1.E.2. If an individual Project Gutenberg-tm electronic work is derived from the public domain (does not contain a notice indicating that it is posted with permission of the copyright holder), the work can be copied and distributed to anyone in the United States without paying any fees or charges. If you are redistributing or providing access to a work with the phrase "Project Gutenberg" associated with or appearing on the work, you must comply either with the requirements of paragraphs 1.E.1 through 1.E.7 or obtain permission for the use of the work and the Project Gutenberg-tm trademark as set forth in paragraphs 1.E.8 or 1.E.9. 1.E.3. If an individual Project Gutenberg-tm electronic work is posted with the permission of the copyright holder, your use and distribution must comply with both paragraphs 1.E.1 through 1.E.7 and any additional terms imposed by the copyright holder. Additional terms will be linked to the Project Gutenberg-tm License for all works posted with the permission of the copyright holder found at the beginning of this work. 1.E.4. Do not unlink or detach or remove the full Project Gutenberg-tm License terms from this work, or any files containing a part of this work or any other work associated with Project Gutenberg-tm. 1.E.5. Do not copy, display, perform, distribute or redistribute this electronic work, or any part of this electronic work, without prominently displaying the sentence set forth in paragraph 1.E.1 with active links or immediate access to the full terms of the Project Gutenberg-tm License. 1.E.6. You may convert to and distribute this work in any binary, compressed, marked up, nonproprietary or proprietary form, including any word processing or hypertext form. However, if you provide access to or distribute copies of a Project Gutenberg-tm work in a format other than "Plain Vanilla ASCII" or other format used in the official version posted on the official Project Gutenberg-tm web site (www.gutenberg.net), you must, at no additional cost, fee or expense to the user, provide a copy, a means of exporting a copy, or a means of obtaining a copy upon request, of the work in its original "Plain Vanilla ASCII" or other form. Any alternate format must include the full Project Gutenberg-tm License as specified in paragraph 1.E.1. 1.E.7. Do not charge a fee for access to, viewing, displaying, performing, copying or distributing any Project Gutenberg-tm works unless you comply with paragraph 1.E.8 or 1.E.9. 1.E.8. You may charge a reasonable fee for copies of or providing access to or distributing Project Gutenberg-tm electronic works provided that - You pay a royalty fee of 20% of the gross profits you derive from the use of Project Gutenberg-tm works calculated using the method you already use to calculate your applicable taxes. The fee is owed to the owner of the Project Gutenberg-tm trademark, but he has agreed to donate royalties under this paragraph to the Project Gutenberg Literary Archive Foundation. Royalty payments must be paid within 60 days following each date on which you prepare (or are legally required to prepare) your periodic tax returns. Royalty payments should be clearly marked as such and sent to the Project Gutenberg Literary Archive Foundation at the address specified in Section 4, "Information about donations to the Project Gutenberg Literary Archive Foundation." - You provide a full refund of any money paid by a user who notifies you in writing (or by e-mail) within 30 days of receipt that s/he does not agree to the terms of the full Project Gutenberg-tm License. You must require such a user to return or destroy all copies of the works possessed in a physical medium and discontinue all use of and all access to other copies of Project Gutenberg-tm works. - You provide, in accordance with paragraph 1.F.3, a full refund of any money paid for a work or a replacement copy, if a defect in the electronic work is discovered and reported to you within 90 days of receipt of the work. - You comply with all other terms of this agreement for free distribution of Project Gutenberg-tm works. 1.E.9. If you wish to charge a fee or distribute a Project Gutenberg-tm electronic work or group of works on different terms than are set forth in this agreement, you must obtain permission in writing from both the Project Gutenberg Literary Archive Foundation and Michael Hart, the owner of the Project Gutenberg-tm trademark. Contact the Foundation as set forth in Section 3 below. 1.F. 1.F.1. Project Gutenberg volunteers and employees expend considerable effort to identify, do copyright research on, transcribe and proofread public domain works in creating the Project Gutenberg-tm collection. Despite these efforts, Project Gutenberg-tm electronic works, and the medium on which they may be stored, may contain "Defects," such as, but not limited to, incomplete, inaccurate or corrupt data, transcription errors, a copyright or other intellectual property infringement, a defective or damaged disk or other medium, a computer virus, or computer codes that damage or cannot be read by your equipment. 1.F.2. LIMITED WARRANTY, DISCLAIMER OF DAMAGES - Except for the "Right of Replacement or Refund" described in paragraph 1.F.3, the Project Gutenberg Literary Archive Foundation, the owner of the Project Gutenberg-tm trademark, and any other party distributing a Project Gutenberg-tm electronic work under this agreement, disclaim all liability to you for damages, costs and expenses, including legal fees. YOU AGREE THAT YOU HAVE NO REMEDIES FOR NEGLIGENCE, STRICT LIABILITY, BREACH OF WARRANTY OR BREACH OF CONTRACT EXCEPT THOSE PROVIDED IN PARAGRAPH F3. YOU AGREE THAT THE FOUNDATION, THE TRADEMARK OWNER, AND ANY DISTRIBUTOR UNDER THIS AGREEMENT WILL NOT BE LIABLE TO YOU FOR ACTUAL, DIRECT, INDIRECT, CONSEQUENTIAL, PUNITIVE OR INCIDENTAL DAMAGES EVEN IF YOU GIVE NOTICE OF THE POSSIBILITY OF SUCH DAMAGE. 1.F.3. LIMITED RIGHT OF REPLACEMENT OR REFUND - If you discover a defect in this electronic work within 90 days of receiving it, you can receive a refund of the money (if any) you paid for it by sending a written explanation to the person you received the work from. If you received the work on a physical medium, you must return the medium with your written explanation. The person or entity that provided you with the defective work may elect to provide a replacement copy in lieu of a refund. If you received the work electronically, the person or entity providing it to you may choose to give you a second opportunity to receive the work electronically in lieu of a refund. If the second copy is also defective, you may demand a refund in writing without further opportunities to fix the problem. 1.F.4. Except for the limited right of replacement or refund set forth in paragraph 1.F.3, this work is provided to you 'AS-IS', WITH NO OTHER WARRANTIES OF ANY KIND, EXPRESS OR IMPLIED, INCLUDING BUT NOT LIMITED TO WARRANTIES OF MERCHANTIBILITY OR FITNESS FOR ANY PURPOSE. 1.F.5. Some states do not allow disclaimers of certain implied warranties or the exclusion or limitation of certain types of damages. If any disclaimer or limitation set forth in this agreement violates the law of the state applicable to this agreement, the agreement shall be interpreted to make the maximum disclaimer or limitation permitted by the applicable state law. The invalidity or unenforceability of any provision of this agreement shall not void the remaining provisions. 1.F.6. INDEMNITY - You agree to indemnify and hold the Foundation, the trademark owner, any agent or employee of the Foundation, anyone providing copies of Project Gutenberg-tm electronic works in accordance with this agreement, and any volunteers associated with the production, promotion and distribution of Project Gutenberg-tm electronic works, harmless from all liability, costs and expenses, including legal fees, that arise directly or indirectly from any of the following which you do or cause to occur: (a) distribution of this or any Project Gutenberg-tm work, (b) alteration, modification, or additions or deletions to any Project Gutenberg-tm work, and (c) any Defect you cause. Section 2. Information about the Mission of Project Gutenberg-tm Project Gutenberg-tm is synonymous with the free distribution of electronic works in formats readable by the widest variety of computers including obsolete, old, middle-aged and new computers. It exists because of the efforts of hundreds of volunteers and donations from people in all walks of life. Volunteers and financial support to provide volunteers with the assistance they need, is critical to reaching Project Gutenberg-tm's goals and ensuring that the Project Gutenberg-tm collection will remain freely available for generations to come. In 2001, the Project Gutenberg Literary Archive Foundation was created to provide a secure and permanent future for Project Gutenberg-tm and future generations. To learn more about the Project Gutenberg Literary Archive Foundation and how your efforts and donations can help, see Sections 3 and 4 and the Foundation web page at http://www.pglaf.org. Section 3. Information about the Project Gutenberg Literary Archive Foundation The Project Gutenberg Literary Archive Foundation is a non profit 501(c)(3) educational corporation organized under the laws of the state of Mississippi and granted tax exempt status by the Internal Revenue Service. The Foundation's EIN or federal tax identification number is 64-6221541. Its 501(c)(3) letter is posted at http://pglaf.org/fundraising. Contributions to the Project Gutenberg Literary Archive Foundation are tax deductible to the full extent permitted by U.S. federal laws and your state's laws. The Foundation's principal office is located at 4557 Melan Dr. S. Fairbanks, AK, 99712., but its volunteers and employees are scattered throughout numerous locations. Its business office is located at 809 North 1500 West, Salt Lake City, UT 84116, (801) 596-1887, email business@pglaf.org. Email contact links and up to date contact information can be found at the Foundation's web site and official page at http://pglaf.org For additional contact information: Dr. Gregory B. Newby Chief Executive and Director gbnewby@pglaf.org Section 4. Information about Donations to the Project Gutenberg Literary Archive Foundation Project Gutenberg-tm depends upon and cannot survive without wide spread public support and donations to carry out its mission of increasing the number of public domain and licensed works that can be freely distributed in machine readable form accessible by the widest array of equipment including outdated equipment. Many small donations ($1 to $5,000) are particularly important to maintaining tax exempt status with the IRS. The Foundation is committed to complying with the laws regulating charities and charitable donations in all 50 states of the United States. Compliance requirements are not uniform and it takes a considerable effort, much paperwork and many fees to meet and keep up with these requirements. We do not solicit donations in locations where we have not received written confirmation of compliance. To SEND DONATIONS or determine the status of compliance for any particular state visit http://pglaf.org While we cannot and do not solicit contributions from states where we have not met the solicitation requirements, we know of no prohibition against accepting unsolicited donations from donors in such states who approach us with offers to donate. International donations are gratefully accepted, but we cannot make any statements concerning tax treatment of donations received from outside the United States. U.S. laws alone swamp our small staff. Please check the Project Gutenberg Web pages for current donation methods and addresses. Donations are accepted in a number of other ways including including checks, online payments and credit card donations. To donate, please visit: http://pglaf.org/donate Section 5. General Information About Project Gutenberg-tm electronic works. Professor Michael S. Hart is the originator of the Project Gutenberg-tm concept of a library of electronic works that could be freely shared with anyone. For thirty years, he produced and distributed Project Gutenberg-tm eBooks with only a loose network of volunteer support. Project Gutenberg-tm eBooks are often created from several printed editions, all of which are confirmed as Public Domain in the U.S. unless a copyright notice is included. Thus, we do not necessarily keep eBooks in compliance with any particular paper edition. Most people start at our Web site which has the main PG search facility: http://www.gutenberg.net This Web site includes information about Project Gutenberg-tm, including how to make donations to the Project Gutenberg Literary Archive Foundation, how to help produce our new eBooks, and how to subscribe to our email newsletter to hear about new eBooks. *** END: FULL LICENSE *** \end{verbatim} \end{document} %%%%%%%%%%%%%%%%%%%%%%%%%%%%% LOGFILE %%%%%%%%%%%%%%%%%%%%%%%%%%%%%%% This is pdfeTeX, Version 3.141592-1.21a-2.2 (MiKTeX 2.4) (preloaded format=latex 2006.2.20) 2 JUL 2006 18:54 entering extended mode **18741-t (18741-t.tex LaTeX2e <2003/12/01> Babel and hyphenation patterns for english, dumylang, nohyphenation, ge rman, ngerman, french, loaded. (C:\texmf\tex\latex\base\book.cls Document Class: book 2004/02/16 v1.4f Standard LaTeX document class (C:\texmf\tex\latex\base\bk12.clo File: bk12.clo 2004/02/16 v1.4f Standard LaTeX file (size option) ) \c@part=\count79 \c@chapter=\count80 \c@section=\count81 \c@subsection=\count82 \c@subsubsection=\count83 \c@paragraph=\count84 \c@subparagraph=\count85 \c@figure=\count86 \c@table=\count87 \abovecaptionskip=\skip41 \belowcaptionskip=\skip42 \bibindent=\dimen102 ) (C:\texmf\tex\latex\amsmath\amsmath.sty Package: amsmath 2000/07/18 v2.13 AMS math features \@mathmargin=\skip43 For additional information on amsmath, use the `?' option. (C:\texmf\tex\latex\amsmath\amstext.sty Package: amstext 2000/06/29 v2.01 (C:\texmf\tex\latex\amsmath\amsgen.sty File: amsgen.sty 1999/11/30 v2.0 \@emptytoks=\toks14 \ex@=\dimen103 )) (C:\texmf\tex\latex\amsmath\amsbsy.sty Package: amsbsy 1999/11/29 v1.2d \pmbraise@=\dimen104 ) (C:\texmf\tex\latex\amsmath\amsopn.sty Package: amsopn 1999/12/14 v2.01 operator names ) \inf@bad=\count88 LaTeX Info: Redefining \frac on input line 211. \uproot@=\count89 \leftroot@=\count90 LaTeX Info: Redefining \overline on input line 307. \classnum@=\count91 \DOTSCASE@=\count92 LaTeX Info: Redefining \ldots on input line 379. LaTeX Info: Redefining \dots on input line 382. LaTeX Info: Redefining \cdots on input line 467. \Mathstrutbox@=\box26 \strutbox@=\box27 \big@size=\dimen105 LaTeX Font Info: Redeclaring font encoding OML on input line 567. LaTeX Font Info: Redeclaring font encoding OMS on input line 568. \macc@depth=\count93 \c@MaxMatrixCols=\count94 \dotsspace@=\muskip10 \c@parentequation=\count95 \dspbrk@lvl=\count96 \tag@help=\toks15 \row@=\count97 \column@=\count98 \maxfields@=\count99 \andhelp@=\toks16 \eqnshift@=\dimen106 \alignsep@=\dimen107 \tagshift@=\dimen108 \tagwidth@=\dimen109 \totwidth@=\dimen110 \lineht@=\dimen111 \@envbody=\toks17 \multlinegap=\skip44 \multlinetaggap=\skip45 \mathdisplay@stack=\toks18 LaTeX Info: Redefining \[ on input line 2666. LaTeX Info: Redefining \] on input line 2667. ) (C:\texmf\tex\latex\amscls\amsthm.sty Package: amsthm 2004/08/06 v2.20 \thm@style=\toks19 \thm@bodyfont=\toks20 \thm@headfont=\toks21 \thm@notefont=\toks22 \thm@headpunct=\toks23 \thm@preskip=\skip46 \thm@postskip=\skip47 \thm@headsep=\skip48 \dth@everypar=\toks24 ) (C:\texmf\tex\latex\amsfonts\amssymb.sty Package: amssymb 2002/01/22 v2.2d (C:\texmf\tex\latex\amsfonts\amsfonts.sty Package: amsfonts 2001/10/25 v2.2f \symAMSa=\mathgroup4 \symAMSb=\mathgroup5 LaTeX Font Info: Overwriting math alphabet `\mathfrak' in version `bold' (Font) U/euf/m/n --> U/euf/b/n on input line 132. )) (C:\texmf\tex\latex\ltxmisc\a4wide.sty Package: a4wide 1994/08/30 (C:\texmf\tex\latex\ntgclass\a4.sty Package: a4 2004/04/15 v1.2g A4 based page layout )) (C:\texmf\tex\latex\stmaryrd\stmaryrd.sty Package: stmaryrd 1994/03/03 St Mary's Road symbol package \symstmry=\mathgroup6 LaTeX Font Info: Overwriting symbol font `stmry' in version `bold' (Font) U/stmry/m/n --> U/stmry/b/n on input line 89. ) (C:\texmf\tex\latex\eepic\epic.sty Enhancements to Picture Environment. Version 1.2 - Released June 1, 1986 \@@multicnt=\count100 \d@lta=\count101 \@delta=\dimen112 \@@delta=\dimen113 \@gridcnt=\count102 \@joinkind=\count103 \@dotgap=\dimen114 \@ddotgap=\dimen115 \@x@diff=\count104 \@y@diff=\count105 \x@diff=\dimen116 \y@diff=\dimen117 \@dotbox=\box28 \num@segments=\count106 \num@segmentsi=\count107 \@datafile=\read1 ) (C:\texmf\tex\latex\eepic\eepicemu.sty Emulation of EEPIC using EPIC. Version 1.1a - Released Febrary 1, 1988 \@spxcnt=\count108 \@spycnt=\count109 \@ispxcnt=\count110 \@ispycnt=\count111 \@cmidxcnt=\count112 \@cmidycnt=\count113 \maxovaldiam=\dimen118 ) (C:\texmf\tex\latex\tools\longtable.sty Package: longtable 2004/02/01 v4.11 Multi-page Table package (DPC) \LTleft=\skip49 \LTright=\skip50 \LTpre=\skip51 \LTpost=\skip52 \LTchunksize=\count114 \LTcapwidth=\dimen119 \LT@head=\box29 \LT@firsthead=\box30 \LT@foot=\box31 \LT@lastfoot=\box32 \LT@cols=\count115 \LT@rows=\count116 \c@LT@tables=\count117 \c@LT@chunks=\count118 \LT@p@ftn=\toks25 ) (C:\texmf\tex\latex\graphics\color.sty Package: color 1999/02/16 v1.0i Standard LaTeX Color (DPC) (C:\texmf\tex\latex\00miktex\color.cfg File: color.cfg 2005/12/29 v1.1 MiKTeX 'color' configuration ) Package color Info: Driver file: pdftex.def on input line 125. (C:\texmf\tex\latex\graphics\pdftex.def File: pdftex.def 2005/06/20 v0.03m graphics/color for pdftex \Gread@gobject=\count119 )) (C:\texmf\tex\latex\graphics\graphicx.sty Package: graphicx 1999/02/16 v1.0f Enhanced LaTeX Graphics (DPC,SPQR) (C:\texmf\tex\latex\graphics\keyval.sty Package: keyval 1999/03/16 v1.13 key=value parser (DPC) \KV@toks@=\toks26 ) (C:\texmf\tex\latex\graphics\graphics.sty Package: graphics 2001/07/07 v1.0n Standard LaTeX Graphics (DPC,SPQR) (C:\texmf\tex\latex\graphics\trig.sty Package: trig 1999/03/16 v1.09 sin cos tan (DPC) ) (C:\texmf\tex\latex\00miktex\graphics.cfg File: graphics.cfg 2005/12/29 v1.2 MiKTeX 'graphics' configuration ) Package graphics Info: Driver file: pdftex.def on input line 80. ) \Gin@req@height=\dimen120 \Gin@req@width=\dimen121 ) (C:\texmf\tex\latex\base\makeidx.sty Package: makeidx 2000/03/29 v1.0m Standard LaTeX package ) (C:\texmf\tex\latex\hyperref\hyperref.sty Package: hyperref 2003/11/30 v6.74m Hypertext links for LaTeX \@linkdim=\dimen122 \Hy@linkcounter=\count120 \Hy@pagecounter=\count121 (C:\texmf\tex\latex\hyperref\pd1enc.def File: pd1enc.def 2003/11/30 v6.74m Hyperref: PDFDocEncoding definition (HO) ) (C:\texmf\tex\latex\00miktex\hyperref.cfg File: hyperref.cfg 2003/03/08 v1.0 MiKTeX 'hyperref' configuration ) Package hyperref Info: Option `plainpages' set `false' on input line 1830. Package hyperref Info: Option `pdfpagelabels' set `true' on input line 1830. Package hyperref Info: Option `colorlinks' set `true' on input line 1830. Package hyperref Info: Hyper figures OFF on input line 1880. Package hyperref Info: Link nesting OFF on input line 1885. Package hyperref Info: Hyper index ON on input line 1888. Package hyperref Info: Plain pages OFF on input line 1895. Package hyperref Info: Backreferencing OFF on input line 1900. Implicit mode ON; LaTeX internals redefined Package hyperref Info: Bookmarks ON on input line 2004. (C:\texmf\tex\latex\ltxmisc\url.sty \Urlmuskip=\muskip11 Package: url 2005/06/27 ver 3.2 Verb mode for urls, etc. ) LaTeX Info: Redefining \url on input line 2143. \Fld@menulength=\count122 \Field@Width=\dimen123 \Fld@charsize=\dimen124 \Choice@toks=\toks27 \Field@toks=\toks28 Package hyperref Info: Hyper figures OFF on input line 2618. Package hyperref Info: Link nesting OFF on input line 2623. Package hyperref Info: Hyper index ON on input line 2626. Package hyperref Info: backreferencing OFF on input line 2633. Package hyperref Info: Link coloring ON on input line 2636. \Hy@abspage=\count123 \c@Item=\count124 \c@Hfootnote=\count125 ) *hyperref using driver hpdftex* (C:\texmf\tex\latex\hyperref\hpdftex.def File: hpdftex.def 2003/11/30 v6.74m Hyperref driver for pdfTeX (C:\texmf\tex\latex\psnfss\pifont.sty Package: pifont 2005/04/12 PSNFSS-v9.2a Pi font support (SPQR) LaTeX Font Info: Try loading font information for U+pzd on input line 63. (C:\texmf\tex\latex\psnfss\upzd.fd File: upzd.fd 2001/06/04 font definitions for U/pzd. ) LaTeX Font Info: Try loading font information for U+psy on input line 64. (C:\texmf\tex\latex\psnfss\upsy.fd File: upsy.fd 2001/06/04 font definitions for U/psy. )) \Fld@listcount=\count126 \@outlinefile=\write3 ) \intwidth=\skip53 \chevron=\skip54 LaTeX Font Info: Try loading font information for U+msa on input line 140. (C:\texmf\tex\latex\amsfonts\umsa.fd File: umsa.fd 2002/01/19 v2.2g AMS font definitions ) LaTeX Font Info: Try loading font information for U+msb on input line 140. (C:\texmf\tex\latex\amsfonts\umsb.fd File: umsb.fd 2002/01/19 v2.2g AMS font definitions ) LaTeX Font Info: Try loading font information for U+stmry on input line 140. (C:\texmf\tex\latex\stmaryrd\ustmry.fd) \equals=\skip55 \c@theorem=\count127 \c@other=\count128 \c@corollary=\count129 \c@ncorollary=\count130 \@indexfile=\write4 Writing index file 18741-t.idx (18741-t.aux) LaTeX Font Info: Checking defaults for OML/cmm/m/it on input line 192. LaTeX Font Info: ... okay on input line 192. LaTeX Font Info: Checking defaults for T1/cmr/m/n on input line 192. LaTeX Font Info: ... okay on input line 192. LaTeX Font Info: Checking defaults for OT1/cmr/m/n on input line 192. LaTeX Font Info: ... okay on input line 192. LaTeX Font Info: Checking defaults for OMS/cmsy/m/n on input line 192. LaTeX Font Info: ... okay on input line 192. LaTeX Font Info: Checking defaults for OMX/cmex/m/n on input line 192. LaTeX Font Info: ... okay on input line 192. LaTeX Font Info: Checking defaults for U/cmr/m/n on input line 192. LaTeX Font Info: ... okay on input line 192. LaTeX Font Info: Checking defaults for PD1/pdf/m/n on input line 192. LaTeX Font Info: ... okay on input line 192. (C:\texmf\tex\context\base\supp-pdf.tex (C:\texmf\tex\context\base\supp-mis.tex loading : Context Support Macros / Miscellaneous (2004.10.26) \protectiondepth=\count131 \scratchcounter=\count132 \scratchtoks=\toks29 \scratchdimen=\dimen125 \scratchskip=\skip56 \scratchmuskip=\muskip12 \scratchbox=\box33 \scratchread=\read2 \scratchwrite=\write5 \zeropoint=\dimen126 \onepoint=\dimen127 \onebasepoint=\dimen128 \minusone=\count133 \thousandpoint=\dimen129 \onerealpoint=\dimen130 \emptytoks=\toks30 \nextbox=\box34 \nextdepth=\dimen131 \everyline=\toks31 \!!counta=\count134 \!!countb=\count135 \recursecounter=\count136 ) loading : Context Support Macros / PDF (2004.03.26) \nofMPsegments=\count137 \nofMParguments=\count138 \MPscratchCnt=\count139 \MPscratchDim=\dimen132 \MPnumerator=\count140 \everyMPtoPDFconversion=\toks32 ) Package hyperref Info: Link coloring ON on input line 192. (C:\texmf\tex\latex\hyperref\nameref.sty Package: nameref 2003/12/03 v2.21 Cross-referencing by name of section \c@section@level=\count141 ) LaTeX Info: Redefining \ref on input line 192. LaTeX Info: Redefining \pageref on input line 192. (18741-t.out) (18741-t.out) [1 {psfonts.map}] [2 ] LaTeX Font Info: Try loading font information for OMS+cmtt on input line 255 . LaTeX Font Info: No file OMScmtt.fd. on input line 255. LaTeX Font Warning: Font shape `OMS/cmtt/m/n' undefined (Font) using `OMS/cmsy/m/n' instead (Font) for symbol `textbackslash' on input line 255. Underfull \hbox (badness 1571) in paragraph at lines 255--255 []\OT1/cmss/m/n/12 A large num-ber of printer er-rors have been cor-rected. The se are shaded [] Underfull \hbox (badness 4713) in paragraph at lines 255--255 [][][][][][]\OT1/cmss/m/n/12 , and de-tails can be found in the source code in the syn-tax [] [1 ] [2 ] ! pdfTeX warning (ext4): destination with the same identifier (name{page.i}) ha s been already used, duplicate ignored \penalty l.302 \begin {center} [1] ! pdfTeX warning (ext4): destination with the same identifier (name{page.ii}) h as been already used, duplicate ignored \penalty l.310 \newpage [2] [3 ] [4 ] (18741-t.toc LaTeX Font Info: Try loading font information for OMS+cmr on input line 2. (C:\texmf\tex\latex\base\omscmr.fd File: omscmr.fd 1999/05/25 v2.5h Standard LaTeX font definitions ) LaTeX Font Info: Font shape `OMS/cmr/m/n' in size <12> not available (Font) Font shape `OMS/cmsy/m/n' tried instead on input line 2. [5]) \tf@toc=\write6 [6] Chapter 1. LaTeX Font Info: Font shape `OMS/cmr/bx/n' in size <17.28> not available (Font) Font shape `OMS/cmsy/b/n' tried instead on input line 380. ! pdfTeX warning (ext4): destination with the same identifier (name{page.1}) ha s been already used, duplicate ignored \penalty l.431 \end{align*} [1 ] ! pdfTeX warning (ext4): destination with the same identifier (name{page.2}) ha s been already used, duplicate ignored \penalty l.500 \item[(3)] $1/2,1/{2^2},1/{2^3},\ldots,1/{2^n},\ldots$ [2] [3] [4] [5] [6] [7] [8] [9] [10] LaTeX Font Warning: Font shape `U/stmry/b/n' undefined (Font) using `U/stmry/m/n' instead on input line 1050. Package hyperref Warning: Token not allowed in a PDFDocEncoded string, (hyperref) removing `math shift' on input line 1050. Package hyperref Warning: Token not allowed in a PDFDocEncoded string, (hyperref) removing `math shift' on input line 1050. [11] [12] [13] Package hyperref Warning: Token not allowed in a PDFDocEncoded string, (hyperref) removing `math shift' on input line 1244. Package hyperref Warning: Token not allowed in a PDFDocEncoded string, (hyperref) removing `math shift' on input line 1244. [14] [15] [16] [17] Package hyperref Warning: Token not allowed in a PDFDocEncoded string, (hyperref) removing `math shift' on input line 1490. Package hyperref Warning: Token not allowed in a PDFDocEncoded string, (hyperref) removing `\pi' on input line 1490. Package hyperref Warning: Token not allowed in a PDFDocEncoded string, (hyperref) removing `math shift' on input line 1490. [18] [19] [20] [21] [22 ] Chapter 2. [23] [24] [25] [26] [27] [28] File: images/fig05.pdf Graphic file (type pdf) [29 ] [30] Package hyperref Warning: Token not allowed in a PDFDocEncoded string, (hyperref) removing `\-command' on input line 2317. Package hyperref Warning: Token not allowed in a PDFDocEncoded string, (hyperref) removing `\@ifnextchar' on input line 2317. [31] [32] Chapter 3. Package hyperref Warning: Token not allowed in a PDFDocEncoded string, (hyperref) removing `\spacefactor' on input line 2390. Package hyperref Warning: Token not allowed in a PDFDocEncoded string, (hyperref) removing `\@m' on input line 2390. [33 ] [34] [35] [36] [37] File: images/fig08.pdf Graphic file (type pdf) File: images/fig09.pdf Graphic file (type pdf) [38 ] [39 ] File: images/fig10.pdf Graphic file (type pdf) [40 ] Overfull \hbox (8.00644pt too wide) in paragraph at lines 2751--2751 []\OT1/cmr/bx/n/17.28 Rational, Ex-po-nen-tial, and Log-a-rith-mic Func-tions. [] LaTeX Font Info: Font shape `OMS/cmr/m/n' in size <10> not available (Font) Font shape `OMS/cmsy/m/n' tried instead on input line 2771. File: images/fig11.pdf Graphic file (type pdf) [41] [42 ] [43] [44] [45] [46 ] Chapter 4. [47] [48] File: images/fig13.pdf Graphic file (type pdf) [49] [50 ] [51] [52] [53] [54] [55] [56] [57] [58] [59] [60] [61] [62] [63] [64] [65] [66] [67] [68 ] Chapter 5. [69] [70] [71] [72] [73] [74] [75] [76] [77] [78] [79] [80 ] Chapter 6. [81] [82] [83] [84] [85] [86] [87] [88] [89] [90] [91] [92] Chapter 7. [93 ] [94] [95] [96] [97] [98] [99] [100] [101] [102] [103] [104] [105] [106] [107] [108] [109] [110] [111] [112] [113] [114] File: images/fig19.pdf Graphic file (type pdf) [115] [116 ] Overfull \hbox (5.61404pt too wide) in paragraph at lines 7294--7298 \OML/cmm/m/it/12 f[]\OT1/cmr/m/n/12 (\OML/cmm/m/it/12 x[]\OT1/cmr/m/n/12 ) \OML /cmm/m/it/12 > \OT1/cmr/m/n/12 0$, then, by The-o-rem [][]23[][], there ex-ists about the point $\OML/cmm/m/it/12 x[]$ \OT1/cmr/m/n/12 a seg-ment [][][][][][] , [] [117] [118] [119] [120] Chapter 8. [121 ] [122] File: images/fig21.pdf Graphic file (type pdf) [123 ] [124] [125] [126] [127] [128] [129] [130] [131] [132] [133] [134] Overfull \hbox (1.33414pt too wide) in paragraph at lines 8335--8341 []\OT1/cmr/m/n/12 The proof de-pends di-rectly upon the the-o-rem that if $[] \ OML/cmm/m/it/12 []\OT1/cmr/m/n/12 (\OML/cmm/m/it/12 x\OT1/cmr/m/n/12 ) = \OML/ cmm/m/it/12 b[]$\OT1/cmr/m/n/12 , and $[] \OML/cmm/m/it/12 []\OT1/cmr/m/n/12 ( \OML/cmm/m/it/12 x\OT1/cmr/m/n/12 ) = [] [135] [136] [137] [138] [139] [140] [141] [142] [143] [144] [145] [146] [147] [148] [149] [150] [151] [152] Chapter 9. [153 ] [154] [155] [156] [157] [158] [159] [160] [161] [162] [163] [164] [165] [166] [167] [168] [169] [170] [171] [172] [173] [174] [175] [176] [177] [178] [179] [180 ] (18741-t.ind [181 ] [182] [183]) ! pdfTeX warning (ext4): destination with the same identifier (name{page.1}) ha s been already used, duplicate ignored \penalty l.11289 [1 ] ! pdfTeX warning (ext4): destination with the same identifier (name{page.2}) ha s been already used, duplicate ignored \penalty l.11390 [2] ! pdfTeX warning (ext4): destination with the same identifier (name{page.3}) ha s been already used, duplicate ignored \penalty l.11485 [3] ! pdfTeX warning (ext4): destination with the same identifier (name{page.4}) ha s been already used, duplicate ignored \penalty l.11586 [4] ! pdfTeX warning (ext4): destination with the same identifier (name{page.5}) ha s been already used, duplicate ignored \penalty l.11748 [5] ! pdfTeX warning (ext4): destination with the same identifier (name{page.6}) ha s been already used, duplicate ignored \penalty l.11862 [6] ! pdfTeX warning (ext4): destination with the same identifier (name{page.7}) ha s been already used, duplicate ignored \penalty l.12039 [7] ! pdfTeX warning (ext4): destination with the same identifier (name{page.8}) ha s been already used, duplicate ignored \penalty l.12160 [8] ! pdfTeX warning (ext4): destination with the same identifier (name{page.9}) ha s been already used, duplicate ignored \penalty l.12329 [9] ! pdfTeX warning (ext4): destination with the same identifier (name{page.10}) h as been already used, duplicate ignored \penalty l.12451 [10] ! pdfTeX warning (ext4): destination with the same identifier (name{page.11}) h as been already used, duplicate ignored \penalty l.12574 [11] ! pdfTeX warning (ext4): destination with the same identifier (name{page.12}) h as been already used, duplicate ignored \penalty l.12747 [12] ! pdfTeX warning (ext4): destination with the same identifier (name{page.13}) h as been already used, duplicate ignored \penalty l.12864 [13] ! pdfTeX warning (ext4): destination with the same identifier (name{page.14}) h as been already used, duplicate ignored \penalty l.12983 [14] ! pdfTeX warning (ext4): destination with the same identifier (name{page.15}) h as been already used, duplicate ignored \penalty l.13116 [15] ! pdfTeX warning (ext4): destination with the same identifier (name{page.16}) h as been already used, duplicate ignored \penalty l.13217 [16] ! pdfTeX warning (ext4): destination with the same identifier (name{page.17}) h as been already used, duplicate ignored \penalty l.13372 [17] ! pdfTeX warning (ext4): destination with the same identifier (name{page.18}) h as been already used, duplicate ignored \penalty l.13479 [18] ! pdfTeX warning (ext4): destination with the same identifier (name{page.19}) h as been already used, duplicate ignored \penalty l.13579 [19] ! pdfTeX warning (ext4): destination with the same identifier (name{page.20}) h as been already used, duplicate ignored \penalty l.13745 [20] ! pdfTeX warning (ext4): destination with the same identifier (name{page.21}) h as been already used, duplicate ignored \penalty l.13852 [21] ! pdfTeX warning (ext4): destination with the same identifier (name{page.22}) h as been already used, duplicate ignored \penalty l.13958 [22] ! pdfTeX warning (ext4): destination with the same identifier (name{page.23}) h as been already used, duplicate ignored \penalty l.13995 \newpage [23] Underfull \vbox (badness 10000) has occurred while \output is active [] [1] Underfull \vbox (badness 10000) has occurred while \output is active [] ! pdfTeX warning (ext4): destination with the same identifier (name{page.}) has been already used, duplicate ignored \penalty l.14381 \end{verbatim} [2] Underfull \vbox (badness 10000) has occurred while \output is active [] ! pdfTeX warning (ext4): destination with the same identifier (name{page.}) has been already used, duplicate ignored \penalty l.14381 \end{verbatim} [3] Underfull \vbox (badness 10000) has occurred while \output is active [] ! pdfTeX warning (ext4): destination with the same identifier (name{page.}) has been already used, duplicate ignored \penalty l.14381 \end{verbatim} [4] Underfull \vbox (badness 10000) has occurred while \output is active [] ! pdfTeX warning (ext4): destination with the same identifier (name{page.}) has been already used, duplicate ignored \penalty l.14381 \end{verbatim} [5] Underfull \vbox (badness 10000) has occurred while \output is active [] ! pdfTeX warning (ext4): destination with the same identifier (name{page.}) has been already used, duplicate ignored \penalty l.14381 \end{verbatim} [6] Underfull \vbox (badness 10000) has occurred while \output is active [] ! pdfTeX warning (ext4): destination with the same identifier (name{page.}) has been already used, duplicate ignored \penalty l.14381 \end{verbatim} [7] Underfull \vbox (badness 10000) has occurred while \output is active [] ! pdfTeX warning (ext4): destination with the same identifier (name{page.}) has been already used, duplicate ignored \penalty l.14381 \end{verbatim} [8] ! pdfTeX warning (ext4): destination with the same identifier (name{page.}) has been already used, duplicate ignored \penalty l.14383 \end{document} [9] (18741-t.aux) LaTeX Font Warning: Some font shapes were not available, defaults substituted. ) Here is how much of TeX's memory you used: 5467 strings out of 95515 61168 string characters out of 1189489 204084 words of memory out of 1120827 7311 multiletter control sequences out of 60000 24815 words of font info for 93 fonts, out of 1000000 for 2000 14 hyphenation exceptions out of 8191 27i,17n,40p,222b,455s stack positions out of 5000i,500n,10000p,200000b,32768s PDF statistics: 2896 PDF objects out of 300000 977 named destinations out of 300000 561 words of extra memory for PDF output out of 65536 Output written on 18741-t.pdf (225 pages, 1350413 bytes).